Human Needs Exam 3

Ace your homework & exams now with Quizwiz!

For a client with cirrhosis, deterioration of hepatic function is best indicated by:

2. difficulty in arousal.

A client with gastroenteritis is admitted to an acute care facility with severe dehydration and electrolyte imbalances. Diagnostic tests reveal the Norwalk virus as the cause of gastroenteritis. Based on this information, the nurse knows that:

2. enteric precautions must be continued.

The nurse is performing an assessment on a client who has developed a paralytic ileus. The client's bowel sounds will be:

2. hypoactive.

The nurse is caring for a client who underwent a subtotal gastrectomy 24 hours ago. The client has a nasogastric (NG) tube. The nurse should:

3. irrigate the NG tube gently with normal saline solution.

When planning care for a client with a small-bowel obstruction, the nurse should consider the primary goal to be:

3. maintaining fluid balance.

A client who received an inhalation anesthetic during GI surgery experiences severe shivering postoperatively. In addition to providing extra blankets, the nurse should:

3. provide oxygen as prescribed.

A client with left hemiparesis is having difficulty handling eating utensils. A nurse asks the physician to request a consult with which discipline?

4. Occupational therapy

Which outcome indicates effective client teaching to prevent constipation?

4. The client reports engaging in a regular exercise regimen.

A client has a newly created colostomy. After participating in counseling with the nurse and receiving support from the spouse, the client decides to change the colostomy pouch unaided. Which behavior suggests that the client is beginning to accept the change in body image?

4. The client touches the altered body part.

A client with hepatitis C develops liver failure and GI hemorrhage. The blood products that would most likely bring about hemostasis in the client are:

4. cryoprecipitate and fresh frozen plasma.

Which nursing diagnosis is a priority in the care of a patient with renal calculi? A. Acute pain B. Risk for constipation C. Deficient fluid volume D. Risk for powerlessness

A. Acute pain Urinary stones are associated with severe abdominal or flank pain. Deficient fluid volume is unlikely to result from urinary stones, whereas constipation is more likely to be an indirect consequence rather than a primary clinical manifestation of the problem. The presence of pain supersedes powerlessness as an immediate focus of nursing care.

Which of these nursing actions included in the plan of care for a patient who is receiving intermittent tube feedings through a percutaneous endoscopic gastrostomy (PEG) tube may be delegated to an LPN/LVN? a. Providing skin care to the area around the tube site b. Assessing the patient's nutritional status at least weekly c. Determining the need for the addition of water to the feedings d. Teaching the patient and family how to administer tube feedings

ANS: A LPN/LVN education and scope of practice include actions such as dressing changes and wound care. Patient teaching and complex assessments (such as patient nutrition and hydration status) require RN-level education and scope of practice.

Which information will the nurse include when teaching a patient with peptic ulcer disease about the effect of ranitidine (Zantac)? a. "Ranitidine absorbs the gastric acid." b. "Ranitidine decreases gastric acid secretion." c. "Ranitidine constricts the blood vessels near the ulcer." d. "Ranitidine covers the ulcer with a protective material."

ANS: B Ranitidine is a histamine-2 (H2) receptor blocker, which decreases the secretion of gastric acid. The response beginning, "Ranitidine constricts the blood vessels" describes the effect of vasopressin. The response "Ranitidine absorbs the gastric acid" describes the effect of antacids. The response beginning "Ranitidine covers the ulcer" describes the action of sucralfate (Carafate). DIF: Cognitive Level: Understand (comprehension) REF: 934 TOP: Nursing Process: Implementation MSC: NCLEX: Physiological Integrity

Which medications will the nurse teach the patient about whose peptic ulcer disease is associated with Helicobacter pylori? a. Sucralfate (Carafate), nystatin (Mycostatin), and bismuth (Pepto-Bismol) b. Amoxicillin (Amoxil), clarithromycin (Biaxin), and omeprazole (Prilosec) c. Famotidine (Pepcid), magnesium hydroxide (Mylanta), and pantoprazole (Protonix) d. Metoclopramide (Reglan), bethanechol (Urecholine), and promethazine (Phenergan)

ANS: B The drugs used in triple drug therapy include a proton pump inhibitor such as omeprazole and the antibiotics amoxicillin and clarithromycin. The other combinations listed are not included in the protocol for H. pylori infection. DIF: Cognitive Level: Understand (comprehension) REF: 946 TOP: Nursing Process: Planning MSC: NCLEX: Physiological Integrity

When preparing to teach an 82-year-old Hispanic patient who lives with an adult daughter about ways to improve nutrition, which action should the nurse take first? a. Ask the daughter about the patient's food preferences. b. Determine who shops for groceries and prepares the meals. c. Question the patient about how many meals per day are eaten. d. Assure the patient that culturally appropriate foods will be included.

ANS: B The family member who shops for groceries and cooks will be in control of the patient's diet, so the nurse will need to ensure that this family member is involved in any teaching or discussion about the patient's nutritional needs. The other information also will be assessed and used but will not be useful in meeting the patient's nutritional needs unless nutritionally appropriate foods are purchased and prepared.

A 22-year-old who is hospitalized with anorexia nervosa is 5 ft 5 in (163 cm) tall and weighs 90 pounds (41 kg). Laboratory tests reveal hypokalemia and iron-deficiency anemia. Which nursing diagnosis has the highest priority for the patient? a. Risk for activity intolerance related to anemia and weakness b. Risk for electrolyte imbalance related to poor eating patterns c. Ineffective health maintenance related to obsession with body image d. Imbalanced nutrition: less than body requirements related to refusal to eat

ANS: B The patient's hypokalemia may lead to life-threatening cardiac dysrhythmias. The other diagnoses also are appropriate for this patient but are not associated with immediate risk for fatal complications.

The nurse is administering IV fluid boluses and nasogastric irrigation to a patient with acute gastrointestinal (GI) bleeding. Which assessment finding is most important for the nurse to communicate to the health care provider? a. The bowel sounds are hyperactive in all four quadrants. b. The patient's lungs have crackles audible to the midchest. c. The nasogastric (NG) suction is returning coffee-ground material. d. The patient's blood pressure (BP) has increased to 142/84 mm Hg.

ANS: B The patient's lung sounds indicate that pulmonary edema may be developing as a result of the rapid infusion of IV fluid and that the fluid infusion rate should be slowed. The return of coffee-ground material in an NG tube is expected for a patient with upper GI bleeding. The BP is slightly elevated but would not be an indication to contact the health care provider immediately. Hyperactive bowel sounds are common when a patient has GI bleeding. DIF: Cognitive Level: Apply (application) REF: 957 OBJ: Special Questions: Prioritization TOP: Nursing Process: Assessment MSC: NCLEX: Physiological Integrity

24. A patient with a peptic ulcer who has a nasogastric (NG) tube develops sudden, severe upper abdominal pain, diaphoresis, and a very firm abdomen. Which action should the nurse take next? a. Irrigate the NG tube. b. Obtain the vital signs. c. Listen for bowel sounds. d. Give the ordered antacid.

ANS: B The patient's symptoms suggest acute perforation, and the nurse should assess for signs of hypovolemic shock. Irrigation of the NG tube, administration of antacids, or both would be contraindicated because any material in the stomach will increase the spillage into the peritoneal cavity. The nurse should assess the bowel sounds, but this is not the first action that should be taken.

When using a soft, silicone nasogastric tube for enteral feedings, the nurse will need to a. avoid giving medications through the feeding tube. b. flush the tubing after checking for residual volumes. c. administer continuous feedings using an infusion pump. d. replace the tube every 3 to 5 days to avoid mucosal damage.

ANS: B The soft silicone feeding tubes are small in diameter and can easily become clogged unless they are flushed after the nurse checks the residual volume. Either intermittent or continuous feedings can be given. The tubes are less likely to cause mucosal damage than the stiffer polyvinyl chloride tubes used for nasogastric suction and do not need to be replaced at certain intervals. Medications can be given through these tubes, but flushing after medication administration is important to avoid clogging

A 58-year-old patient has just been admitted to the emergency department with nausea and vomiting. Which information requires the most rapid intervention by the nurse? a. The patient has been vomiting for 4 days. b. The patient takes antacids 8 to 10 times a day. c. The patient is lethargic and difficult to arouse. d. The patient has undergone a small intestinal resection.

ANS: C A lethargic patient is at risk for aspiration, and the nurse will need to position the patient to decrease aspiration risk. The other information is also important to collect, but it does not require as quick action as the risk for aspiration. DIF: Cognitive Level: Apply (application) REF: 927 OBJ: Special Questions: Prioritization TOP: Nursing Process: Assessment MSC: NCLEX: Physiological Integrity

Which finding in the mouth of a patient who uses smokeless tobacco is suggestive of oral cancer? a. Bleeding during tooth brushing b. Painful blisters at the lip border c. Red, velvety patches on the buccal mucosa d. White, curdlike plaques on the posterior tongue

ANS: C A red, velvety patch suggests erythroplasia, which has a high incidence (greater than 50%) of progression to squamous cell carcinoma. The other lesions are suggestive of acute processes (e.g., gingivitis, oral candidiasis, herpes simplex). DIF: Cognitive Level: Understand (comprehension) REF: 929 TOP: Nursing Process: Assessment MSC: NCLEX: Physiological Integrity

The nurse is assessing a patient who had a total gastrectomy 8 hours ago. What information is most important to report to the health care provider? a. Absent bowel sounds b. Complaints of incisional pain c. Temperature 102.1° F (38.9° C) d. Scant nasogastric (NG) tube drainage

ANS: C An elevation in temperature may indicate leakage at the anastomosis, which may require return to surgery or keeping the patient NPO. The other findings are expected in the immediate postoperative period for patients who have this surgery. DIF: Cognitive Level: Apply (application) REF: 953 TOP: Nursing Process: Assessment MSC: NCLEX: Physiological Integrity

29. The health care provider prescribes antacids and sucralfate (Carafate) for treatment of a patient's peptic ulcer. The nurse will teach the patient to take a. antacids 30 minutes before the sucralfate. b. sucralfate at bedtime and antacids before meals. c. antacids after eating and sucralfate 30 minutes before eating. d. sucralfate and antacids together 30 minutes before each meal.

ANS: C Sucralfate is most effective when the pH is low and should not be given with or soon after antacid. Antacids are most effective when taken after eating. Administration of sucralfate 30 minutes before eating and antacids just after eating will ensure that both drugs can be most effective. The other regimens will decrease the effectiveness of the medications.

20. A 22-year-old patient with Escherichia coli O157:H7 food poisoning is admitted to the hospital with bloody diarrhea and dehydration. All of the following orders are received. Which order will the nurse question? a. Infuse lactated Ringer's solution at 250 mL/hr. b. Monitor blood urea nitrogen and creatinine daily. c. Administer loperamide (Imodium) after each stool. d. Provide a clear liquid diet and progress diet as tolerated.

ANS: C Use of antidiarrheal agents is avoided with this type of food poisoning. The other orders are appropriate.

The nurse will anticipate teaching a patient experiencing frequent heartburn about a. a barium swallow. b. radionuclide tests. c. endoscopy procedures. d. proton pump inhibitors.

ANS: D Because diagnostic testing for heartburn that is probably caused by gastroesophageal reflux disease (GERD) is expensive and uncomfortable, proton pump inhibitors are frequently used for a short period as the first step in the diagnosis of GERD. The other tests may be used but are not usually the first step in diagnosis. DIF: Cognitive Level: Apply (application) REF: 932-933 TOP: Nursing Process: Planning MSC: NCLEX: Physiological Integrity

During the postoperative period after surgery for a kidney transplant, the client's creatinine level is 3.1 mg/dL. What should the nurse do first in response to this laboratory result?

Assess for decreased urine output

A patient with a history of end-stage kidney disease secondary to diabetes mellitus has presented to the outpatient dialysis unit for his scheduled hemodialysis. Which assessments should the nurse prioritize before, during, and after his treatment? A. Level of consciousness B. Blood pressure and fluid balance C. Temperature, heart rate, and blood pressure D. Assessment for signs and symptoms of infection

B. Blood pressure and fluid balance Although all of the assessments are relevant to the care of a patient receiving hemodialysis, the nature of the procedure indicates a particular need to monitor the patient's blood pressure and fluid balance.

A 22-year-old patient's blood pressure at her physical done for her new job was 110/68. At the health fair two months later, her blood pressure is 154/96. What renal problem should the nurse be aware of that could contribute to this abrupt rise in blood pressure? A. Renal trauma B. Renal artery stenosis C. Renal vein thrombosis D. Benign nephrosclerosis

B. Renal artery stenosis Renal artery stenosis contributes to an abrupt rise in blood pressure, especially in people under 30 or over 50 years of age. Renal trauma usually has hematuria. Renal vein thrombosis causes flank pain, hematuria, fever, or nephrotic syndrome. Benign nephrosclerosis usually occurs in adults 30 to 50 years of age and is a result of vascular changes resulting from hypertension.

A 54-year-old patient admitted with diabetes mellitus, malnutrition, osteomyelitis, and alcohol abuse has a serum amylase level of 280 U/L and a serum lipase level of 310 U/L. To what diagnosis does the nurse attribute these findings? A. Malnutrition B. Osteomyelitis C. Alcohol abuse D. Diabetes mellitus

C. Alcohol Use The patient with alcohol abuse could develop pancreatitis as a complication, which would increase the serum amylase (normal 30-122 U/L) and serum lipase (normal 31-186 U/L) levels as shown.

A 9-year-old child with chronic kidney disease is undergoing peritoneal dialysis. For which associated complication should the nurse monitor the child?

Cloudy return dialysate

A patient asks the nurse about taking phentermine and topiramate (Qsymia) for weight loss. To avoid side effects, it is important for the nurse to determine whether the patient has a history of a. glaucoma. b. hypertension. c. valvular heart disease. d. irritable bowel disease.

Correct answer: a Rationale: Qsymia is a combination of phentermine and topiramate. It must not be used in patients with glaucoma or hyperthyroidism.

A 90-year-old healthy man is suffering from dysphagia. The nurse explains what age-related change of the GI tract is the most likely cause of his difficulty? a. Xerostomia b. Esophageal cancer c. Decreased taste buds d. Thinner abdominal wall

Correct answer: a Rationale: Xerostomia, decreased volume of saliva, leads to dry oral mucosa and dysphagia. Esophageal cancer is not an age-related change. Decreased taste buds and a thinner abdominal wall do not contribute to difficulty swallowing.

A patient is admitted to the hospital with left upper quadrant (LUQ) pain. What may be a possible source of the pain? a. Liver b. Pancreas c. Appendix d. Gallbladder

Correct answer: b Rationale: The pancreas is located in the left upper quadrant, the liver is in the right upper quadrant, the appendix is in the right lower quadrant, and the gallbladder is in the right upper quadrant.

What is a normal finding on physical examination of the abdomen? a. Auscultation of bruits b. Observation of visible pulsations c. Percussion of liver dullness in the left midclavicular line d. Palpation of the spleen 1 to 2 cm below the left costal margin

Correct answer: b Rationale: The pulsation of the aorta in the epigastric area is a normal finding. Bruits indicate that blood flow is abnormal, the liver is percussed in the right midclavicular line, and a normal spleen cannot be palpated.

An 85-year-old woman seen in the primary care provider's office for a well check complains of difficulty swallowing. What common effect of aging should the nurse assess for as a possible cause? a. Anosmia b. Xerostomia c. Hypochlorhydria d. Salivary gland tumor

Correct answer: b Rationale: Xerostomia (decreased saliva production), or dry mouth, affects many older adults and may be associated with difficulty swallowing (dysphagia). Anosmia is loss of sense of smell. Hypochlorhydria, a decrease in stomach acid, does not affect swallowing. Salivary gland tumors are not common.

The nurse monitors the laboratory results of the patient with protein-calorie malnutrition during treatment. Which result is an indication of improvement in the patient's condition? a. Decreased lymphocytes b. Increased serum potassium c. Increased serum transferrin d. Decreased serum prealbumin

Correct answer: c Rationale: Serum transferrin is a protein that is synthesized by the liver and used for iron transport and decreases when there is protein deficiency. An increase in the protein would indicate a more positive nitrogen balance with amino acids available for synthesis. Decreased lymphocytes and serum prealbumin are indicators of protein depletion and increased serum potassium shows continuing failure of the sodium-potassium pump.

Which explanation about weight reduction should be included when teaching the obese patient and her obese husband? a. Weight gain is caused by psychologic factors. b. Daily weighing is recommended to monitor weight loss. c. Fat is not burned until the glycogen-water pool is depleted. d. Men lose weight less quickly than women because they have a higher percentage of metabolically less-active fat.

Correct answer: c Rationale: With reducing diets that severely restrict carbohydrates, the body's glycogen stores become depleted within a few days. The glycogen normally binds to water in fat cells and it is this water loss that causes weight loss in the first few days. Fat is not burned until the glycogen-water pool is depleted. Although psychosocial components (i.e., using food for comfort or reward and inability to buy high-nutritional quality food) may have an influence on weight gain, these factors along with lack of physical exercise, underestimation of portion size, and genetics contribute to weight gain. Weekly weighing is recommended as a more reliable indicator of weight loss because daily weighing shows frequent fluctuation from retained water (including urine) and elimination of feces. Men are able to lose weight more quickly than women because women have a higher percentage of metabolically less-active fat.

A complete nutritional assessment including anthropometric measurements is important for the patient who a. has a BMI of 25.5 kg/m2. b. complains of frequent nocturia. c. reports a 5-year history of constipation. d. reports an unintentional weight loss of 10 lb in 2 months.

Correct answer: d Rationale: A loss of more than 5% of usual body weight over 6 months, whether intentional or unintentional, is a critical indicator for further assessment.

The nurse is reviewing the home medication list for a 44-year-old man admitted with suspected hepatic failure. Which medication could cause hepatotoxicity? a. Nitroglycerin b. Digoxin (Lanoxin) c. Ciprofloxacin (Cipro) d. Acetaminophen (Tylenol)

Correct answer: d Rationale: Many chemicals and drugs are potentially hepatotoxic (see Table 39-6) and result in significant patient harm unless monitored closely. For example, chronic high doses of acetaminophen and nonsteroidal antiinflammatory drugs (NSAIDs) may be hepatotoxic.

A patient is admitted to the hospital with a diagnosis of diarrhea with dehydration. The nurse recognizes that increased peristalsis resulting in diarrhea can be related to a. sympathetic inhibition. b. mixing and propulsion. c. sympathetic stimulation. d. parasympathetic stimulation.

Correct answer: d Rationale: Peristalsis is increased by parasympathetic stimulation.

The best nutritional therapy plan for a person who is obese is a. the Zone diet. b. the Atkins diet. c. Sugar Busters. d. foods from the basic food groups.

Correct answer: d Rationale: Restricted food intake is a cornerstone for any weight loss or maintenance program. A good weight loss plan should include foods from the basic food groups.

Which teaching points are important when providing information to a patient with metabolic syndrome (select all that apply)? a. Stop smoking. b. Monitor weight daily. c. Increase level of activity. d. Decrease saturated fat intake. e. Reduce weight and maintain lower weight. f. Check blood glucose each morning prior to eating.

Correct answers: a, c, d, e Rationale: Patients with metabolic syndrome need to lower their risk factors by reducing and maintaining weight, increasing physical activity, establishing healthy diet habits, and smoking cessation. Some patients with metabolic syndrome are diabetic and would need to monitor glucose levels frequently. When monitoring weight reduction, it is recommended to check weight weekly, not daily.

A community health nurse is conducting an initial assessment of a new patient. Which assessments should the nurse include when screening the patient for metabolic syndrome (select all that apply)? a. Blood pressure b. Resting heart rate c. Physical endurance d. Waist circumference e. Fasting blood glucose

Correct answers: a, d, e Rationale: The diagnostic criteria for metabolic syndrome include elevated blood pressure, fasting blood glucose, waist circumference, triglycerides, and HDL cholesterol. Resting heart rate and physical endurance are not part of the diagnostic criteria.

In preparing to care for the obese patient with cancer, what physiologic problems is this patient at a greater risk for having (select all that apply)? a. Tinnitus b. Fractures c. Sleep apnea d. Trousseau's sign e. Type 2 diabetes mellitus f. Gastroesophageal reflux disease (GERD)

Correct answers: c, e, f Rationale: Obese patients are at a higher risk for cancer, sleep apnea and sleep deprivation, type 2 diabetes mellitus, gastroesophageal reflux disease (GERD), nonalcoholic steatohepatits, osteoarthritis, and cardiovascular problems. The other options are not related to obesity.

A nurse teaches the signs of organ rejection to a client who had a kidney transplant. What should be included in the education?

Elevated blood pressure

A client has a kidney transplant. The nurse should monitor for which signs associated with rejection of the transplant? (Select all that apply.)

Fever, Oliguria, Weight gain

A nurse teaches a client who is scheduled for a kidney transplant about the need for immunosuppressive medications. The nurse determines that the client understands the teaching when the client states, "I must take these medications:

For the rest of my life."

before HD

Nurse should complete assessment of fluid status, condition of access, temperature, skin condition

Which of the following problems is expected in a client who is in end-stage renal failure? a) anemia b) thalassemia c) renal calculi d) hypotension

a) anemia

The client has been diagnosed to have glomerulonephritis. What should the nurse observe in the urine? a) blood b) pus c) white blood cells d) glucose

a) blood

The client with chronic renal failure has an indwelling abdominal catheter for peritoneal dialysis. The client spills water on the catheter dressing while bathing. The nurse should immediately: a) change the dressing b) reinforce the dressing c) flush the peritoneal dialysis catheter d) scrub the catheter with povidine-iodine

a) change the dressing - Clients with peritoneal dialysis catheters are at high risk for infection. A wet dressing is a conduit for bacteria to reach the catheter insertion site. The nurse ensures that the dressing is kept dry at all times. Reinforcing the dressing is not a safe practice to prevent infection in this circumstance. Flushing the catheter is not indicated. Scrubbing the catheter with povidone-iodine is done at the time of connection or disconnection of peritoneal dialysis.

The client has end-stage renal disease. He had undergone kidney transplant 5 days ago. Which of the following is the most important intervention for the client to prevent infection? a) observe asepsis b) increase fluid intake c) avoid clients with flu d) avoid crowded places

a) observe asepsis

The physician orders a combination of Sulfamethoxazole and Phenazopyridine hydrochloride (Azogantrisol) for a patient. Which therapeutic effect should this combination drug have: a) plain relief and a decreased WBC count b) equal fluid intake and output c) polyuria with reddish stain d) increased complaints of bladder spasm after 20 minutes

a) plain relief and a decreased WBC count

Number the following in the order of the phases of exchange in PD. Begin with 1 and end with 3. ____ a. Drain ____ b. Dwell ____ c. Inflow

a. 3; b. 2; c. 1

Which complication of chronic kidney disease is treated with erythropoietin (EPO)? a. Anemia b. Hypertension c. Hyperkalemia d. Mineral and bone disorder

a. Erythropoietin is used to treat anemia, as it stimulates the bone marrow to produce red blood cells.

Assessment findings suggestive of peritonitis include a. rebound abdominal pain b. a soft, distended abdomen c. dull, continuous abdominal pain d. observing that the patient is restless

a. rebound abdominal pain Rationale: With peritoneal irritation, the abdomen is hard, like a board, and the patient has severe abdominal pain that is worse with any sudden movement. The patient lies very still. Palpating the abdomen and releasing the hands suddenly causes sudden movement within the abdomen and severe pain. This is called rebound tenderness.

The client who has a history of gout also is diagnosed with urolithisis and the stones are determined to be of uric acid type. The nurse gives the client instructions in which foods to limit, including: a) milk b) liver c) apples d) carrots

b) liver

Three year old Carlo has been admitted to the pediatric unit with a tentative diagnosis of nephrotic syndrome: At Carlo's last check-up when he was 2 1/2 years old, his BP was 95/60, PR was 110/min and weight was 15 kg. Which unexpected assessment today would the nurse report to help the diagnosis? a) BP: 95/60 b) weight: 20 kg c) PR: 110 d) temp: 37 C

b) weight: 20 kg during the toddler period, the child gains 2.5 kg a year. Carlo has gained 5 kg in only 6 months. In nephrotic syndrome, this excessive weight gain is due to edema.

What is the most serious electrolyte disorder associated with kidney disease? a. Hypocalcemia b. Hyperkalemia c. Hyponatremia d. Hypermagnesemia

b. Hyperkalemia can lead to life-threatening dysrhythmias. Hypocalcemia leads to an accelerated rate of bone remodeling and potentially to tetany. Hyponatremia may lead to confusion. Elevated sodium levels lead to edema, hypertension, and heart failure. Hypermagnesemia may decrease reflexes, mental status, and blood pressure.

In replying to a patient's questions about the seriousness of her chronic kidney disease (CKD), the nurse knows that the stage of CKD is based on what? a. Total daily urine output b. Glomerular filtration rate c. Degree of altered mental status d. Serum creatinine and urea levels

b. Stages of chronic kidney disease are based on the GFR. No specific markers of urinary output, mental status, or azotemia classify the degree of chronic kidney disease (CKD).

A patient with ulcerative colitis undergoes the first phase of a total proctocolectomy with ileal pouch and anal anastomosis. On postoperative assessment of the patient, what should the nurse expect to find? a. A rectal tube set to low continuous suction b. A loop ileostomy with a plastic rod to hold it in place c. A colostomy stoma with an NG tube in place to provide pouch irrigations d. A permanent ileostomy stoma in the right lower quadrant of the abdomen

b. The initial procedure for a total proctocolectomy with ileal pouch and anal anastomosis includes a colectomy, rectal mucosectomy, ileal reservoir construction, ileoanal anastomosis, and a temporary ileostomy. A loop ileostomy is the most common temporary ileostomy and it may be held in place with a plastic rod for the first week. A rectal tube to suction is not indicated in any of the surgical procedures for ulcerative colitis. A colostomy is not used and an NG tube would not be used to irrigate the pouch. A permanent ileostomy stoma would be expected following a total proctocolectomy with a permanent ileostomy.

How is the most common form of malabsorption syndrome treated? a. Administration of antibiotics b. Avoidance of milk and milk products c. Supplementation with pancreatic enzymes d. Avoidance of gluten found in wheat, barley, oats, and rye

b. The most common type of malabsorption syndrome is lactose intolerance and it is managed by restricting the intake of milk and milk products. Antibiotics are used in cases of bacterial infections that cause malabsorption, pancreatic enzyme supplementation is used for pancreatic insufficiency, and restriction of gluten is necessary for control of adult celiac disease (celiac sprue, gluten-induced enteropathy).

The client in end-stage of renal failure had undergone kidney transplant. Which of the following assessment findings indicate kidney transplant rejection? a) increased urinary output, BUN = 15 mg/dL b) HCT = 50%, Hgb = 17 g/dl c) decreased urinary output, sudden weight gain d) decreased urinary output, sudden weight loss

c) decreased urinary output, sudden weight gain

Diagnosis of acute pyelonephritis has been established your nursing intervention includes the following except: a) provide health teaching and discharge planning b) administer antibiotic c) measure I and O d) provide adequate comfort and rest

c) measure I and O

What laboratory findings are expected in ulcerative colitis as a result of diarrhea and vomiting? a. Increased albumin c. Decreased Na+, K+, Mg+, Cl-, and HCO3 - b. Elevated white blood cells (WBCs) d. Decreased hemoglobin (Hgb) and hematocrit (Hct)

c. In the patient with ulcerative colitis, decreased Na+, K+, Mg+, Cl-, and HCO3 - are a result of diarrhea and vomiting. Hypoalbuminemia may be present. Elevated WBCs occur with toxic megacolon. Decreased hemoglobin (Hgb) and hematocrit (Hct) occur with bloody diarrhea.

The nurse should teach the patient with chronic constipation that which food has the highest dietary fiber? a. Peach c. Dried beans b. Popcorn d. Shredded wheat

c. Of the foods listed, dried beans contain the highest amount of dietary fiber and are an excellent source of soluble fiber. Bran and berries also have large amounts of fiber.

In the oliguric phase of renal failure, what is the most appropriate nursing diagnosis? a) fluid volume deficit b) activity intolerance c) ineffective breathing pattern d) fluid volume excess

d) fluid volume excess

A client has been admitted to the hospital with a diagnosis of acute glomerulonephritis. During history-taking the nurse first asks the client about a recent history of: a) bleeding ulcer b) deep vein thrombosis c) myocardial infarction d) streptococcal infection

d) streptococcal infection

Which serum laboratory value indicates to the nurse that the patient's CKD is getting worse? a. Decreased BUN b. Decreased sodium c. Decreased creatinine d. Decreased calculated glomerular filtration rate (GFR)

d. As GFR decreases, BUN and serum creatinine levels increase. Although elevated BUN and creatinine indicate that waste products are accumulating, the calculated GFR is considered a more accurate indicator of kidney function than BUN or serum creatinine.

When teaching the patient about the diet for diverticular disease, which foods should the nurse recommend? a. White bread, cheese, and green beans b. Fresh tomatoes, pears, and corn flakes c. Oranges, baked potatoes, and raw carrots d. Dried beans, All Bran (100%) cereal, and raspberries

d. Dried beans, All Bran (100%) cereal, and raspberries A high fiber diet is recommended for diverticular disease. Dried beans, All Bran (100%) cereal, and raspberries all have higher amounts of fiber than white bread, cheese, green beans, fresh tomatoes, pears, corn flakes, oranges, baked potatoes, and raw carrots.

drain

- 3rd phase of PD cycle - Lasts 15 to 30 minutes - May be facilitated by gently massaging abdomen or changing position

A client is evaluated for severe pain in the right upper abdominal quadrant, which is accompanied by nausea and vomiting. The physician diagnoses acute cholecystitis and cholelithiasis. For this client, which nursing diagnosis takes top priority?

1. Acute pain related to biliary spasms

When preparing a client for a hemorrhoidectomy, the nurse should take which action?

1. Administer an enema as ordered.

While obtaining a client's medication history, the nurse learns that the client takes ranitidine (Zantac), as prescribed, to treat a peptic ulcer. The nurse continues gathering medication history data to assess for potential drug interactions. The nurse should instruct the client to avoid taking a drug from which class with ranitidine?

1. Antacids

Which diagnostic test would be used first to evaluate a client with acute upper GI bleeding?

1. Endoscopy

A client is recovering from a small-bowel resection. To relieve pain, the physician prescribes meperidine (Demerol), 75 mg I.M. every 4 hours. How soon after administration should meperidine's onset of action occur?

2. 15 to 30 minutes

After admission for acute appendicitis, a client undergoes an appendectomy. He complains of moderate postsurgical pain for which the physician prescribes pentazocine (Talwin), 50 mg by mouth every 4 hours. How soon after administration of this drug can the nurse expect the client to feel relief?

2. 15 to 30 minutes

When a client resumes oral feedings after having gastric resection, the nurse watches for early manifestations of dumping syndrome. The vasomotor disturbances associated with this syndrome usually occur how soon after eating?

2. 5 to 30 minutes

The nurse is monitoring a client receiving paregoric to treat diarrhea for drug interactions. Which drugs can produce additive constipation when given with an opium preparation?

2. Anticholinergic drugs

Which condition is most likely to have a nursing diagnosis of Deficient fluid volume?

2. Pancreatitis

A client who has been treated for diverticulitis is being discharged on oral propantheline bromide (Pro-Banthine). The nurse should instruct the client to take the drug at which times?

3. 30 minutes before meals and at bedtime

The nurse caring for a client with small-bowel obstruction would plan to implement which nursing intervention first?

4. Administering I.V. fluids

Which medication should the nurse expect to administer to a client with constipation?

4. Docusate sodium (Colace)

The surgeon is discussing surgery with a client diagnosed with colon cancer. The client is visibly shaken over the possibility of a colostomy. Based on the client's response, the surgeon should collaborate with which health team member?

4. Enterostomal nurse

The urinalysis of a male patient reveals a high microorganism count. What data should the nurse use to determine the area of the urinary tract that is infected (select all that apply)? A. Pain location B. Fever and chills C. Mental confusion D. Urinary hesitancy E. Urethral discharge F. Post-void dribbling

A, E. Although all the manifestations are evident with urinary tract infections (UTIs), pain location is useful in differentiating among pyelonephritis, cystitis, and urethritis because flank pain is characteristic of pyelonephritis, but dysuria is characteristic of cystitis and urethritis. Urethral discharge is indicative of urethritis, not pyelonephritis or cystitis. Fever and chills and mental confusion are nonspecific indicators of UTIs. Urinary hesitancy and postvoid dribbling may occur with a UTI but may also occur with prostate enlargement in the male patient.

When providing discharge teaching for the patient after a laparoscopic cholecystectomy, what information should the nurse include? A. A lower-fat diet may be better tolerated for several weeks. B. Do not return to work or normal activities for 3 weeks. C. Bile-colored drainage will probably drain from the incision. D. Keep the bandages on and the puncture site dry until it heals.

A. A lower-fat diet may be better tolerated for several weeks. Although the usual diet can be resumed, a low-fat diet is usually better tolerated for several weeks following surgery. Normal activities can be gradually resumed as the patient tolerates. Bile-colored drainage or pus, redness, swelling, severe pain, and fever may all indicate infection. The bandage may be removed the day after surgery, and the patient can shower.

When caring for a patient with a biliary obstruction, the nurse will anticipate administering which vitamin supplements (select all that apply)? A. Vitamin A B. Vitamin D C. Vitamin E D. Vitamin K E. Vitamin B

A. Vitamin A B. Vitamin D C. Vitamin E D. Vitamin K Biliary obstruction prevents bile from entering the small intestine and thus prevents the absorption of fat-soluble vitamins. Vitamins A, D, E, and K are all fat-soluble and thus would need to be supplemented in a patient with biliary obstruction.

Vasopressin (Pitressin) 0.2 units/min infusion is prescribed for a patient with acute arterial gastrointestinal (GI) bleeding. The vasopressin label states vasopressin 20 units/50 mL normal saline. How many mL/min will the nurse infuse? ____________________

ANS: 0.5 There are 0.4 units/1 mL. An infusion of 0.5 mL/min will result in the patient receiving 0.2 mL/min as prescribed.

18. A patient who has had several episodes of bloody diarrhea is admitted to the emergency department. Which action should the nurse anticipate taking? a. Obtain a stool specimen for culture. b. Administer antidiarrheal medications. c. Teach about adverse effects of nonsteroidal anti-inflammatory drugs (NSAIDs). d. Provide education about antibiotic therapy.

ANS: A Patients with bloody diarrhea should have a stool culture for E. coli O157:H7. NSAIDs may cause occult blood in the stools, but not diarrhea. Antidiarrheal medications usually are avoided for possible infectious diarrhea to avoid prolonging the infection. Antibiotic therapy in the treatment of infectious diarrhea is controversial because it may precipitate kidney complications.

4. When the nurse is assessing the mouth of a patient who uses smokeless tobacco for signs of oral cancer, which finding will be of most concern? a. Bleeding during tooth brushing b. Painful blisters at the border of the lips c. Red, velvety patches on the buccal mucosa d. White, curdlike plaques on the posterior tongue

ANS: C A red, velvety patch suggests erythroplasia, which has a high incidence (greater than 50%) of progression to squamous cell carcinoma. The other lesions are suggestive of acute processes (gingivitis, oral candidiasis, and herpes simplex).

A patient with a body mass index (BMI) of 17 kg/m2 and a low albumin level is being admitted by the nurse. Which assessment finding will the nurse expect to find? a. Restlessness b. Hypertension c. Pitting edema d. Food allergies

ANS: C Edema occurs when serum albumin levels and plasma oncotic pressure decrease. The blood pressure and level of consciousness are not directly affected by malnutrition. Food allergies are not an indicator of nutritional status.

A 26-year-old woman has been admitted to the emergency department with nausea and vomiting. Which action could the RN delegate to unlicensed assistive personnel (UAP)? a. Auscultate the bowel sounds. b. Assess for signs of dehydration. c. Assist the patient with oral care. d. Ask the patient about the nausea.

ANS: C Oral care is included in UAP education and scope of practice. The other actions are all assessments that require more education and a higher scope of nursing practice. DIF: Cognitive Level: Apply (application) REF: 15-16 OBJ: Special Questions: Delegation TOP: Nursing Process: Planning MSC: NCLEX: Safe and Effective Care Environment

Which choice from the hospital menu indicates that the patient has understood the nurse's teaching about choosing high calorie, high protein foods? a. Baked fish with applesauce b. Beef noodle soup and canned corn c. Fresh vegetables with yogurt topping d. Fried chicken with potatoes and gravy

ANS: D Foods that are high in calories include fried foods and those covered with sauces. High protein foods include meat and dairy products. The other choices are lower in calories and protein.

35. Which of these nursing actions should the RN working in the emergency department delegate to nursing assistive personnel who help with the care of a patient who has been admitted with nausea and vomiting? a. Auscultate the bowel sounds. b. Assess for signs of dehydration. c. Ask the patient what precipitated the nausea. d. Assist the patient with oral care after vomiting.

ANS: D Oral care is included in nursing assistive personnel education and scope of practice. The other actions are all assessments that require more education and a higher scope of nursing practice.

A client with cirrhosis is ordered to have a daily measurement of his abdominal girth. Identify the anatomical landmark where the tape measure should be placed when obtaining this measurement.

Abdominal girth should be measured at the umbilicus to obtain the most accurate measurement.

A nurse is caring for a client with end-stage renal disease who has a mature arteriovenous (AV) fistula. What nursing care should be included in the client's plan of care? (Select all that apply.)

Auscultate for a bruit., Palpate the site to identify a thrill, Avoid drawing blood from the affected extremity.

The patient with right upper quadrant abdominal pain has an abdominal ultrasound that reveals cholelithiasis. What should the nurse expect to do for this patient? A. Prevent all oral intake. B. Control abdominal pain. C. Provide enteral feedings. D. Avoid dietary cholesterol.

B. Control abdominal pain. Patients with cholelithiasis can have severe pain, so controlling pain is important until the problem can be treated. NPO status may be needed if the patient will have surgery but will not be used for all patients with cholelithiasis. Enteral feedings should not be needed, and avoiding dietary cholesterol is not used to treat cholelithiasis

Which assessment finding is a consequence of the oliguric phase of AKI? A. Hypovolemia B. Hyperkalemia C. Hypernatremia D. Thrombocytopenia

B. Hyperkalemia In AKI the serum potassium levels increase because the normal ability of the kidneys to excrete potassium is impaired. Sodium levels are typically normal or diminished, whereas fluid volume is normally increased because of decreased urine output. Thrombocytopenia is not a consequence of AKI, although altered platelet function may occur in AKI.

When planning nutritional interventions for a healthy 83-year-old man, the nurse recognizes what factor is most likely to affect his nutritional status? a. Living alone on a fixed income b. Changes in cardiovascular function c. An increase in GI motility and absorption d. Snacking between meal, resulting in obesity

Correct answer: a Rationale: Socioeconomic conditions frequently have the greatest effect on the nutritional status of the healthy older adult. Limited income and social isolation can result in the "tea and toast" meals of the older adult. The other options do not interfere with nutritional status.

What contributes to increased protein-calorie needs? a. Surgery b. Vegan diet c. Lowered temperature d. Cultural or religious beliefs

Correct answer: a Rationale: With surgery a patient will recover more rapidly with a balanced nutritional status before the surgery and increased protein is needed for healing after the surgery. Following a vegan diet does not put the patient at risk of low protein intake. A lowered temperature will not cause increased protein need. Following religious and cultural beliefs would not be expected to affect an increased need for protein.

A patient has an elevated blood level of indirect (unconjugated) bilirubin. One cause of this finding is that a. the gallbladder is unable to contract to release stored bile. b. bilirubin is not being conjugated and excreted into the bile by the liver. c. the Kupffer cells in the liver are unable to remove bilirubin from the blood. d. there is an obstruction in the biliary tract preventing flow of bile into the small intestine.

Correct answer: b Rationale: Bilirubin is a pigment derived from the breakdown of hemoglobin and is insoluble in water. Bilirubin is bound to albumin for transport to the liver and is referred to as unconjugated. An indirect bilirubin determination is a measurement of unconjugated bilirubin, and the level may be elevated in hepatocellular and hemolytic conditions.

As gastric contents move into the small intestine, the bowel is normally protected from the acidity of gastric contents by the a. inhibition of secretin release. b. release of bicarbonate by the pancreas. c. release of pancreatic digestive enzymes. d. release of gastrin by the duodenal mucosa.

Correct answer: b Rationale: The hormone secretin stimulates the pancreas to secrete fluid with a high concentration of bicarbonate. This alkaline secretion enters the duodenum and neutralizes acid in the chyme.

What is an indication for parenteral nutrition that is not an appropriate indication for enteral tube feedings? a. Head and neck cancer b. Hypermetabolic states c. Malabsorption syndrome d. Protein-calorie malnutrition

Correct answer: c Rationale: In malabsorption syndrome, foods that are ingested into the intestinal tract cannot be digested or absorbed and tube feedings infused into the intestinal tract would not be absorbed. All of the other conditions can be treated with enteral or parenteral nutrition, depending on the patient's needs.

When caring for the patient with heart failure, the nurse knows that which gastrointestinal process is most dependent on cardiac output and may affect the patient's nutritional status? a. Ingestion b. Digestion c. Absorption d. Elimination

Correct answer: c Rationale: Substances that interface with the absorptive surfaces of the GI tract (primarily in the small intestine) diffuse across the intestinal membranes into intestinal capillaries and are then carried to other parts of the body for use in energy production. The cardiac output provides the blood flow for this absorption of nutrients to occur.

When teaching a patient about weight reduction diets, the nurse teaches the patient that an appropriate single serving of a food is a. a 6-inch bagel. b. 1 cup of chopped vegetables. c. a piece of cheese the size of three dice. d. a chicken breast the size of a deck of cards.

Correct answer: d Rationale: A chicken breast the size of a deck of cards is about 3 oz, a recommended portion size of meat. Other normal portions include a 3-inch bagel, 1/2 cup of chopped vegetables, and a piece of cheese the size of six dice.

A severely obese patient has undergone Roux-en-Y gastric bypass surgery. In planning postoperative care, the nurse anticipates that the patient a. may have severe diarrhea early in the postoperative period. b. will not be allowed to ambulate for 1 to 2 days postoperatively. c. will require nasogastric suction until the incision heals. d. may have only liquids orally, and in very limited amounts, during the postoperative period.

Correct answer: d Rationale: During the immediate postoperative period, water and sugar-free clear liquids are given (30 mL every 2 hours while the patient is awake).

What is a clinical manifestation of age-related changes in the GI system that the nurse may find in an older patient? a. Gastric hyperacidity b. Intolerance to fatty foods c. Yellowish tinge to the skin d. Reflux of gastric contents into the esophagus

Correct answer: d Rationale: There is decreased tone of the lower esophageal sphincter with again and regurgitation of gastric contents back into the esophagus occurs, causing heartburn and belching. There is a decrease in hydrochloric acid secretion with aging. Jaundice and intolerance to fatty foods are symptoms of liver or gallbladder disease and are not normal age-related findings.

Health risks associated with obesity include (select all that apply) a. colorectal cancer. b. rheumatoid arthritis. c. polycystic ovary syndrome. d. nonalcoholic steatohepatitis. e. systemic lupus erythematosus.

Correct answers: a, c, d Rationale: Health risks associated with obesity include cardiovascular disease (related to increased low-density lipoprotein levels, increased triglyceride levels, and decreased high-density lipoprotein levels), hypertension, sleep apnea, obesity hypoventilation syndrome, reduced chest wall compliance, increased work of breathing, decreased total lung capacity and functional residual capacity, type 2 diabetes mellitus (i.e., hyperinsulinemia and insulin resistance), osteoarthritis, hyperuricemia, gout, gastroesophageal reflux disease, gallstones, nonalcoholic steatohepatitis, fatty liver and cirrhosis, cancer (mainly breast, endometrial, kidney, colorectal, pancreas, esophagus, and gallbladder), psychosocial problems (employment, education, and health care), low self-esteem, withdrawal from social interactions, and major depression.

Lab results: Potassium: 5.8 mEq/L, Sodium: 140 meq/L, Calcium: 9.0 mg/dL, Blood pressure: 180/100 A client is admitted to a medical unit with the diagnosis of acute kidney failure. The nurse reviews the client's laboratory data, performs a physical assessment, and obtains the client's vital signs. What should the nurse conclude the client is most likely experiencing?

Hyperkalemia

A client who had a suprapubic prostatectomy returns from the post-anesthesia care unit and accidentally pulls out the urethral catheter. What should the nurse do first?

Notify the health care provider.

Lab results: Sodium 135 mEq/L, Potassium 6mEq/L, Hemoglobin 8.5 g/L, Creatinine clearance 20 ml/min An older adult client is admitted to the hospital with a diagnosis of chronic kidney disease. The nurse reviews the client's medical record. Which clinical finding is a priority to be communicated to the primary health care provider?

Potassium level

The client with continuous ambulatory peritoneal dialysis (CAPD) has cloudy dialysate. Which of the following is the best initial nursing action? a) send fluid to the laboratory for culture b) administer antibiotic c) do nothing, this is expected d) stop drainage of fluid

a) send fluid to the laboratory for culture cloudy diasylate indicates infection (peritonitis). Culture of the fluid must be done to determine the microorganism present.

The nurse explains to the patient undergoing ostomy surgery that the procedure that maintains the most normal functioning of the bowel is: a. a sigmoid colostomy b. a transverse colostomy c. a descending colostomy d. an ascending colostomy

a. a sigmoid colostomy Rationale: The more distal the ostomy is, the more the intestinal contents resemble feces eliminated from an intact colon and rectum. Output from a sigmoid colostomy resembles normally formed stool, and some patients are able to regulate emptying time so they do not need to wear a collection bag.

A patient undergoes peritoneal dialysis exchanges several times each day. What should the nurse plan to increase in the patient's diet? a) Fat b) Protein c) Calories d) Carbohydrates

b

The nurse is planning care for a 68-year-old patient with an abdominal mass and suspected bowel obstruction. Which factor in the patient's history increases the patient's risk for colorectal cancer? a. Osteoarthritis b. History of colorectal polyps c. History of lactose intolerance d. Use of herbs as dietary supplements

b. History of colorectal polyps A history of colorectal polyps places this patient at risk for colorectal cancer. This tissue can degenerate over time and become malignant. Osteoarthritis, lactose intolerance, and the use of herbs do not pose additional risk to the patient.

What information would have the highest priority to be included in preoperative teaching for a 68-year-old patient scheduled for a colectomy? a. How to care for the wound b. How to deep breathe and cough c. The location and care of drains after surgery d. Which medications will be used during surgery

b. How to deep breathe and cough Because anesthesia, an abdominal incision, and pain can impair the patient's respiratory status in the postoperative period, it is of high priority to teach the patient to cough and deep breathe. Otherwise, the patient could develop atelectasis and pneumonia, which would delay early recovery from surgery and hospital discharge. Care for the wound and location and care of the drains will be briefly discussed preoperatively, but done again with higher priority after surgery. Knowing which drugs will be used during surgery may not be meaningful to the patient and should be reviewed with the patient by the anesthesiologist.

In contrast to diverticulitis, the patient with diverticulosis: a. has rectal bleeding b. often has no symptoms c. has localized cramping pain d. frequently develops peritonitis

b. often has no symptoms Rationale: Many people with diverticulosis have no symptoms. Patients with diverticulitis have symptoms of inflammation. Diverticulitis can lead to obstruction or perforation.

The patient with CKD is brought to the emergency department with Kussmaul respirations. What does the nurse know about CKD that could cause this patient's Kussmaul respirations? a. Uremic pleuritis is occurring. b. There is decreased pulmonary macrophage activity. c. They are caused by respiratory compensation for metabolic acidosis. d. Pulmonary edema from heart failure and fluid overload is occurring.

c. Kussmaul respirations occur with severe metabolic acidosis when the respiratory system is attempting to compensate by removing carbon dioxide with exhalations. Uremic pleuritis would cause a pleural friction rub. Decreased pulmonary macrophage activity increases the risk of pulmonary infection. Dyspnea would occur with pulmonary edema.

Which of the following anti-hypertensive medications is contraindicated for clients with renal insufficiency? a) beta-adrenergic blockers b) calcium-channel blockers c) direct-acting vasodilators d) angiotensin-converting enzyme inhibitors

d) angiotensin-converting enzyme inhibitors

The client with chronic renal failure who is scheduled for hemodialysis this morning is due to receive a daily dose of enalapril (Vasotec). The nurse should plan to administer this medication: a) during dialysis b) just before dialysis c) the day after dialysis d) on return form dialysis

d) on return form dialysis

The nurse is preparing to administer a scheduled dose of docusate sodium (Colace) when the patient reports an episode of loose stool and does not want to take the medication. What is the appropriate action by the nurse? a. Write an incident report about this untoward event. b. Attempt to have the family convince the patient to take the ordered dose. c. Withhold the medication at this time and try to administer it later in the day. d. Chart the dose as not given on the medical record and explain in the nursing progress notes.

d. Chart the dose as not given on the medical record and explain in the nursing progress notes. Whenever a patient refuses medication, the dose should be charted as not given with an explanation of the reason documented in the nursing progress notes. In this instance, the refusal indicates good judgment by the patient, and the patient should not be encouraged to take it today.

On the first postoperative day the nurse is caring for a patient who has had a RouxenY gastric bypass procedure. Which assessment finding should be reported immediately to the surgeon? a. Use of patientcontrolled analgesia (PCA) several times an hour for pain b. Irritation and skin breakdown in skinfolds c. Bilateral crackles audible at both lung bases d. Emesis of bilecolored fluid past the nasogastric (NG) tube

d. Emesis of bilecolored fluid past the nasogastric (NG) tube ANS: D Vomiting with an NG tube in place indicates that the NG tube needs to be repositioned by the surgeon to avoid putting stress on the gastric sutures. The nurse should implement actions to decrease skin irritation and have the patient cough and deep breathe, but these do not indicate a need for rapid notification of the surgeon. Frequent

After receiving change-of-shift report, which patient should the nurse assess first? a. Patient who is scheduled for the drain phase of a peritoneal dialysis exchange b. Patient with stage 4 chronic kidney disease who has an elevated phosphate level c. Patient with stage 5 chronic kidney disease who has a potassium level of 3.4 mEq/L d. Patient who has just returned from having hemodialysis and has a heart rate of 124/min.

d. Patient who has just returned from having hemodialysis and has a heart rate of 124/min.

A client with chronic kidney disease is admitted to the hospital with severe infection and anemia. The client is depressed and irritable. The client's spouse asks the nurse about the anticipated plan of care. What is an appropriate nursing response?

"The intake of meat, eggs, and cheese will be restricted so the kidneys can clear the body of waste products."

A client with renal colic is scheduled for extracorporeal shock-wave lithotripsy. The night before the procedure, the client puts the call light on frequently and has many demands. Which would be an appropriate statement for the nurse to make?

"You are facing a new experience tomorrow; tell me what concerns you have."

dwell

- 2nd phase of PD cycle - aka equilibration - diffusion & osmosis occur btwn pts blood & peritoneal cavity - duration of dwell time varies depending on method of PD (can be mins - hrs)

To verify the placement of a gastric feeding tube, the nurse should perform at least two tests. One test requires instilling air into the tube with a syringe and listening with a stethoscope for air passing into the stomach. What is another test method?

1. Aspiration of gastric contents and testing for a pH less than 6

A client with constipation is prescribed an irrigating enema. Which steps should the nurse take when administering an enema?

1. Assist the client into the left-lateral Sims' position., 2. Lubricate the distal end of the rectal catheter., 6. Be sure to keep the solution container below 18" above bed level.

Which infections require contact precautions?

1. Clostridium difficile, 3. Methicillin-resistant staphylococcus aureus

A client is recovering from an ileostomy that was performed to treat inflammatory bowel disease. During discharge teaching, the nurse should stress the importance of:

1. increasing fluid intake to prevent dehydration.

A client with amebiasis, an intestinal infection, is prescribed metronidazole (Flagyl). When teaching the client about adverse reactions to this drug, the nurse should mention:

1. metallic taste.

A client with cholelithiasis has a gallstone lodged in the common bile duct. When assessing this client, the nurse expects to note:

1. yellow sclerae.

A client undergoes a barium swallow fluoroscopy that confirms gastroesophageal reflux disease (GERD). Based on this diagnosis, the client should be instructed to take which action?

2. Avoid caffeine and carbonated beverages., 4. Stop smoking., 5.Take antacids 1 hour and 3 hours after meals.

A 72-year-old client seeks help for chronic constipation. This is a common problem for elderly clients due to several factors related to aging. Which of the following is one such factor?

2. Decreased abdominal strength

The nurse is developing a plan of care for a client with hepatitis A. What is the main route of transmission of this hepatitis virus?

2. Feces

One day after undergoing a traditional cholecystectomy, a client is scheduled to stand at the bedside and walk. What should a nurse teach the client to do before standing and walking for the first time after surgery?

2. Flex her legs when moving to a sitting position.

An 86-year-old client with a history of atrial fibrillation takes 5 mg of warfarin (Coumadin) daily. Warfarin therapy makes the client at risk for which complications?

2. Hemorrhage, 3. Hepatitis, 5. Hematuria

As part of a routine screening for colorectal cancer, a client must undergo fecal occult blood testing. Which foods should the nurse instruct the client to avoid 48 to 72 hours before the test and throughout the collection period?

2. Red meat,. 3. Turnips, 4. Horseradish

The nurse is caring for a client with an endotracheal tube who receives enteral feedings through a feeding tube. Before each tube feeding, the nurse checks for tube placement in the stomach as well as residual volume. The purpose of the nurse's actions is to avoid:

2. aspiration.

Following a liver transplant a client develops ascites. The nurse should teach the client to:

2. brace the abdomen with a pillow during coughing.

A 68-year-old male is being admitted to the hospital with abdominal pain, anemia, and bloody stools. He complains of feeling weak and dizzy. He has rectal pressure and needs to urinate and move his bowels. The nurse should help him:

3. onto the bedpan.

A client with viral hepatitis A is being treated in an acute care facility. Because the client requires enteric precautions, the nurse should:

3. wear gloves when caring for the client and wash her hands after touching the client.

A nurse is irrigating an open wound of the abdomen. In which direction should she arrange for the irrigation solution to flow through the wound?

4. From the top inside of the wound, through the wound, and then out

A physician asks a nurse to witness an informed consent of a client scheduled for gastric bypass surgery. What should the nurse do?

4. Sign the consent only if she sees the client sign it.

After taking an antacid, the client asks the nurse where antacids act in the body. How should the nurse respond?

4. Stomach

The nurse is teaching a group of middle-aged men about peptic ulcers. When discussing risk factors for peptic ulcers, the nurse should mention:

4. alcohol abuse and smoking.

A client is diagnosed with shigellosis. The nurse teaches the client and family how the disease is transmitted and treated and discusses the need for enteric precautions. The nurse should explain that enteric precautions must be maintained:

4. until three fecal cultures are negative for Shigella.

A 78-year-old patient has Stage 3 CKD and is being taught about a low potassium diet. The nurse knows the patient understands the diet when the patient selects which foods to eat? A. Apple, green beans, and a roast beef sandwich B. Granola made with dried fruits, nuts, and seeds C. Watermelon and ice cream with chocolate sauce D. Bran cereal with ½ banana and milk and orange juice

A. Apple, green beans, and a roast beef sandwich When the patient selects an apple, green beans, and a roast beef sandwich, the patient demonstrates understanding of the low potassium diet. Granola, dried fruits, nuts and seeds, milk products, chocolate sauce, bran cereal, banana, and orange juice all have elevated levels of potassium, at or above 200 mg per 1/2 cup.

The patient with type 2 diabetes has a second UTI within one month of being treated for a previous UTI. Which medication should the nurse expect to teach the patient about taking for this infection? A. Ciprofloxacin (Cipro) B. Fosfomycin (Monurol) C. Nitrofurantoin (Macrodantin) D. Trimethoprim/sulfamethoxazole (Bactrim)

A. Ciprofloxacin (Cipro) This UTI is a complicated UTI because the patient has type 2 diabetes and the UTI is recurrent. Ciprofloxacin (Cipro) would be used for a complicated UTI. Fosfomycin (Monurol), nitrofurantoin (Macrodantin), and trimethoprim/sulfamethoxazole (Bactrim) should be used for uncomplicated UTIs.

19. A patient is hospitalized with vomiting of "coffee-ground" emesis. The nurse will anticipate preparing the patient for a. endoscopy. b. angiography. c. gastric analysis testing. d. barium contrast studies.

ANS: A Endoscopy is the primary tool for visualization and diagnosis of upper gastrointestinal (GI) bleeding. Angiography is used only when endoscopy cannot be done because it is more invasive and has more possible complications. Gastric analysis testing may help with determining the cause of gastric irritation, but it is not used for acute GI bleeding. Barium studies are helpful in determining the presence of gastric lesions, but not whether the lesions are actively bleeding.

A 66-year-old patient has a body mass index (BMI) of 31 kg/m2, a normal C-reactive protein level, and low transferrin and albumin levels. The nurse will plan patient teaching to increase the patient's intake of foods that are high in a. iron. b. protein. c. calories. d. carbohydrate.

ANS: B The patient's C-reactive protein and transferrin levels indicate low protein stores. The BMI is in the obese range, so increasing caloric intake is not indicated. The data do not indicate a need for increased carbohydrate or iron intake.

7. When admitting a patient with a stroke who is unconscious and unresponsive to stimuli, the nurse learns from the patient's family that the patient has a history of gastroesophageal reflux disease (GERD). The nurse will plan to do frequent assessments of the patient's a. apical pulse. b. bowel sounds. c. breath sounds. d. abdominal girth.

ANS: C Because GERD may cause aspiration, the unconscious patient is at risk for developing aspiration pneumonia. Bowel sounds, abdominal girth, and apical pulse will not be affected by the patient's stroke or GERD and do not require more frequent monitoring than the routine.

1. A patient with deep partial-thickness burns experiences severe pain associated with nausea during dressing changes. Which action will be most useful in decreasing the patient's nausea? a. The patient NPO for 2 hours before and after dressing changes. b. Avoid performing dressing changes close to the patient's mealtimes. c. Administer the prescribed morphine sulfate before dressing changes. d. Give the ordered prochlorperazine (Compazine) before dressing changes.

ANS: C Because the patient's nausea is associated with severe pain, it is likely that it is precipitated by stress and pain. The best treatment will be to provide adequate pain medication before dressing changes. The nurse should avoid doing painful procedures close to mealtimes, but nausea/vomiting that occur at other times also should be addressed. Keeping the patient NPO does not address the reason for the nausea and vomiting and will have an adverse effect on the patient's nutrition. Administration of antiemetics is not the best choice for a patient with nausea caused by pain.

A patient returned from a laparoscopic Nissen fundoplication for hiatal hernia 4 hours ago. Which assessment finding is most important for the nurse to address immediately? a. The patient is experiencing intermittent waves of nausea. b. The patient complains of 7/10 (0 to 10 scale) abdominal pain. c. The patient has absent breath sounds in the left anterior chest. d. The patient has hypoactive bowel sounds in all four quadrants.

ANS: C Decreased breath sounds on one side may indicate a pneumothorax, which requires rapid diagnosis and treatment. The nausea and abdominal pain should also be addressed but they are not as high priority as the patient's respiratory status. The patient's decreased bowel sounds are expected after surgery and require ongoing monitoring but no other action. DIF: Cognitive Level: Apply (application) REF: 935 OBJ: Special Questions: Prioritization TOP: Nursing Process: Assessment MSC: NCLEX: Physiological Integrity

14. The nurse will plan to teach the patient with newly diagnosed achalasia that a. a liquid or blenderized diet will be necessary. b. drinking fluids with meals should be avoided. c. endoscopic procedures may be used for treatment. d. lying down and resting after meals is recommended.

ANS: C Endoscopic and laparoscopic procedures are the most effective therapy for improving symptoms caused by achalasia. Patients are advised to drink fluid with meals. Keeping the head elevated after eating will improve esophageal emptying. A semisoft diet is recommended to improve esophageal emptying.

Which patient should the nurse assess first after receiving change-of-shift report? a. A patient with nausea who has a dose of metoclopramide (Reglan) due b. A patient who is crying after receiving a diagnosis of esophageal cancer c. A patient with esophageal varices who has a blood pressure of 92/58 mm Hg d. A patient admitted yesterday with gastrointestinal (GI) bleeding who has melena

ANS: C The patient's history and blood pressure indicate possible hemodynamic instability caused by GI bleeding. The data about the other patients do not indicate acutely life-threatening complications. DIF: Cognitive Level: Apply (application) REF: 956 OBJ: Special Questions: Prioritization

40. All of the following orders are received for a patient who has vomited 1500 mL of bright red blood. Which order will the nurse implement first? a. Insert a nasogastric (NG) tube and connect to suction. b. Administer intravenous (IV) famotidine (Pepcid) 40 mg. c. Draw blood for typing and crossmatching. d. Infuse 1000 mL of lactated Ringer's solution.

ANS: D Because the patient has vomited a large amount of blood, correction of hypovolemia and prevention of hypovolemic shock are the priorities. The other actions also are important to implement quickly but are not the highest priorities.

6. The nurse is assessing a patient with gastroesophageal reflux disease (GERD) who is experiencing increasing discomfort. Which patient statement indicates that additional patient education about GERD is needed? a. "I take antacids between meals and at bedtime each night." b. "I sleep with the head of the bed elevated on 4-inch blocks." c. "I quit smoking several years ago, but I still chew a lot of gum." d. "I eat small meals throughout the day and have a bedtime snack."

ANS: D GERD is exacerbated by eating late at night, and the nurse should plan to teach the patient to avoid eating at bedtime. The other patient actions are appropriate to control symptoms of GERD.

9. After the nurse teaches a patient with gastroesophageal reflux disease (GERD) about recommended dietary modifications, which diet choice for a snack 2 hours before bedtime indicates that the teaching has been effective? a. Chocolate pudding b. Glass of low-fat milk c. Peanut butter sandwich d. Cherry gelatin and fruit

ANS: D Gelatin and fruit are low fat and will not decrease lower esophageal sphincter (LES) pressure. Foods like chocolate are avoided because they lower LES pressure. Milk products increase gastric acid secretion. High-fat foods such as peanut butter decrease both gastric emptying and LES pressure.

The nurse is performing an admission assessment on a 20-year-old college student who is being admitted for electrolyte disorders of unknown etiology. Which assessment finding is most important to report to the health care provider? a. The patient's knuckles are macerated. b. The patient uses laxatives on a daily basis. c. The patient has a history of weight fluctuations. d. The patient's serum potassium level is 2.9 mEq/L.

ANS: D The low serum potassium level may cause life-threatening cardiac dysrhythmias and potassium supplementation is needed rapidly. The other information also will be reported because it suggests that bulimia may be the etiology of the patient's electrolyte disturbances, but it does not suggest imminent life-threatening complications.

31. A patient with a recent 20-pound unintended weight loss is diagnosed with stomach cancer. Which nursing action will be included in the plan of care? a. Refer the patient for hospice services. b. Infuse IV fluids through a central line. c. Teach the patient about antiemetic therapy. d. Offer supplemental feedings between meals.

ANS: D The patient data indicate a poor nutritional state and improvement in nutrition will be helpful in improving response to therapies such as surgery, chemotherapy, or radiation. Nausea and vomiting are not common clinical manifestations of stomach cancer. There is no indication that the patient requires hospice or IV fluid infusions.

A 73-year-old patient is diagnosed with stomach cancer after an unintended 20-pound weight loss. Which nursing action will be included in the plan of care? a. Refer the patient for hospice services. b. Infuse IV fluids through a central line. c. Teach the patient about antiemetic therapy. d. Offer supplemental feedings between meals.

ANS: D The patient data indicate a poor nutritional state and improvement in nutrition will be helpful in improving the response to therapies such as surgery, chemotherapy, or radiation. Nausea and vomiting are not common clinical manifestations of stomach cancer. There is no indication that the patient requires hospice or IV fluid infusions. DIF: Cognitive Level: Apply (application) REF: 953 TOP: Nursing Process: Planning MSC: NCLEX: Physiological Integrity

A nurse is caring for a client with end-stage renal disease. Which clinical indicators of end-stage renal disease should the nurse expect? (Select all that apply.)

Azotemia, Hypertension

A nurse is admitting a patient with the diagnosis of advanced renal carcinoma. Based upon this diagnosis, the nurse will expect to find what clinical manifestations as the "classic triad" occurring in patients with renal cancer? A. Fever, chills, flank pain B. Hematuria, flank pain, palpable mass C. Hematuria, proteinuria, palpable mass D. Flank pain, palpable abdominal mass, and proteinuria

B. Hematuria, flank pain, palpable mass There are no characteristic early symptoms of renal carcinoma. The classic manifestations of gross hematuria, flank pain, and a palpable mass are those of advanced disease.

As a component of the head-to-toe assessment of a patient who has been recently transferred to the clinical unit, the nurse is preparing to palpate the patient's kidneys. How should the nurse position the patient for this assessment? A. Prone B. Supine C. Seated at the edge of the bed D. Standing, facing away from the nurse

B. Supine To palpate the right kidney, the patient is positioned supine, and the nurse's left hand is placed behind and supports the patient's right side between the rib cage and the iliac crest. The right flank is elevated with the left hand, and the right hand is used to palpate deeply for the right kidney. The normal-sized left kidney is rarely palpable because the spleen lies directly on top of it.

Priority Decision: Following auscultation of the abdomen, what should the nurse's next action be? a. Lightly percuss over all four quadrants b. Have the patient empty his or her bladder c. Inspect perianal and anal areas for color, masses, rashes, and scars d. Perform deep palpation to delineate abdominal organs and masses

Correct answer: a Rationale: The abdomen should be assessed in the following sequence: inspection, auscultation, percussion, palpation. The patient should empty his or her bladder before assessment begins.

A 35-year-old man with a family history of adenomatous polyposis had a colonoscopy with removal of multiple polyps. Which signs and symptoms should the nurse teach the patient to report immediately? a. Fever and abdominal pain b. Flatulence and liquid stool c. Loudly audible bowel sounds d. Sleepiness and abdominal cramps

Correct answer: a Rationale: The patient should be taught to observe for signs of rectal bleeding and peritonitis. Fever, malaise, and abdominal pain and distention could indicate a perforated bowel with peritonitis.

A patient is receiving peripheral parenteral nutrition. The parenteral nutrition solution is completed before the new solution arrives on the unit. The nurse administers a. 20% intralipids. b. 5% dextrose solution. c. 0.45% normal saline solution. d. 5% lactated Ringer's solution

Correct answer: b Rationale: If a peripheral parenteral nutrition (PPN) formula bag empties before the next solution is available, a 5% dextrose solution (based on the amount of dextrose in the peripheral PN solution) may be administered to prevent hypoglycemia.

A 62-year-old woman patient is scheduled for a percutaneous transhepatic cholangiography to restore biliary drainage. The nurse discusses the patient's health history and is most concerned if the patient makes which statement? a."I am allergic to bee stings." b. "My tongue swells when I eat shrimp." c. "I have had epigastric pain for 2 months." d. "I have a pacemaker because my heart rate was slow."

Correct answer: b Rationale: The percutaneous transhepatic cholangiography procedure will include the use of radiopaque contrast medium. Patients allergic to shellfish and iodine are also allergic to contrast medium. Having a pacemaker will not affect the patient during this procedure. It would be expected that the patient would have some epigastric pain given the patient's condition.

A patient who is scheduled for surgery with general anesthesia in 1 hour is observed with a moist, but empty water glass in his hand. Which assessment finding may indicate that the patient drank a glass of water? a. Flat abdomen without movement upon inspection b. Tenderness at left upper quadrant upon palpation c. Easily heard, loud gurgling in the right upper quadrant d. High-pitched, hollow sounds in the left upper quadrant

Correct answer: c Rationale: If the patient drank water on an empty stomach, gurgling can be assessed without a stethoscope or assessed with auscultation. High-pitched, hollow sounds are tympanic and indicate an empty cavity. A flat abdomen and tenderness do not indicate that the patient drank a glass of water.

The nurse is providing care for a patient who has been admitted to the hospital for the treatment of nephrotic syndrome. What are priority nursing assessments in the care of this patient? A. Assessment of pain and level of consciousness B. Assessment of serum calcium and phosphorus levels C. Blood pressure and assessment for orthostatic hypotension D. Daily weights and measurement of the patient's abdominal girth

D. Daily weights and measurement of the patient's abdominal girth Peripheral edema is characteristic of nephrotic syndrome, and a key nursing responsibility in the care of patients with the disease is close monitoring of abdominal girth, weights, and extremity size. Pain, level of consciousness, and orthostatic blood pressure are less important in the care of patients with nephrotic syndrome. Abnormal calcium and phosphorus levels are not commonly associated with the diagnosis of nephrotic syndrome.

A nurse is assigned to care for a client with nephrotic syndrome. The nurse assesses which important parameter on a daily basis? a) weight b) albumin levels c) activity tolerance d) blood urea nitrogen (BUN) level

a) weight

Priority Decision: A patient on a medical unit has a potassium level of 6.8 mEq/L. What is the priority action that the nurse should take? a. Place the patient on a cardiac monitor. b. Check the patient's blood pressure (BP). c. Instruct the patient to avoid high-potassium foods. d. Call the lab and request a redraw of the lab to verify results.

a. Dysrhythmias may occur with an elevated potassium level and are potentially lethal. Monitor the rhythm while contacting the physician or calling the rapid response team. Vital signs should be checked. Depending on the patient's history and cause of increased potassium, instruct the patient about dietary sources of potassium; however, this would not help at this point. The nurse may want to recheck the value but until then the heart rhythm needs to be monitored.

Priority Decision: A patient is admitted to the emergency department with acute abdominal pain. What nursing intervention should the nurse implement first? a. Measurement of vital signs b. Administration of prescribed analgesics c. Assessment of the onset, location, intensity, duration, and character of the pain d. Physical assessment of the abdomen for distention, bowel sounds, and pigmentation changes

a. The patient with an acute abdomen may have significant fluid or blood loss into the abdomen and evaluation of blood pressure (BP) and heart rate (HR) should be the first intervention, followed by assessment of the abdomen and the nature of the pain. Analgesics should be used cautiously until a diagnosis can be determined so that symptoms are not masked.

Sodium polystyrene sulfonate (Kayexalate) is ordered for a patient with hyperkalemia. Before administering the medication, the nurse should assess the a. bowel sounds. b. blood glucose. c. blood urea nitrogen (BUN). d. level of consciousness (LOC).

a. bowel sounds. Rationale: Only sodium polystyrene sulfonate (Kayexalate) and dialysis actually remove potassium from the body. Never give this drug to a patient with a paralytic ileus because bowel necrosis can occur.

A client has been admitted to the hospital for urinary tract infection and dehydration. The nurse determines that the client has received adequate volume replacement if the blood urea nitrogen level drops to: a) 3 mg/dL b) 15 mg/dL c) 29 mg/dL d) 35 mg/dL

b) 15 mg/dL the normal blood urea nitrogen level is 8 to 25 mg/dL

The client is in end-stage renal failure (ESRD). Which of the following foods may be allowed for the client? a) banana b) apple c) carrot cake d) cantaloupe

b) apple APPLES ARE LOW IN POTASSIUM

The client had been diagnosed to have chronic renal failure. He had undergone hemodialysis for the first time. What signs and symptoms when experienced by the client suggest that he is experiencing disequilibrium syndrome? a) restlessness, hypotension, headache b) nausea and vomiting, hypertension, dizziness c) lethargy, hypotension, dizziness d) thachycardia, hypotension, headache

b) nausea and vomiting, hypertension, dizziness disequilibrium syndrome is caused by more rapid removal of waste products from the blood from the brain. This is due to the presence of blood-brain barrier. This causes increased intracranial pressure.

The client with chronic renal failure is undergoing peritoneal dialysis. He asks why the nurse monitors his blood glucose levels. Which of the following will be the most appropriate response by the nurse? a) I have to check if you have diabetes mellitus b) the dialysate contains glucose c) the procedure may lower your blood glucose levels d) it is a routine procedure for every client who undergoes the treatment

b) the dialysate contains glucose

When considering the following causes of acute abdomen, the nurse should know that surgery would be indicated for (select all that apply)? a. pancreatitis c. foreign-body perforation b. acute ischemic bowel d. pelvic inflammatory disease e. ruptured ectopic pregnancy f. ruptured abdominal aneurysm

b, c, e, f. An immediate surgical consult is needed for acute ischemic bowel, foreign-body perforation, ruptured ectopic pregnancy, or ruptured abdominal aneurysm. A diagnostic laparoscopy may be done or a laparotomy may be done to repair a ruptured abdominal aneurysm or remove the appendix. Surgery is not needed for pancreatitis or pelvic inflammatory disease, as these can be diagnosed and treated without surgery.

A patient who is given a bisacodyl (Dulcolax) suppository asks the nurse how long it will take to work. The nurse replies that the patient will probably need to use the bedpan or commode within which time frame after administration? a. 2-5 minutes b. 15-60 minutes c. 2-4 hours d. 6-8 hours

b. 15-60 minutes Bisacodyl suppositories usually are effective within 15 to 60 minutes of administration, so the nurse should plan accordingly to assist the patient to use the bedpan or commode.

A 61-year-old patient with suspected bowel obstruction had a nasogastric tube inserted at 4:00 AM. The nurse shares in the morning report that the day shift staff should check the tube for patency at what times? a. 7:00 AM, 10:00 AM, and 1:00 PM b. 8:00 AM, 12:00 PM, and 4:00 PM c. 9:00 AM and 3:00 PM d. 9:00 AM, 12:00 PM, and 3:00 PM

b. 8:00 AM, 12:00 PM, and 4:00 PM A nasogastric tube should be checked for patency routinely at 4-hour intervals. Thus if the tube were inserted at 4:00 AM, it would be due to be checked at 8:00 AM, 12:00 PM, and 4:00 PM.

On examining a patient 8 hours after having surgery to create a colostomy, what should the nurse expect to find? a. Hyperactive, high-pitched bowel sounds b. A brick-red, puffy stoma that oozes blood c. A purplish stoma, shiny and moist with mucus d. A small amount of liquid fecal drainage from the stoma

b. A normal new colostomy stoma should appear bright red, have mild to moderate edema, and have a small amount of bleeding or oozing of blood when touched. A purplish stoma indicates inadequate blood supply and should be reported. The colostomy will not have any fecal drainage for 2 to 4 days but there may be some earlier mucus or serosanguineous drainage. Bowel sounds after extensive bowel surgery will be diminished or absent

An important nursing intervention for a patient with a small intestinal obstruction who has an NG tube is to a. offer ice chips to suck PRN. b. provide mouth care every 1 to 2 hours. c. irrigate the tube with normal saline every 8 hours. d. keep the patient supine with the head of the bed elevated 30 degrees.

b. Mouth care should be done frequently for the patient with a small intestinal obstruction who has an NG tube because of vomiting, fecal taste and odor, and mouth breathing. No ice chips are allowed when a patient is NPO because of a bowel obstruction. The NG tube should be checked for patency and irrigated as ordered. The position of the patient should be one of comfort.

Which information will the nurse monitor in order to determine the effectiveness of prescribed calcium carbonate (Caltrate) for a patient with chronic kidney disease (CKD)? a. Blood pressure b. Phosphate level c. Neurologic status d. Creatinine clearance

b. Phosphate level Rationale: Normally, the kidneys control the levels of phosphate in your blood, and the balance between phosphate and calcium in your body. When your kidneys are not working, the level of phosphate in your blood can build up. Serum phosphate level must be lowered before calcium or vitamin D is administered.

A physician just told a patient that she has a volvulus. When the patient asks the nurse what this is, what is the best description for the nurse to give her? a. Bowel folding on itself c. Emboli of arterial supply to the bowel b. Twisting of bowel on itself d. Protrusion of bowel in weak or abnormal opening

b. Volvulus is the bowel twisting on itself. The bowel folding on itself is intussusception. Emboli of arterial blood supply to the bowel is vascular obstruction. Protrusion of bowel in a weak or abnormal opening is a hernia

Which clinical manifestations of inflammatory bowel disease are common to both patients with ulcerative colitis (UC) and Crohn's disease (select all that apply)? a. Restricted to rectum b. Strictures are common. c. Bloody, diarrhea stools d. Cramping abdominal pain e. Lesions penetrate intestine.

c & d Clinical manifestations of UC and Crohn's disease include bloody diarrhea, cramping abdominal pain, and nutritional disorders. Intestinal lesions associated with UC are usually restricted to the rectum before moving into the colon. Lesions that penetrate the intestine or cause strictures are characteristic of Crohn's disease.

A adult client has had laboratory work done as part of a routine physical examination. The nurse interprets that the client may have a mild degree of renal insufficiency if which of the following serum creatinine levels is noted? a) 0.2 mg/dlL b) 0.5 mg/dL c) 1.9 mg/dL d) 3.5 mg/dL

c) 1.9 mg/dL the normal serum creatinine level foadults is 0.6 to 1.3 mg/dL. The client with a mild degree of renal insufficiency would have a slight elevated level. A creatinie level of 0.2 mg/dL is low, and a level of 0.5 mg/dL is just below normal. A creeatinie level of 3.5 mg/dL may be associated with acute or chronic renal failure.

The home care nurse is making follow-up visits to a client following renal transplant. The nurse assesses the client for which signs of acute graft rejection? a) hypotension, graft tenderness, and anemia b) hypertension, oliguria, thirst, and hypothermia c) fever, hypertension, graft tenderness, and malaise d) fever, vomiting, hypotension, and copious amounts of dilute urine

c) fever, hypertension, graft tenderness, and malaise

The nurse has completed client teaching with the hemodialysis client about self-monitoring between hemodialysis treatments. The nurse determines that the best understands the information if the client states to record daily the: a) amount of activity b) pulse and respiratory rate c) intake and output and weight d) blood urea nitrogen and creatinine levels

c) intake and output and weight

A nurse is assessing a client who is diagnosed with cystitis. Which assessment finding is inconsistent with the typical clinical manifestations noted in this disorder? a) hematuria b) low back pain c) urinary retention d) burning on urination

c) urinary retention

An 83-year-old female patient was found lying on the bathroom floor. She said she fell 2 days ago and has not been able to take her heart medicine or eat or drink anything since then. What conditions could be causing prerenal AKI in this patient (select all that apply)? a. Anaphylaxis b. Renal calculi c. Hypovolemia d. Nephrotoxic drugs e. Decreased cardiac output

c, e. Because the patient has had nothing to eat or drink for 2 days, she is probably dehydrated and hypovolemic. Decreased cardiac output (CO) is most likely because she is older and takes heart medicine, which is probably for heart failure or hypertension. Both hypovolemia and decreased CO cause prerenal AKI. Anaphylaxis is also a cause of prerenal AKI but is not likely in this situation. Nephrotoxic drugs would contribute to intrarenal causes of AKI and renal calculi would be a postrenal cause of AKI.

The patient with chronic kidney disease is considering whether to use peritoneal dialysis (PD) or hemodialysis (HD). What are advantages of PD when compared to HD (select all that apply)? a. Less protein loss b. Rapid fluid removal c. Less cardiovascular stress d. Decreased hyperlipidemia e. Requires fewer dietary restrictions

c, e. Peritoneal dialysis is less stressful for the cardiovascular system and requires fewer dietary restrictions. Peritoneal dialysis actually contributes to more protein loss and increased hyperlipidemia. The fluid and creatinine removal are slower with peritoneal dialysis than hemodialysis.

During a routine screening colonoscopy on a 56-year-old patient, a rectosigmoidal polyp was identified and removed. The patient asks the nurse if his risk for colon cancer is increased because of the polyp. What is the best response by the nurse? a. "It is very rare for polyps to become malignant but you should continue to have routine colonoscopies." b. "Individuals with polyps have a 100% lifetime risk of developing colorectal cancer and at an earlier age than those without polyps." c. "All polyps are abnormal and should be removed but the risk for cancer depends on the type and if malignant changes are present." d. "All polyps are premalignant and a source of most colon cancer. You will need to have a colonoscopy every 6 months to check for new polyps."

c. Although all polyps are abnormal growths, the most common type of polyp (hyperplastic) is non-neoplastic, as are inflammatory, lipomas, and juvenile polyps. However, adenomatous polyps are characterized by neoplastic changes in the epithelium and most colorectal cancers appear to arise from these polyps. Only patients with a family history of familial adenomatous polyposis (FAP) have close to a 100% lifetime risk of developing colorectal cancer.

The patient is receiving the following medications. Which one is prescribed to relieve symptoms rather than treat a disease? a. Corticosteroids c. Antidiarrheal agents b. 6-Mercaptopurine d. Sulfasalazine (Azulfidine)

c. Antidiarrheal agents only relieve symptoms. Corticosteroids, 6-mercaptopurine, and sulfasalazine (Azulfidine) are used to treat and control inflammation with various diseases.

A 60-year-old African American patient is afraid she might have anal cancer. What assessment finding puts her at high risk for anal cancer? a. Alcohol use c. Human papillomavirus (HPV) b. Only one sexual partner d. Use of a condom with sexual intercourse

c. Human papillomavirus (HPV) is associated with about 80% of anal cancer cases. Other risk factors include multiple sexual partners, smoking, receptive anal sex, and HIV infection, as well as being female, age 60, and African American. The other options are not considered risk factors for anal cancer.

A client is being admitted to the hospital with a diagnosis of urolithiasis and ureteral colic. The nurse assesses the client for pain that is: a) dull and aching in the costovetebal area b) aching and camplike thoughout the abdomen c) sharp and radiating posteriorly to the spinal column d) excruciating, wavelike, and radiating toward the genitalia

d) excruciating, wavelike, and radiating toward the genitalia

Priority Decision: What is the most appropriate snack for the nurse to offer a patient with stage 4 CKD? a. Raisins b. Ice cream c. Dill pickles d. Hard candy

d. A patient with CKD may have unlimited intake of sugars and starches (unless the patient is diabetic) and hard candy is an appropriate snack and may help to relieve the metallic and urine taste that is common in the mouth. Raisins are a high-potassium food. Ice cream contains protein and phosphate and counts as fluid. Pickled foods have high sodium content.

The nurse is caring for a 68-year-old patient admitted with abdominal pain, nausea, and vomiting. The patient has an abdominal mass, and a bowel obstruction is suspected. The nurse auscultating the abdomen listens for which type of bowel sounds that are consistent with the patient's clinical picture? a. Low-pitched and rumbling above the area of obstruction b. High-pitched and hypoactive below the area of obstruction c. Low-pitched and hyperactive below the area of obstruction d. High-pitched and hyperactive above the area of obstruction

d. High-pitched and hyperactive above the area of obstruction Early in intestinal obstruction, the patient's bowel sounds are hyperactive and high-pitched, sometimes referred to as "tinkling" above the level of the obstruction. This occurs because peristaltic action increases to "push past" the area of obstruction. As the obstruction becomes complete, bowel sounds decrease and finally become absent.

The nurse would question the use of which cathartic agent in a patient with renal insufficiency? a. Bisacodyl (Dulcolax) b. Lubiprostone (Amitiza) c. Cascara sagrada (Senekot) d. Magnesium hydroxide (Milk of Magnesia)

d. Magnesium hydroxide (Milk of Magnesia) Milk of Magnesia may cause hypermagnesemia in patients with renal insufficiency. The nurse should question this order with the health care provider. Bisacodyl, lubiprostone, and cascara sagrada are safe to use in patients with renal insufficiency as long as the patient is not currently dehydrated.

A client has just been diagnosed with hepatitis A. On assessment, the nurse expects to note:

2. anorexia, nausea, and vomiting.

Hemodialysis (HD)

- an artificial membrane is used as the semipermeable membrane & is in contact w/ pts blood - the gradient is created by increasing pressure in the blood compartment (positive pressure) or decreasing pressure in the dialysate compartment (negative pressure)

PD complications

- exit site infection -> most common = staph - peritonitis - hernias - lower back probs -> due to increased intraabdominal pressure - bleeding - pulmonary complications - protein loss -> usually 0.5 g/L daily; or as high as 10-20g/day

What is the primary nursing diagnosis for a client with a bowel obstruction?

1. Deficient fluid volume

The nurse is assessing a client who complains of abdominal pain, nausea, and diarrhea. When examining the client's abdomen, which sequence should the nurse use?

2. Inspection, auscultation, percussion, and palpation

A client takes 30 ml of magnesium hydroxide and aluminum hydroxide with simethicone (Maalox TC) by mouth 1 hour and 3 hours after each meal and at bedtime for treatment of a duodenal ulcer. Why does the client take this antacid so frequently?

2. It has a short duration of action.

A client seeks medical attention after developing acute abdominal pain. Which action by the nurse would help ensure accurate auscultation of the client's bowel sounds?

2. Making sure the client's bladder is empty before auscultating

A client with advanced cirrhosis has a prothrombin time (PT) of 15 seconds, compared with a control time of 11 seconds. The nurse expects to administer:

2. phytonadione (Mephyton).

The nurse is teaching a client about malabsorption syndrome and its treatment. The client asks which part of the GI tract absorbs food. The nurse tells the client that products of digestion are absorbed mainly in the:

2. small intestine.

A client is undergoing an extensive diagnostic workup for a suspected GI problem. The nurse discovers that the client has a family history of ulcer disease. Which blood type also is a risk factor for duodenal ulcers?

4. Type O

A client is in the late stage of cirrhosis. When planning the client's diet, the nurse should focus on providing increased amounts of:

4. carbohydrate.

Priority Decision: Before selecting a weight reduction plan with an obese patient, what is most important for the nurse to first assess? a. The patient's motivation to lose weight b. The length of time that the patient has been obese c. Whether financial considerations will affect the patient's choices d. The patient's anthropometric measures of height, weight, BMI, waist-to-hip ratio, and skinfold thickness

Correct answer: a Rationale: Motivation is essential. Focus on the reasons for wanting to lose weight. The rest of the options will asset in planning the weight loss if the patient is motivated.

In preparing a patient for a colonoscopy, the nurse explains that a. a signed permit is not necessary. b. sedation may be used during the procedure. c. only one cleansing enema is necessary for preparation. d. a light meal should be eaten the day before the procedure.

Correct answer: b Rationale: Sedation is induced during a colonoscopy. A signed consent form is necessary for a colonoscopy. A cathartic or enema is administered the night before the procedure, and more than one enema may be necessary. Patients may need to be kept on clear liquids 1 to 2 days before the procedure.

A 40-year-old severely obese female patient with type 2 diabetes wants to lose weight. After learning about the surgical procedures, she thinks a combination of restrictive and malabsorptive surgery would be best. Which procedure should the nurse teach her about? a. Lipectomy b. Roux-en-Y gastric bypass c. Adjustable gastric banding d. Vertical sleeve gastrectomy

Correct answer: b Rationale: The Roux-en-Y gastric bypass is a common combination of restrictive (limiting the size of the stomach) and malabsorptive (less food is absorbed) surgery. Lipectomy is used to remove unsightly flabby folds of adipose tissue. Adjustable gastric banding is the most common restrictive procedure. Vertical sleeve gastrectomy is a restrictive procedure that preserves stomach function.

In developing a weight reduction program with a 45-year-old female patient who weighs 197 lb, the nurse encourages the patient to set a weight loss goal of how many pounds in 4 weeks? a. 1-2 b. 3-5 c. 4-8 d. 5-10

Correct answer: c Rationale: A realistic weight loss goal for patients is 1 to 2 lb/wk, which prevents the patient from becoming frustrated at not meeting weight loss goals.

What is the main underlying risk factor for metabolic syndrome? a. Age b. Heart disease c. Insulin resistance d. High cholesterol levels

Correct answer: c Rationale: Insulin resistance is the main underlying risk factor for metabolic syndrome. Aging is associated with metabolic syndrome. High cholesterol, hypertension, and increased clotting risk are characteristics of metabolic syndrome.

A patient received a small-bore nasogastric (NG) tube after a laryngectomy. What should be the nurse's priority intervention before starting the enteral feeding? a. Aspiration b. Auscultation of air c. Set head of bed at 40 degrees. d. Verify NG tube placement on x-ray.

Correct answer: d Rationale: It is imperative to ensure that an NG tube is situated in the GI tract rather than the patient's lungs. When an NG tube has been recently inserted, it is important to confirm this placement with an x-ray that will identify the tube's radiopaque tip. Aspiration and air auscultation may not differentiate between gastric and respiratory placement of the tube. The head of bed elevated at least 30 degrees is to prevent aspiration. To determine the maintenance of the feeding tube's proper position, the exit site of the tube is marked at the time of the x-ray and the external portion measured to allow for assessment of a change position with a change in the length of the tube.

A nurse is reviewing the laboratory reports of a client with a diagnosis of end-stage renal disease. What test result should the nurse anticipate?

Potassium of 6.3 mEq/L

The client hemodialyzed suddenly becomes short of breath and complains of chest pain. The client is tachycardic, pale and anxious. The nurse suspects air embolism. The priority action for the nurse is to: a) discontinue dialysis and notify the physician b) monitor vital signs every 15 minutes for the next hour c) continue dialysis at a slower rate after checking the lines for air d) bolus the client with 500 ml of normal saline to break up the air embolus

a) discontinue dialysis and notify the physician

Metabolic acidosis occurs in the oliguric phase of AKI as a result of impairment of a. ammonia synthesis. b. excretion of sodium. c. excretion of bicarbonate. d. conservation of potassium.

a. Metabolic acidosis occurs in AKI because the kidneys cannot synthesize ammonia or excrete acid products of metabolism, resulting in an increased acid load. Sodium is lost in urine because the kidneys cannot conserve sodium. Impaired excretion of potassium results in hyperkalemia. Bicarbonate is normally generated and reabsorbed by the functioning kidney to maintain acid/base balance.

The wound, ostomy, and continence (WOC) nurse selects the site where the ostomy will be placed. What should be included in the consideration for the site? a. The patient must be able to see the site. b. Outside the rectus muscle area is the best site. c. It is easier to seal the drainage bag to a protruding area. d. The ostomy will need irrigation, so area should not be tender.

a. The patient must be able to see the site. In selection of the ostomy site, the WOC nurse will want a site visible to the patient so the patient can take care of it, within the rectus muscle to avoid hernias, and on a flat surface to more easily create a good seal with the drainage bag.

A colectomy is scheduled for a 38-year-old woman with ulcerative colitis. The nurse should plan to include what prescribed measure in the preoperative preparation of this patient? a. Instruction on irrigating a colostomy b. Administration of a cleansing enema c. A high-fiber diet the day before surgery d. Administration of IV antibiotics for bowel preparation

b. Administration of a cleansing enema Preoperative preparation for bowel surgery typically includes bowel cleansing with antibiotics, such as oral neomycin and cleansing enemas, including Fleet enemas. Instructions to irrigate the colostomy will be done postoperatively. Oral antibiotics are given preoperatively, and an IV antibiotic may be used in the OR. A clear liquid diet will be used the day before surgery with the bowel cleansing.

Which of the following complaints is common in a client with pyelonephritis? a) right upper quadrant pain b) left upper quadrant pain c) pain at the costovertebral region d) pain at the suprapubic region

c) pain at the costovertebral region

A patient with AKI is a candidate for continuous renal replacement therapy (CRRT). What is the most common indication for use of CRRT? a. Azotemia b. Pericarditis c. Fluid overload d. Hyperkalemia

c. Continuous renal replacement therapy (CRRT) is indicated for the patient with AKI as an alternative or adjunct to hemodialysis to slowly remove solutes and fluid in the hemodynamically unstable patient. It is especially useful for treatment of fluid overload, but hemodialysis is indicated for treatment of hyperkalemia, pericarditis, or other serious effects of uremia.

The patient has peritonitis, which is a major complication of appendicitis. What treatment will the nurse plan to include? a. Peritoneal lavage c. IV fluid replacement b. Peritoneal dialysis d. Increased oral fluid intake

c. IV fluid replacement along with antibiotics, NG suction, analgesics, and surgery would be expected. Peritoneal lavage may be used to determine abdominal trauma. Peritoneal dialysis would not be performed. Oral fluids would be avoided with peritonitis.

Peritoneal dialysis (PD)

- peritoneal membrane acts as semipermeable membrane - excess fluid is removed by increasing the osmolality of the dialysate by adding glucose - access obtained by inserting catehter through anterior abd wall - usually done via surgery - preparation: empty bladder & bowel, weigh pt, signed consent - Tenckhoff catheter is used

Continuous venovenous hemodialysis (CVVHD)

- type of continual renal replacement therapy (CRRT) - uses dialysate - dialysate bags attached to distal end of hemofilter - Fluid pumped countercurrent to blood flow - Ideal treatment for patient who needs fluid/solute control but cannot tolerate rapid fluid shifts with HD

A client, age 82, is admitted to an acute care facility for treatment of an acute flare-up of a chronic GI condition. In addition to assessing the client for complications of the current illness, the nurse monitors for age-related changes in the GI tract. Which age-related change increases the risk of anemia?

1. Atrophy of the gastric mucosa

The physician orders morphine for a client who complains of postoperative abdominal pain. For maximum pain relief, when should the nurse anticipate administering morphine?

1. Before the pain becomes severe

A client with inflammatory bowel disease undergoes an ileostomy. On the first day after surgery, the nurse notes that the client's stoma appears dusky. How should the nurse interpret this finding?

1. Blood supply to the stoma has been interrupted.

Which nursing intervention should the nurse perform for a client receiving enteral feedings through a gastrostomy tube?

1. Change the tube feeding solutions and tubing at least every 24 hours.

A client with mild diarrhea, fever, and abdominal discomfort is being evaluated for inflammatory bowel disease (IBD). Which statement about IBD is true?

1. Diarrhea is the most common sign of IBD.

A client is preparing to undergo abdominal paracentesis. Which nursing interventions should be performed before the procedure?

1. Explain the procedure to the client., 2. Make sure informed consent was obtained., 3. Instruct the client to void.

The nurse is teaching a client how to irrigate his stoma. Which action indicates that the client needs more teaching?

1. Hanging the irrigation bag 24" to 36" (60 to 90 cm) above the stoma

While a client is being prepared for discharge, the nasogastric (NG) feeding tube becomes clogged. To remedy this problem and teach the client's family how to handle it at home, what should the nurse do?

1. Irrigate the tube with cola

A client is receiving a cleansing enema. During the procedure, the client reports abdominal cramping. What should the nurse do?

1. Lower the fluid bag so that the instillation slows.

A nurse is assigned the care of six clients and has the aid of a nursing assistant. Which task is appropriate for the nurse to delegate to the nursing assistant?

1. Measuring and recording nasogastric tube output

A nurse is caring for a client with an ileostomy. What is the most common complication of this procedure?

1. Peristomal skin irritation

When caring for a client who has had constipation for 4 days, what should be the nurse's primary client care concern?

1. Promoting defecation

A client is admitted to the health care facility with abdominal pain, a low-grade fever, abdominal distention, and weight loss. The physician diagnoses acute pancreatitis. What is the primary goal of nursing care for this client?

1. Relieving abdominal pain

A client with severe inflammatory bowel disease is receiving total parenteral nutrition (TPN). When monitoring TPN, the nurse must take care to maintain the prescribed flow rate because giving TPN too rapidly may cause:

1. hyperglycemia.

While preparing a client for cholecystectomy, the nurse explains that incentive spirometry will be used after surgery primarily to:

1. increase respiratory effectiveness.

A 32-year-old male client with appendicitis is experiencing severe abdominal pain. An abdominal X-ray film reveals intraperitoneal air. The nurse should prepare the client for:

1. surgery.

A 58-year-old client with osteoarthritis is admitted to the hospital with peptic ulcer disease. Which findings are commonly associated with peptic ulcer disease?

2. History of nonsteroidal anti-inflammatory drug (NSAID) use, 3. Epigastric pain that's relieved by antacids, 5. Nausea and weight loss

The nurse is assessing a client who is receiving total parenteral nutrition (TPN). Which finding suggests that the client has developed hyperglycemia?

2. Increased urine output

A client with severe abdominal pain is being evaluated for appendicitis. What is the most common cause of appendicitis?

2. Obstruction of the appendix

When preparing a client, age 50, for surgery to treat appendicitis, the nurse assists in formulating a nursing diagnosis of Risk for infection related to inflammation, perforation, and surgery. What is the rationale for choosing this nursing diagnosis?

2. Obstruction of the appendix reduces arterial flow, leading to ischemia, inflammation, and rupture of the appendix

When caring for a client with acute pancreatitis, the nurse should use which comfort measure?

2. Positioning the client on the side with the knees flexed

A client is scheduled for bowel resection with anastomosis involving the large intestine. Because of the surgical site, the nurse assists in formulating the nursing diagnosis of Risk for infection. To complete the nursing diagnosis statement, which "related-to" phrase should be added?

2. Related to the presence of bacteria at the surgical site

After undergoing a liver biopsy, the client should be placed in which position?

2. Right lateral decubitus position

As a result of a viral infection, a client develops gastroenteritis. The physician prescribes kaolin and pectin mixture (Kaopectate), 60 ml by mouth after each loose bowel movement, up to eight doses daily. The client asks the nurse how soon the medication will take effect. How should the nurse respond?

2. Within 30 minutes

For a client who must undergo colon surgery, the physician orders preoperative cleansing enemas and neomycin sulfate (Mycifradin). The rationale for neomycin use in this client is to:

2. decrease the intestinal bacteria count.

When evaluating a client for complications of acute pancreatitis, the nurse would observe for:

2. decreased urine output.

A client is admitted with suspected cirrhosis. During assessment, the nurse is most likely to detect:

2. muscle wasting.

A client with peptic ulcer disease is prescribed aluminum-magnesium complex (Riopan). When teaching about this antacid preparation, the nurse should instruct the client to take it with:

2. water.

When caring for a client with hepatitis B, the nurse should monitor closely for the development of which finding associated with a decrease in hepatic function?

4. Irritability and drowsiness

While palpating a client's right upper quadrant, the nurse would expect to find which of the following structures?

4. Liver

A client is admitted to the health care facility with nausea, vomiting, and abdominal cramps and distention. Which test result is most significant?

4. Serum potassium level of 3 mEq/L

The nurse is caring for a client who underwent a subtotal gastrectomy. To manage dumping syndrome, the nurse should advise the client to:

4. drink liquids only between meals.

The physician prescribes lactulose (Cephulac), 30 ml by mouth three times daily, when a client with cirrhosis develops an increased serum ammonia level. To evaluate the effectiveness of lactulose, the nurse should monitor:

4. level of consciousness (LOC).

Alterations in hepatic blood flow resulting from a drug interaction also can affect:

4. metabolism and excretion.

The patient with cirrhosis is being taught self-care. Which statement indicates the patient needs more teaching? A. "If I notice a fast heart rate or irregular beats, this is normal for cirrhosis." B. "I need to take good care of my belly and ankle skin where it is swollen." C. "A scrotal support may be more comfortable when I have scrotal edema." D. "I can use pillows to support my head to help me breathe when I am in bed."

A. "If I notice a fast heart rate or irregular beats, this is normal for cirrhosis." If the patient with cirrhosis experiences a fast or irregular heart rate, it may be indicative of hypokalemia and should be reported to the health care provider, as this is not normal for cirrhosis. Edematous tissue is subject to breakdown and needs meticulous skin care. Pillows and a semi-Fowler's or Fowler's position will increase respiratory efficiency. A scrotal support may improve comfort if there is scrotal edema.

The nurse is caring for a woman recently diagnosed with viral hepatitis A. Which individual should the nurse refer for an immunoglobin (IG) injection? B. A friend who delivers meals to the patient and family each week C. A relative with a history of hepatitis A who visits the patient daily D. A child living in the home who received the hepatitis A vaccine 3 months ago

A. A caregiver who lives in the same household with the patient IG is recommended for persons who do not have anti-HAV antibodies and are exposed as a result of close contact with persons who have HAV or foodborne exposure. Persons who have received a dose of HAV vaccine more than 1 month previously or who have a history of HAV infection do not require IG.

Eight months after the delivery of her first child, a 31-year-old woman has sought care because of occasional incontinence that she experiences when sneezing or laughing. Which measure should the nurse first recommend in an attempt to resolve the woman's incontinence? A. Kegel exercises B. Use of adult incontinence pads C. Intermittent self-catheterization D. Dietary changes including fluid restriction

A. Kegel exercises Patients who experience stress incontinence frequently benefit from Kegel exercises (pelvic floor muscle exercises). The use of incontinence pads does not resolve the problem, and intermittent self-catheterization would be a premature recommendation. Dietary changes are not likely to influence the patient's urinary continence.

A patient with type 2 diabetes and cirrhosis asks the nurse if it would be okay to take silymarin (milk thistle) to help minimize liver damage. The nurse responds based on what knowledge? A. Milk thistle may affect liver enzymes and thus alter drug metabolism. B. Milk thistle is generally safe in recommended doses for up to 10 years. C. There is unclear scientific evidence for the use of milk thistle in treating cirrhosis. D. Milk thistle may elevate the serum glucose levels and is thus contraindicated in diabetes.

A. Milk thistle may affect liver enzymes and thus alter drug metabolism. There is good scientific evidence that there is no real benefit from using milk thistle to protect the liver cells from toxic damage in the treatment of cirrhosis. Milk thistle does affect liver enzymes and thus could alter drug metabolism. Therefore patients will need to be monitored for drug interactions. It is noted to be safe for up to 6 years, not 10 years, and it may lower, not elevate, blood glucose levels.

The patient has a form of glomerular inflammation that is progressing rapidly. She is gaining weight, and the urine output is steadily declining. What is the priority nursing intervention? A. Monitor the patient's cardiac status. B. Teach the patient about hand washing. C. Obtain a serum specimen for electrolytes. D. Increase direct observation of the patient.

A. Monitor the patient's cardiac status. The nurse's priority is to monitor the patient's cardiac status. With the rapidly progressing glomerulonephritis, renal function begins to fail and fluid, potassium, and hydrogen retention lead to hypervolemia, hyperkalemia, and metabolic acidosis. Excess fluid increases the workload of the heart, and hyperkalemia can lead to life-threatening dysrhythmias. Teaching about hand washing and observation of the patient are important nursing interventions but are not the priority. Electrolyte measurement is a collaborative intervention that will be done as ordered by the health care provider.

The patient with cirrhosis has an increased abdominal girth from ascites. The nurse should know that this fluid gathers in the abdomen for which reasons (select all that apply)? A. There is decreased colloid oncotic pressure from the liver's inability to synthesize albumin. B. Hyperaldosteronism related to damaged hepatocytes increases sodium and fluid retention. C. Portal hypertension pushes proteins from the blood vessels, causing leaking into the peritoneal cavity. D. Osmoreceptors in the hypothalamus stimulate thirst, which causes the stimulation to take in fluids orally. E. Overactivity of the enlarged spleen results in increased removal of blood cells from the circulation, which decreases the vascular pressure.

A. There is decreased colloid oncotic pressure from the liver's inability to synthesize albumin. B. Hyperaldosteronism related to damaged hepatocytes increases sodium and fluid retention. C. Portal hypertension pushes proteins from the blood vessels, causing leaking into the peritoneal cavity. The ascites related to cirrhosis are caused by decreased colloid oncotic pressure from the lack of albumin from liver inability to synthesize it and the portal hypertension that shifts the protein from the blood vessels to the peritoneal cavity, and hyperaldosteronism which increases sodium and fluid retention. The intake of fluids orally and the removal of blood cells by the spleen do not directly contribute to ascites

When caring for a patient with liver disease, the nurse recognizes the need to prevent bleeding resulting from altered clotting factors and rupture of varices. Which nursing interventions would be appropriate to achieve this outcome (select all that apply)? A. Use smallest gauge needle possible when giving injections or drawing blood. B. Teach patient to avoid straining at stool, vigorous blowing of nose, and coughing. C. Advise patient to use soft-bristle toothbrush and avoid ingestion of irritating food. D. Apply gentle pressure for the shortest possible time period after performing venipuncture. E. Instruct patient to avoid aspirin and NSAIDs to prevent hemorrhage when varices are present.

A. Use smallest gauge needle possible when giving injections or drawing blood. B. Teach patient to avoid straining at stool, vigorous blowing of nose, and coughing. C. Advise patient to use soft-bristle toothbrush and avoid ingestion of irritating food. E. Instruct patient to avoid aspirin and NSAIDs to prevent hemorrhage when varices are present. Using the smallest gauge needle for injections will minimize the risk of bleeding into the tissues. Avoiding straining, nose blowing, and coughing will reduce the risk of hemorrhage at these sites. The use of a soft-bristle toothbrush and avoidance of irritating food will reduce injury to highly vascular mucous membranes. The nurse should apply gentle but prolonged pressure to venipuncture sites to minimize the risk of bleeding. Aspirin and NSAIDs should not be used in patients with liver disease because they interfere with platelet aggregation, thus increasing the risk for bleeding

COMPLETION 1. While caring for a comatose patient who is receiving continuous enteral nutrition through a soft nasogastric tube, the nurse notes the presence of new crackles in the patient's lungs. In which order will the nurse take the following actions? Put a comma and space between each answer choice (a, b, c, d, etc.) ____________________ a. Turn off the tube feeding. b. Obtain the patient's oxygen saturation. c. Check the tube feeding residual volume. d. Notify the patient's health care provider.

ANS: A, B, C, D The assessment data indicate that aspiration may have occurred, and the nurse's first action should be to turn off the tube feeding to avoid further aspiration. The next action should be to check the oxygen saturation because this may indicate the need for immediate respiratory suctioning or oxygen administration. The residual volume should be obtained because it provides data about possible causes of aspiration. Finally, the health care provider should be notified and informed of all the assessment data the nurse has just obtained.

36. The health care provider prescribes the following therapies for a patient who has been admitted with dehydration and hypotension after 3 days of nausea and vomiting. Which order will the nurse implement first? a. Infuse normal saline at 250 mL/hr. b. Administer IV ondansetron (Zofran). c. Provide oral care with moistened swabs. d. Insert a 16-gauge nasogastric (NG) tube.

ANS: A Because the patient has severe dehydration, rehydration with IV fluids is the priority. The other orders should be accomplished as quickly as possible after the IV fluids are initiated.

When assessing a patient who is a vegan, which finding may indicate the need for cobalamin supplementation? a. Paresthesias b. Ecchymoses c. Dry, scaly skin d. Gingival swelling

ANS: A Cobalamin (vitamin B12) cannot be obtained from foods of plant origin, so the patient will be most at risk for signs of cobalamin deficiency, such as anemia and peripheral neuropathy. The other symptoms listed are associated with other nutritional deficiencies but would not be associated with a vegan diet.

Which patient statement indicates that the nurse's teaching following a gastroduodenostomy has been effective? a. "Vitamin supplements may prevent anemia." b. "Persistent heartburn is common after surgery." c. "I will try to drink more liquids with my meals." d. "I will need to choose high carbohydrate foods."

ANS: A Cobalamin deficiency may occur after partial gastrectomy, and the patient may need to receive cobalamin via injections or nasal spray. Although peptic ulcer disease may recur, persistent heartburn is not expected after surgery and the patient should call the health care provider if this occurs. Ingestion of liquids with meals is avoided to prevent dumping syndrome. Foods that have moderate fat and low carbohydrate should be chosen to prevent dumping syndrome. DIF: Cognitive Level: Apply (application) REF: 952-953 TOP: Nursing Process: Evaluation MSC: NCLEX: Physiological Integrity

16. Cobalamin injections have been prescribed for a patient with chronic atrophic gastritis. The nurse determines that teaching regarding the injections has been effective when the patient states, a. "The cobalamin injections will prevent me from becoming anemic." b. "These injections will increase the hydrochloric acid in my stomach." c. "These injections will decrease my risk for developing stomach cancer." d. "The cobalamin injections need to be taken until my inflamed stomach heals."

ANS: A Cobalamin supplementation prevents the development of pernicious anemia. The incidence of stomach cancer is higher in patients with chronic gastritis, but cobalamin does not reduce the risk for stomach cancer. Chronic gastritis may cause achlorhydria, but cobalamin does not correct this. The loss of intrinsic factor secretion with chronic gastritis is permanent, and the patient will need lifelong supplementation with cobalamin.

13. A patient has just arrived on the postoperative unit after having a laparoscopic esophagectomy for treatment of esophageal cancer. Which nursing action should be included in the postoperative plan of care? a. Elevate the head of the bed to at least 30 degrees. b. Reposition the nasogastric (NG) tube if drainage stops or decreases. c. Notify the doctor immediately about bloody NG drainage. d. Start oral fluids when the patient has active bowel sounds.

ANS: A Elevation of the head of the bed decreases the risk for reflux and aspiration of gastric secretions. The NG tube should not be repositioned without consulting with the health care provider. Bloody NG drainage is expected for the first 8 to 12 hours. A swallowing study is needed before oral fluids are started.

The nurse will anticipate preparing a 71-year-old female patient who is vomiting "coffee-ground" emesis for a. endoscopy. b. angiography. c. barium studies. d. gastric analysis.

ANS: A Endoscopy is the primary tool for visualization and diagnosis of upper gastrointestinal (GI) bleeding. Angiography is used only when endoscopy cannot be done because it is more invasive and has more possible complications. Barium studies are helpful in determining the presence of gastric lesions, but not whether the lesions are actively bleeding. Gastric analysis testing may help with determining the cause of gastric irritation, but it is not used for acute GI bleeding. DIF: Cognitive Level: Apply (application) REF: 954 TOP: Nursing Process: Planning MSC: NCLEX: Physiological Integrity

A 50-year-old patient who underwent a gastroduodenostomy (Billroth I) earlier today complains of increasing abdominal pain. The patient has no bowel sounds and 200 mL of bright red nasogastric (NG) drainage in the last hour. The highest priority action by the nurse is to a. contact the surgeon. b. irrigate the NG tube. c. monitor the NG drainage. d. administer the prescribed morphine.

ANS: A Increased pain and 200 mL of bright red NG drainage 12 hours after surgery indicate possible postoperative hemorrhage, and immediate actions such as blood transfusion and/or return to surgery are needed. Because the NG is draining, there is no indication that irrigation is needed. Continuing to monitor the NG drainage is not an adequate response. The patient may need morphine, but this is not the highest priority action. DIF: Cognitive Level: Apply (application) REF: 946 OBJ: Special Questions: Prioritization TOP: Nursing Process: Implementation MSC: NCLEX: Physiological Integrity

25. Twelve hours after undergoing a gastroduodenostomy (Billroth I), a patient complains of increasing abdominal pain. The patient has absent bowel sounds and 200 mL of bright red nasogastric (NG) drainage in the last hour. The most appropriate action by the nurse at this time is to a. notify the surgeon. b. irrigate the NG tube. c. administer the prescribed morphine. d. continue to monitor the NG drainage.

ANS: A Increased pain and 200 mL of bright red NG drainage 12 hours after surgery indicate possible postoperative hemorrhage, and immediate actions such as blood transfusion and/or return to surgery are needed. Because the NG is draining, there is no indication that irrigation is needed. The patient may need morphine, but this is not the highest priority action. Continuing to monitor the NG drainage is not an adequate response.

A patient who has just been started on continuous tube feedings of a full-strength commercial formula at 100 mL/hr using a closed system method has six diarrhea stools the first day. Which action should the nurse plan to take? a. Slow the infusion rate of the tube feeding. b. Check gastric residual volumes more frequently. c. Change the enteral feeding system and formula every 8 hours. d. Discontinue administration of water through the feeding tube.

ANS: A Loose stools indicate poor absorption of nutrients and indicate a need to slow the feeding rate or decrease the concentration of the feeding. Water should be given when patients receive enteral feedings to prevent dehydration. When a closed enteral feeding system is used, the tubing and formula are changed every 24 hours. High residual volumes do not contribute to diarrhea.

Which action should the nurse in the emergency department anticipate for a 23-year-old patient who has had several episodes of bloody diarrhea? a. Obtain a stool specimen for culture. b. Administer antidiarrheal medication. c. Provide teaching about antibiotic therapy. d. Teach about adverse effects of acetaminophen (Tylenol).

ANS: A Patients with bloody diarrhea should have a stool culture for E. coli O157:H7. Antidiarrheal medications are usually avoided for possible infectious diarrhea to avoid prolonging the infection. Antibiotic therapy in the treatment of infectious diarrhea is controversial because it may precipitate kidney complications. Acetaminophen does not cause bloody diarrhea. DIF: Cognitive Level: Apply (application) REF: 957 TOP: Nursing Process: Planning MSC: NCLEX: Physiological Integrity

21. The health care provider orders intravenous (IV) ranitidine (Zantac) for a patient with gastrointestinal (GI) bleeding caused by peptic ulcer disease. When teaching the patient about the effect of the medication, which information will the nurse include? a. "Ranitidine decreases secretion of gastric acid." b. "Ranitidine neutralizes the acid in the stomach." c. "Ranitidine constricts the blood vessels in the stomach and decreases bleeding." d. "Ranitidine covers the ulcer with a protective material that promotes healing."

ANS: A Ranitidine is a histamine-2 (H2) receptor blocker, which decreases the secretion of gastric acid. The response beginning, "Ranitidine constricts the blood vessels" describes the effect of vasopressin. The response beginning "Ranitidine neutralizes the acid" describes the effect of antacids. And the response beginning "Ranitidine covers the ulcer" describes the action of sucralfate (Carafate).

32. When counseling a patient with a family history of stomach cancer about ways to decrease risk for developing stomach cancer, the nurse will teach the patient to avoid a. smoked foods such as bacon and ham. b. foods that cause abdominal distention. c. chronic use of H2 blocking medications. d. emotionally or physically stressful situations.

ANS: A Smoked foods such as bacon, ham, and smoked sausage increase the risk for stomach cancer. Use of H2 blockers, stressful situations, and abdominal distention are not associated with an increased incidence of stomach cancer.

30. Which information will be best for the nurse to include when teaching a patient with peptic ulcer disease (PUD) about dietary management of the disease? a. "Avoid foods that cause pain after you eat them." b. "High-protein foods are least likely to cause pain." c. "You will need to remain on a bland diet indefinitely." d. "You should avoid eating many raw fruits and vegetables."

ANS: A The best information is that each individual should choose foods that are not associated with postprandial discomfort. Raw fruits and vegetables may irritate the gastric mucosa, but chewing well seems to decrease this and some patients may tolerate these well. High-protein foods help to neutralize acid, but they also stimulate hydrochloric (HCl) acid secretion and may increase discomfort for some patients. Bland diets may be recommended during an acute exacerbation of PUD, but there is little scientific evidence to support their use.

27. A patient recovering from a gastrojejunostomy (Billroth II) for treatment of a duodenal ulcer develops dizziness, weakness, and palpitations about 20 minutes after eating. To avoid recurrence of these symptoms, the nurse teaches the patient to a. lie down for about 30 minutes after eating. b. choose foods that are high in carbohydrates. c. increase the amount of fluid intake with meals. d. drink sugared fluids or eat candy after each meal.

ANS: A The patient is experiencing symptoms of dumping syndrome, which may be reduced by lying down after eating. Increasing fluid intake and choosing high carbohydrate foods will increase the risk for dumping syndrome. Having a sweet drink or hard candy will correct the hypoglycemia that is associated with dumping syndrome but will not prevent dumping syndrome.

23. A patient with a bleeding duodenal ulcer has a nasogastric (NG) tube in place, and the health care provider orders 30 mL of aluminum hydroxide/magnesium hydroxide (Maalox) to be instilled through the tube every hour. To evaluate the effectiveness of this treatment, the nurse a. periodically aspirates and tests gastric pH. b. monitors arterial blood gas values on a daily basis. c. checks each stool for the presence of occult blood. d. measures the amount of residual stomach contents hourly.

ANS: A The purpose for antacids is to increase gastric pH. Checking gastric pH is the most direct way of evaluating the effectiveness of the medication. Arterial blood gases may change slightly, but this does not directly reflect the effect of antacids on gastric pH. Because the patient has upper gastrointestinal (GI) bleeding, occult blood in the stools will appear even after the acute bleeding has stopped. The amount of residual stomach contents is not a reflection of resolution of bleeding or of gastric pH.

A patient who is receiving continuous enteral nutrition through a small-bore silicone feeding tube has a computed tomography (CT) scan ordered and will have to be placed in a flat position for the scan. Which action by the nurse is best? a. Shut the feeding off 30 to 60 minutes before the scan. b. Ask the health care provider to reschedule the CT scan. c. Connect the feeding tube to continuous suction during the scan. d. Send the patient to CT scan with oral suction in case of aspiration.

ANS: A The tube feeding should be shut off 30 to 60 minutes before any procedure requiring the patient to lie flat. Because the CT scan is ordered for diagnosis of patient problems, rescheduling is not usually an option. Prevention, rather than treatment, of aspiration is needed. Small-bore feeding tubes are soft and collapse easily with aspiration or suction, making nasogastric suction of gastric contents unreliable.

5. To decrease the risk for cancers of the tongue and buccal mucosa, which information will the nurse include when teaching a patient who is seen for an annual physical exam in the outpatient clinic? a. Avoid use of cigarettes and smokeless tobacco. b. Use sunscreen when outside even on cloudy days. c. Complete antibiotics used to treat throat infections. d. Use antivirals to treat herpes simplex virus (HSV) infections.

ANS: A Tobacco use greatly increases the risk for oral cancer. Acute throat infections do not increase risk for oral cancer, although chronic irritation of the oral mucosa does increase risk. Sun exposure does not increase the risk for cancers of the buccal mucosa. Human papillomavirus (HPV) infection is associated with increased risk, but HSV infection is not a risk factor for oral cancer.

5. Which information will the nurse include when teaching adults to decrease the risk for cancers of the tongue and buccal mucosa? a. Avoid use of cigarettes and smokeless tobacco. b. Use sunscreen when outside even on cloudy days. c. Complete antibiotic courses used to treat throat infections. d. Use antivirals to treat herpes simplex virus (HSV) infections.

ANS: A Tobacco use greatly increases the risk for oral cancer. Acute throat infections do not increase the risk for oral cancer, although chronic irritation of the oral mucosa does increase risk. Sun exposure does not increase the risk for cancers of the buccal mucosa. Human papillomavirus (HPV) infection is associated with an increased risk, but HSV infection is not a risk factor for oral cancer. DIF: Cognitive Level: Apply (application) REF: 929 TOP: Nursing Process: Planning MSC: NCLEX: Health Promotion and Maintenance

MULTIPLE RESPONSE 1. During a busy day, the nurse admits all of the following patients to the medical-surgical unit. Which patients are most important to refer to the dietitian for a complete nutritional assessment (select all that apply)? a. A 24-year-old who has a history of weight gains and losses b. A 53-year-old who complains of intermittent nausea for the past 2 days c. A 66-year-old who is admitted for débridement of an infected surgical wound d. A 45-year-old admitted with chest pain and possible myocardial infarction (MI) e. A 32-year-old with rheumatoid arthritis who takes prednisone (Deltasone) daily

ANS: A, C, E Weight fluctuations, use of corticosteroids, and draining or infected wounds all suggest that the patient may be at risk for malnutrition. Patients with chest pain or MI are not usually poorly nourished. Although vomiting that lasts 5 days places a patient at risk, nausea that has persisted for 2 days does not always indicate poor nutritional status or risk for health problems caused by poor nutrition.

34. Which information about a patient who has just been admitted to the hospital with nausea and vomiting will require the most rapid intervention by the nurse? a. The patient has taken only sips of water. b. The patient is lethargic and difficult to arouse. c. The patient's chart indicates a recent resection of the small intestine. d. The patient has been vomiting several times a day for the last 4 days.

ANS: B A lethargic patient is at risk for aspiration, and the nurse will need to position the patient to decrease aspiration risk. The other information also is important to collect, but it does not require as quick action as the risk for aspiration.

Parenteral nutrition (PN) containing amino acids and dextrose was ordered and hung 24 hours ago for a malnourished patient. The nurse observes that about 50 mL remain in the PN container. Which action is best for the nurse to take? a. Ask the health care provider to clarify the written PN order. b. Add a new container of PN using the current tubing and filter. c. Hang a new container of PN and change the IV tubing and filter. d. Infuse the remaining 50 mL and then hang a new container of PN.

ANS: B All PN solutions are changed at 24 hours. PN solutions containing dextrose and amino acids require a change in tubing and filter every 72 hours rather than daily. Infusion of the additional 50 mL will increase patient risk for infection. Changing the IV tubing and filter more frequently than required will unnecessarily increase costs. The nurse (not the health care provider) is responsible for knowing the indicated times for tubing and filter changes.

A 49-year-old man has been admitted with hypotension and dehydration after 3 days of nausea and vomiting. Which order from the health care provider will the nurse implement first? a. Insert a nasogastric (NG) tube. b. Infuse normal saline at 250 mL/hr. c. Administer IV ondansetron (Zofran). d. Provide oral care with moistened swabs.

ANS: B Because the patient has severe dehydration, rehydration with IV fluids is the priority. The other orders should be accomplished as quickly as possible after the IV fluids are initiated. DIF: Cognitive Level: Apply (application) REF: 927 OBJ: Special Questions: Prioritization TOP: Nursing Process: Implementation MSC: NCLEX: Physiological Integrity

2. Which item should the nurse offer to the patient who is to restart oral intake after being NPO due to nausea and vomiting? a. Glass of orange juice b. Dish of lemon gelatin c. Cup of coffee with cream d. Bowl of hot chicken broth

ANS: B Clear cool liquids are usually the first foods started after a patient has been nauseated. Acidic foods such as orange juice, very hot foods, and coffee are poorly tolerated when patients have been nauseated. DIF: Cognitive Level: Apply (application) REF: 928 TOP: Nursing Process: Implementation MSC: NCLEX: Physiological Integrity

The nurse determines that teaching regarding cobalamin injections has been effective when the patient with chronic atrophic gastritis states which of the following? a. "The cobalamin injections will prevent gastric inflammation." b. "The cobalamin injections will prevent me from becoming anemic." c. "These injections will increase the hydrochloric acid in my stomach." d. "These injections will decrease my risk for developing stomach cancer."

ANS: B Cobalamin supplementation prevents the development of pernicious anemia. Chronic gastritis may cause achlorhydria, but cobalamin does not correct this. The loss of intrinsic factor secretion with chronic gastritis is permanent, and the patient will need lifelong supplementation with cobalamin. The incidence of stomach cancer is higher in patients with chronic gastritis, but cobalamin does not reduce the risk for stomach cancer. DIF: Cognitive Level: Apply (application) REF: 941-942 TOP: Nursing Process: Evaluation MSC: NCLEX: Physiological Integrity

38. Which of these assessment findings in a patient with a hiatal hernia who returned from a laparoscopic Nissen fundoplication 4 hours ago is most important for the nurse to address immediately? a. The patient is experiencing intermittent waves of nausea. b. The patient has absent breath sounds throughout the left lung. c. The patient has decreased bowel sounds in all four quadrants. d. The patient complains of 6/10 (0 to 10 scale) abdominal pain.

ANS: B Decreased breath sounds on one side may indicate a pneumothorax, which requires rapid diagnosis and treatment. The abdominal pain and nausea also should be addressed but they are not as high priority as the patient's respiratory status. The patient's decreased bowel sounds are expected after surgery and require ongoing monitoring but no other action.

Which action should the nurse take first in order to improve calorie and protein intake for a patient who eats only about 50% of each meal because of "feeling too tired to eat much." a. Teach the patient about the importance of good nutrition. b. Serve multiple small feedings of high-calorie, high-protein foods. c. Obtain an order for enteral feedings of liquid nutritional supplements. d. Consult with the health care provider about providing parenteral nutrition (PN).

ANS: B Eating small amounts of food frequently throughout the day is less fatiguing and will improve the patient's ability to take in more nutrients. Teaching the patient may be appropriate, but will not address the patient's inability to eat more because of fatigue. Tube feedings or PN may be needed if the patient is unable to take in enough nutrients orally, but increasing the oral intake should be attempted first.

Which information will the nurse include for a patient with newly diagnosed gastroesophageal reflux disease (GERD)? a. "Peppermint tea may reduce your symptoms." b. "Keep the head of your bed elevated on blocks." c. "You should avoid eating between meals to reduce acid secretion." d. "Vigorous physical activities may increase the incidence of reflux

ANS: B Elevating the head of the bed will reduce the incidence of reflux while the patient is sleeping. Peppermint will decrease lower esophageal sphincter (LES) pressure and increase the chance for reflux. Small, frequent meals are recommended to avoid abdominal distention. There is no need to make changes in physical activities because of GERD. DIF: Cognitive Level: Apply (application) REF: 935 TOP: Nursing Process: Implementation MSC: NCLEX: Physiological Integrity

Which nursing action should be included in the postoperative plan of care for a patient after a laparoscopic esophagectomy? a. Notify the doctor about bloody nasogastric (NG) drainage. b. Elevate the head of the bed to at least 30 degrees. c. Reposition the NG tube if drainage stops. d. Start oral fluids when the patient has active bowel sounds.

ANS: B Elevation of the head of the bed decreases the risk for reflux and aspiration of gastric secretions. The NG tube should not be repositioned without consulting with the health care provider. Bloody NG drainage is expected for the first 8 to 12 hours. A swallowing study is needed before oral fluids are started. DIF: Cognitive Level: Apply (application) REF: 939 TOP: Nursing Process: Planning MSC: NCLEX: Physiological Integrity

22. The family member of a patient who has suffered massive abdominal trauma in an automobile accident asks the nurse why the patient is receiving famotidine (Pepcid). The nurse will explain that the medication will a. prevent aspiration of gastric contents. b. inhibit the development of stress ulcers. c. lower the chance for H. pylori infection. d. decrease the risk for nausea and vomiting.

ANS: B Famotidine is administered to prevent the development of physiologic stress ulcers, which are associated with a major physiologic insult such as massive trauma. Famotidine does not decrease nausea or vomiting, prevent aspiration, or prevent H. pylori infection.

A family member of a 28-year-old patient who has suffered massive abdominal trauma in an automobile accident asks the nurse why the patient is receiving famotidine (Pepcid). The nurse will explain that the medication will a. decrease nausea and vomiting. b. inhibit development of stress ulcers. c. lower the risk for H. pylori infection. d. prevent aspiration of gastric contents.

ANS: B Famotidine is administered to prevent the development of physiologic stress ulcers, which are associated with a major physiologic insult such as massive trauma. Famotidine does not decrease nausea or vomiting, prevent aspiration, or prevent H. pylori infection. DIF: Cognitive Level: Apply (application) REF: 934 TOP: Nursing Process: Implementation MSC: NCLEX: Physiological Integrity

A 26-year-old patient with a family history of stomach cancer asks the nurse about ways to decrease the risk for developing stomach cancer. The nurse will teach the patient to avoid a. emotionally stressful situations. b. smoked foods such as ham and bacon. c. foods that cause distention or bloating. d. chronic use of H2 blocking medications.

ANS: B Smoked foods such as bacon, ham, and smoked sausage increase the risk for stomach cancer. Stressful situations, abdominal distention, and use of H2 blockers are not associated with an increased incidence of stomach cancer. DIF: Cognitive Level: Understand (comprehension) REF: 951 TOP: Nursing Process: Implementation MSC: NCLEX: Physiological Integrity

Which information about dietary management should the nurse include when teaching a patient with peptic ulcer disease (PUD)? a. "You will need to remain on a bland diet." b. "Avoid foods that cause pain after you eat them." c. "High-protein foods are least likely to cause you pain." d. "You should avoid eating any raw fruits and vegetables."

ANS: B The best information is that each individual should choose foods that are not associated with postprandial discomfort. Raw fruits and vegetables may irritate the gastric mucosa, but chewing well seems to decrease this problem and some patients may tolerate these foods well. High-protein foods help neutralize acid, but they also stimulate hydrochloric (HCl) acid secretion and may increase discomfort for some patients. Bland diets may be recommended during an acute exacerbation of PUD, but there is little scientific evidence to support their use. DIF: Cognitive Level: Apply (application) REF: 947 TOP: Nursing Process: Implementation MSC: NCLEX: Physiological Integrity

17. A patient with peptic ulcer disease associated with the presence of Helicobacter pylori is treated with triple drug therapy. The nurse will plan to teach the patient about a. sucralfate (Carafate), nystatin (Mycostatin), and bismuth (Pepto-Bismol). b. amoxicillin (Amoxil), clarithromycin (Biaxin), and omeprazole (Prilosec). c. famotidine (Pepcid), magnesium hydroxide (Mylanta), and pantoprazole (Protonix). d. metoclopramide (Reglan), bethanechol (Urecholine), and promethazine (Phenergan).

ANS: B The drugs used in triple drug therapy include a proton pump inhibitor such as omeprazole and the antibiotics amoxicillin and clarithromycin. The other combinations listed are not included in the protocol for H. pylori infection.

The nurse and a licensed practical/vocational nurse (LPN/LVN) are working together to care for a patient who had an esophagectomy 2 days ago. Which action by the LPN/LVN requires that the nurse intervene? a. The LPN/LVN uses soft swabs to provide for oral care. b. The LPN/LVN positions the head of the bed in the flat position. c. The LPN/LVN encourages the patient to use pain medications before coughing. d. The LPN/LVN includes the enteral feeding volume when calculating intake and output.

ANS: B The patient's bed should be in Fowler's position to prevent reflux and aspiration of gastric contents. The other actions by the LPN/LVN are appropriate. DIF: Cognitive Level: Apply (application) REF: 939 OBJ: Special Questions: Delegation TOP: Nursing Process: Evaluation MSC: NCLEX: Safe and Effective Care Environment

A 44-year-old man admitted with a peptic ulcer has a nasogastric (NG) tube in place. When the patient develops sudden, severe upper abdominal pain, diaphoresis, and a firm abdomen, which action should the nurse take? a. Irrigate the NG tube. b. Check the vital signs. c. Give the ordered antacid. d. Elevate the foot of the bed.

ANS: B The patient's symptoms suggest acute perforation, and the nurse should assess for signs of hypovolemic shock. Irrigation of the NG tube, administration of antacids, or both would be contraindicated because any material in the stomach will increase the spillage into the peritoneal cavity. Elevating the foot of the bed may increase abdominal pressure and discomfort, as well as making it more difficult for the patient to breathe. DIF: Cognitive Level: Apply (application) REF: 948 TOP: Nursing Process: Implementation MSC: NCLEX: Physiological Integrity

A 68-year-old patient with a bleeding duodenal ulcer has a nasogastric (NG) tube in place, and the health care provider orders 30 mL of aluminum hydroxide/magnesium hydroxide (Maalox) to be instilled through the tube every hour. To evaluate the effectiveness of this treatment, the nurse a. monitors arterial blood gas values daily. b. periodically aspirates and tests gastric pH. c. checks each stool for the presence of occult blood. d. measures the volume of residual stomach contents.

ANS: B The purpose for antacids is to increase gastric pH. Checking gastric pH is the most direct way of evaluating the effectiveness of the medication. Arterial blood gases may change slightly, but this does not directly reflect the effect of antacids on gastric pH. Because the patient has upper gastrointestinal (GI) bleeding, occult blood in the stools will appear even after the acute bleeding has stopped. The amount of residual stomach contents is not a reflection of resolution of bleeding or of gastric pH. DIF: Cognitive Level: Apply (application) REF: 946 TOP: Nursing Process: Evaluation MSC: NCLEX: Physiological Integrity

How many grams of protein will the nurse recommend to meet the minimum daily requirement for a patient who weighs 145 pounds (66 kg)? a. 36 b. 53 c. 75 d. 98

ANS: B The recommended daily protein intake is 0.8 to 1 g/kg of body weight, which for this patient is 66 kg × 0.8 g = 52.8 or 53 g/day.

A 68-year-old male patient with a stroke is unconscious and unresponsive to stimuli. After learning that the patient has a history of gastroesophageal reflux disease (GERD), the nurse will plan to do frequent assessments of the patient's a. apical pulse. b. bowel sounds. c. breath sounds. d. abdominal girth.

ANS: C Because GERD may cause aspiration, the unconscious patient is at risk for developing aspiration pneumonia. Bowel sounds, abdominal girth, and apical pulse will not be affected by the patient's stroke or GERD and do not require more frequent monitoring than the routine. DIF: Cognitive Level: Apply (application) REF: 932 TOP: Nursing Process: Assessment MSC: NCLEX: Physiological Integrity

Which order from the health care provider will the nurse implement first for a patient who has vomited 1200 mL of blood? a. Give an IV H2 receptor antagonist. b. Draw blood for typing and crossmatching. c. Administer 1000 mL of lactated Ringer's solution. d. Insert a nasogastric (NG) tube and connect to suction.

ANS: C Because the patient has vomited a large amount of blood, correction of hypovolemia and prevention of hypovolemic shock are the priorities. The other actions also are important to implement quickly but are not the highest priorities. DIF: Cognitive Level: Apply (application) REF: 947 | 955 OBJ: Special Questions: Prioritization TOP: Nursing Process: Implementation MSC: NCLEX: Physiological Integrity

A 53-year-old male patient with deep partial-thickness burns from a chemical spill in the workplace experiences severe pain followed by nausea during dressing changes. Which action will be most useful in decreasing the patient's nausea? a. Keep the patient NPO for 2 hours before and after dressing changes. b. Avoid performing dressing changes close to the patient's mealtimes. c. Administer the prescribed morphine sulfate before dressing changes. d. Give the ordered prochlorperazine (Compazine) before dressing changes.

ANS: C Because the patient's nausea is associated with severe pain, it is likely that it is precipitated by stress and pain. The best treatment will be to provide adequate pain medication before dressing changes. The nurse should avoid doing painful procedures close to mealtimes, but nausea/vomiting that occur at other times also should be addressed. Keeping the patient NPO does not address the reason for the nausea and vomiting and will have an adverse effect on the patient's nutrition. Administration of antiemetics is not the best choice for a patient with nausea caused by pain. DIF: Cognitive Level: Apply (application) REF: 927 TOP: Nursing Process: Implementation MSC: NCLEX: Physiological Integrity

A 38-year old woman receiving chemotherapy for breast cancer develops a Candida albicans oral infection. The nurse will anticipate the need for a. hydrogen peroxide rinses. b. the use of antiviral agents. c. administration of nystatin (Mycostatin) tablets. d. referral to a dentist for professional tooth cleaning.

ANS: C Candida albicans is treated with an antifungal such as nystatin. Oral saltwater rinses may be used but will not cure the infection. Antiviral agents are used for viral infections such as herpes simplex. Referral to a dentist is indicated for gingivitis but not for Candida infection. DIF: Cognitive Level: Apply (application) REF: 928 TOP: Nursing Process: Planning MSC: NCLEX: Physiological Integrity

26. The nurse implements discharge teaching for a patient following a gastroduodenostomy for treatment of a peptic ulcer. Which patient statement indicates that the teaching has been effective? a. "Persistent heartburn is expected after surgery." b. "I will try to drink liquids along with my meals." c. "Vitamin supplements may be needed to prevent problems with anemia." d. "I will need to choose foods that are low in fat and high in carbohydrate."

ANS: C Cobalamin deficiency may occur after partial gastrectomy, and the patient may need to receive cobalamin via injections or nasal spray. Foods that have moderate fat and low carbohydrate should be chosen to prevent dumping syndrome. Ingestion of liquids with meals is avoided to prevent dumping syndrome. Although peptic ulcer disease may recur, persistent heartburn is not expected after surgery and the patient should call the health care provider if this occurs.

After the nurse has completed teaching a patient with newly diagnosed eosinophilic esophagitis about the management of the disease, which patient action indicates that the teaching has been effective? a. Patient orders nonfat milk for each meal. b. Patient uses the prescribed corticosteroid inhaler. c. Patient schedules an appointment for allergy testing. d. Patient takes ibuprofen (Advil) to control throat pain.

ANS: C Eosinophilic esophagitis is frequently associated with environmental allergens, so allergy testing is used to determine possible triggers. Corticosteroid therapy may be prescribed, but the medication will be swallowed, not inhaled. Milk is a frequent trigger for attacks. NSAIDs are not used for eosinophilic esophagitis. DIF: Cognitive Level: Apply (application) REF: 939 TOP: Nursing Process: Evaluation MSC: NCLEX: Physiological Integrity

All of the following nursing actions are included in the plan of care for a patient who is malnourished. Which action is appropriate for the nurse to delegate to nursing assistive personnel (NAP)? a. Assist the patient to choose high nutrition items from the menu. b. Monitor the patient for skin breakdown over the bony prominences. c. Offer the patient the prescribed nutritional supplement between meals. d. Assess the patient's strength while ambulating the patient in the room.

ANS: C Feeding the patient and assisting with oral intake are included in NAP education and scope of practice. Assessing the patient and assisting the patient in choosing high nutrition foods require LPN/LVN- or RN-level education and scope of practice.

A 46-year-old female with gastroesophageal reflux disease (GERD) is experiencing increasing discomfort. Which patient statement indicates that additional teaching about GERD is needed? a. "I take antacids between meals and at bedtime each night." b. "I sleep with the head of the bed elevated on 4-inch blocks." c. "I eat small meals during the day and have a bedtime snack." d. "I quit smoking several years ago, but I still chew a lot of gum."

ANS: C GERD is exacerbated by eating late at night, and the nurse should plan to teach the patient to avoid eating at bedtime. The other patient actions are appropriate to control symptoms of GERD. DIF: Cognitive Level: Apply (application) REF: 933 TOP: Nursing Process: Evaluation MSC: NCLEX: Physiological Integrity

Which patient choice for a snack 2 hours before bedtime indicates that the nurse's teaching about gastroesophageal reflux disease (GERD) has been effective? a. Chocolate pudding b. Glass of low-fat milk c. Cherry gelatin with fruit d. Peanut butter and jelly sandwich

ANS: C Gelatin and fruit are low fat and will not decrease lower esophageal sphincter (LES) pressure. Foods such as chocolate are avoided because they lower LES pressure. Milk products increase gastric acid secretion. High-fat foods such as peanut butter decrease both gastric emptying and LES pressure. DIF: Cognitive Level: Apply (application) REF: 933 TOP: Nursing Process: Evaluation MSC: NCLEX: Physiological Integrity

An 80-year-old who is hospitalized with peptic ulcer disease develops new-onset auditory hallucinations. Which prescribed medication will the nurse discuss with the health care provider before administration? a. Sucralfate (Carafate) b. Omeprazole (Prilosec) c. Metoclopramide (Reglan) d. Aluminum hydroxide (Amphojel)

ANS: C Metoclopramide can cause central nervous system (CNS) side effects ranging from anxiety to hallucinations. Hallucinations are not a side effect of proton-pump inhibitors, mucosal protectants, or antacids. DIF: Cognitive Level: Apply (application) REF: 934 TOP: Nursing Process: Implementation MSC: NCLEX: Physiological Integrity

After 6 hours of parenteral nutrition (PN) infusion, the nurse checks a patient's capillary blood glucose level and finds it to be 120 mg/dL. The most appropriate action by the nurse is to a. obtain a venous blood glucose specimen. b. slow the infusion rate of the PN infusion. c. recheck the capillary blood glucose in 4 hours. d. notify the health care provider of the glucose level.

ANS: C Mild hyperglycemia is expected during the first few days after PN is started and requires ongoing monitoring. Because the glucose elevation is small and expected, notification of the health care provider is not necessary. There is no need to obtain a venous specimen for comparison. Slowing the rate of the infusion is beyond the nurse's scope of practice and will decrease the patient's nutritional intake.

A patient is receiving tube feedings through a percutaneous endoscopic gastrostomy (PEG). Which action will the nurse include in the plan of care? a. Keep the patient positioned on the left side. b. Obtain a daily x-ray to verify tube placement. c. Check the gastric residual volume every 4 to 6 hours. d. Avoid giving bolus tube feedings through the PEG tube.

ANS: C The gastric residual volume is assessed every 4 to 6 hours to decrease the risk for aspiration. The patient does not need to be positioned on the left side. An x-ray is obtained immediately after placement of the PEG tube to check position, but daily x-rays are not needed. Bolus feedings can be administered through a PEG tube.

At his first postoperative checkup appointment after a gastrojejunostomy (Billroth II), a patient reports that dizziness, weakness, and palpitations occur about 20 minutes after each meal. The nurse will teach the patient to a. increase the amount of fluid with meals. b. eat foods that are higher in carbohydrates. c. lie down for about 30 minutes after eating. d. drink sugared fluids or eat candy after meals.

ANS: C The patient is experiencing symptoms of dumping syndrome, which may be reduced by lying down after eating. Increasing fluid intake and choosing high carbohydrate foods will increase the risk for dumping syndrome. Having a sweet drink or hard candy will correct the hypoglycemia that is associated with dumping syndrome but will not prevent dumping syndrome. DIF: Cognitive Level: Apply (application) REF: 949-950 TOP: Nursing Process: Implementation MSC: NCLEX: Physiological Integrity

37. After receiving change-of-shift report, which patient should the nurse assess first? a. A patient who was admitted yesterday with gastrointestinal (GI) bleeding and has melena b. A patient who is crying after receiving a diagnosis of esophageal cancer c. A patient with esophageal varices who has a blood pressure of 96/54 mm Hg d. A patient with nausea who has a dose of metoclopramide (Reglan) scheduled

ANS: C The patient's history and blood pressure indicate possible hemodynamic instability caused by GI bleeding. The data about the other patients do not indicate acutely life-threatening complications.

41. A patient with acute gastrointestinal (GI) bleeding is receiving normal saline IV at a rate of 500 mL/hr. Which assessment finding obtained by the nurse is most important to communicate immediately to the health care provider? a. The patient's blood pressure (BP) has increased to 142/94 mm Hg. b. The nasogastric (NG) suction is returning coffee-ground material. c. The patient's lungs have crackles audible to the midline. d. The bowel sounds are very hyperactive in all four quadrants.

ANS: C The patient's lung sounds indicate that pulmonary edema may be developing as a result of the rapid infusion of IV fluid and that the fluid infusion rate should be slowed. The return of coffee-ground material in an NG tube is expected for a patient with upper GI bleeding. The BP is slightly elevated but would not be an indication to contact the health care provider immediately. Hyperactive bowel sounds are common when a patient has GI bleeding.

11. A 62-year-old patient who has been diagnosed with esophageal cancer tells the nurse, "I know that my chances are not very good, but I do not feel ready to die yet." Which response by the nurse is most appropriate? a. "You may have quite a few years still left to live." b. "Thinking about dying will only make you feel worse." c. "Having this new diagnosis must be very hard for you." d. "It is important that you be realistic about your prognosis."

ANS: C This response is open-ended and will encourage the patient to further discuss feelings of anxiety or sadness about the diagnosis. Patients with esophageal cancer have only a low survival rate, so the response "You may have quite a few years still left to live" is misleading. The response beginning, "Thinking about dying" indicates that the nurse is not open to discussing the patient's fears of dying. And the response beginning, "It is important that you be realistic," discourages the patient from feeling hopeful, which is important to patients with any life-threatening diagnosis.

A 58-year-old woman who recently has been diagnosed with esophageal cancer tells the nurse, "I do not feel ready to die yet." Which response by the nurse is most appropriate? a. "You may have quite a few years still left to live." b. "Thinking about dying will only make you feel worse." c. "Having this new diagnosis must be very hard for you." d. "It is important that you be realistic about your prognosis."

ANS: C This response is open-ended and will encourage the patient to further discuss feelings of anxiety or sadness about the diagnosis. Patients with esophageal cancer have only a low survival rate, so the response "You may have quite a few years still left to live" is misleading. The response beginning, "Thinking about dying" indicates that the nurse is not open to discussing the patient's fears of dying. The response beginning, "It is important that you be realistic," discourages the patient from feeling hopeful, which is important to patients with any life-threatening diagnosis. DIF: Cognitive Level: Apply (application) REF: 939 TOP: Nursing Process: Implementation MSC: NCLEX: Psychosocial Integrity

The nurse notes that the peripheral parenteral nutrition (PN) bag has only 20 mL left and a new PN bag has not yet arrived from the pharmacy. Which intervention is the priority? a. Monitor the patient's capillary blood glucose until a new PN bag is hung b. Flush the peripheral line with saline and wait until the new PN bag is available c. Infuse 5% dextrose in water until the new PN bag is delivered from the pharmacy d. Decrease the rate of the current PN infusion to 10 mL/hr until the new bag arrives

ANS: C To prevent hypoglycemia, the nurse should infuse a 5% dextrose solution until the next PN bag can be started. Decreasing the rate of the ordered PN infusion is beyond the nurse's scope of practice. Flushing the line and then waiting for the next bag may lead to hypoglycemia. Monitoring the capillary blood glucose is appropriate but is not the priority.

A 57-year-old man with Escherichia coli O157:H7 food poisoning is admitted to the hospital with bloody diarrhea and dehydration. Which order will the nurse question? a. Infuse lactated Ringer's solution at 250 mL/hr. b. Monitor blood urea nitrogen and creatinine daily. c. Administer loperamide (Imodium) after each stool. d. Provide a clear liquid diet and progress diet as tolerated.

ANS: C Use of antidiarrheal agents is avoided with this type of food poisoning. The other orders are appropriate. DIF: Cognitive Level: Apply (application) REF: 958 TOP: Nursing Process: Implementation MSC: NCLEX: Physiological Integrity

33. Which assessment finding in a patient who had a total gastrectomy 12 hours previously is most important to report to the health care provider? a. Absent bowel sounds b. Scant nasogastric (NG) tube drainage c. Complaints of incisional pain d. Temperature 102.1° F (38.9° C)

ANS: D An elevation in temperature may indicate leakage at the anastomosis, which may require return to surgery or keeping the patient NPO. The other findings are expected in the immediate postoperative period for patients who have this surgery.

10. A patient who recently has been experiencing frequent heartburn is seen in the clinic. The nurse will anticipate teaching the patient about a. barium swallow. b. radionuclide tests. c. endoscopy procedures. d. proton pump inhibitors.

ANS: D Because diagnostic testing for heartburn that is probably caused by gastroesophageal reflux disease (GERD) is expensive and uncomfortable, proton pump inhibitors are frequently used for a short period as the first step in the diagnosis of GERD. The other tests may be used but are not usually the first step in diagnosis.

A patient with protein calorie malnutrition who has had abdominal surgery is receiving parenteral nutrition (PN). Which assessment information obtained by the nurse is the best indicator that the patient is receiving adequate nutrition? a. Blood glucose is 110 mg/dL. b. Serum albumin level is 3.5 mg/dL. c. Fluid intake and output are balanced. d. Surgical incision is healing normally.

ANS: D Because poor wound healing is a possible complication of malnutrition for this patient, normal healing of the incision is an indicator of the effectiveness of the PN in providing adequate nutrition. Blood glucose is monitored to prevent the complications of hyperglycemia and hypoglycemia, but it does not indicate that the patient's nutrition is adequate. The intake and output will be monitored but do not indicate that the PN is effective. The albumin level is in the low-normal range but does not reflect adequate caloric intake, which is also important for the patient.

3. A patient who is receiving chemotherapy develops a Candida albicans oral infection. The nurse will anticipate the need for a. hydrogen peroxide rinses. b. the use of antiviral agents. c. referral to a dentist for professional tooth cleaning. d. administration of nystatin (Mycostatin) oral tablets.

ANS: D Candida albicans is treated with an antifungal such as nystatin. Oral saltwater rinses may be used but will not cure the infection. Antiviral agents are used for viral infections such as herpes simplex. Referral to a dentist is indicated for gingivitis but not for Candida infection.

12. Which information will the nurse include when teaching a patient with newly diagnosed gastroesophageal reflux disease (GERD)? a. "Peppermint tea may be helpful in reducing your symptoms." b. "You should avoid eating between meals to reduce acid secretion." c. "Vigorous physical activities may increase the incidence of reflux." d. "It will be helpful to keep the head of your bed elevated on blocks."

ANS: D Elevating the head of the bed will reduce the incidence of reflux while the patient is sleeping. Peppermint will lower LES pressure and increase the chance for reflux. Small, frequent meals are recommended to avoid abdominal distention. There is no need to make changes in physical activities because of GERD.

When a 72-year-old patient is diagnosed with achalasia, the nurse will teach the patient that a. lying down after meals is recommended. b. a liquid or blenderized diet will be necessary. c. drinking fluids with meals should be avoided. d. treatment may include endoscopic procedures.

ANS: D Endoscopic and laparoscopic procedures are the most effective therapy for improving symptoms caused by achalasia. Keeping the head elevated after eating will improve esophageal emptying. A semisoft diet is recommended to improve esophageal emptying. Patients are advised to drink fluid with meals. DIF: Cognitive Level: Apply (application) REF: 940 TOP: Nursing Process: Planning MSC: NCLEX: Physiological Integrity

28. A patient who requires daily use of a nonsteroidal anti-inflammatory drug (NSAID) for management of severe rheumatoid arthritis has recently developed melena. The nurse will anticipate teaching the patient about a. substitution of acetaminophen (Tylenol) for the NSAID. b. use of enteric-coated NSAIDs to reduce gastric irritation. c. reasons for using corticosteroids to treat the rheumatoid arthritis. d. the benefits of misoprostol (Cytotec) in protecting the gastrointestinal (GI) mucosa.

ANS: D Misoprostol, a prostaglandin analog, reduces acid secretion and incidence of upper GI bleeding associated with NSAID use. Enteric coating of NSAIDs does not reduce the risk for GI bleeding. Corticosteroids increase the risk for ulcer development and will not be substituted for NSAIDs for this patient. Acetaminophen will not be effective in treating the patient's rheumatoid arthritis.

A 62-year-old man patient who requires daily use of a nonsteroidal antiinflammatory drug (NSAID) for the management of severe rheumatoid arthritis has recently developed melena. The nurse will anticipate teaching the patient about a. substitution of acetaminophen (Tylenol) for the NSAID. b. use of enteric-coated NSAIDs to reduce gastric irritation. c. reasons for using corticosteroids to treat the rheumatoid arthritis. d. misoprostol (Cytotec) to protect the gastrointestinal (GI) mucosa

ANS: D Misoprostol, a prostaglandin analog, reduces acid secretion and the incidence of upper GI bleeding associated with NSAID use. Enteric coating of NSAIDs does not reduce the risk for GI bleeding. Corticosteroids increase the risk for ulcer development, and will not be substituted for NSAIDs for this patient. Acetaminophen will not be effective in treating the patient's rheumatoid arthritis. DIF: Cognitive Level: Apply (application) REF: 954 | 956 TOP: Nursing Process: Planning MSC: NCLEX: Physiological Integrity

The health care provider prescribes antacids and sucralfate (Carafate) for treatment of a patient's peptic ulcer. The nurse will teach the patient to take a. sucralfate at bedtime and antacids before each meal. b. sucralfate and antacids together 30 minutes before meals. c. antacids 30 minutes before each dose of sucralfate is taken. d. antacids after meals and sucralfate 30 minutes before meals.

ANS: D Sucralfate is most effective when the pH is low and should not be given with or soon after antacids. Antacids are most effective when taken after eating. Administration of sucralfate 30 minutes before eating and antacids just after eating will ensure that both drugs can be most effective. The other regimens will decrease the effectiveness of the medications. DIF: Cognitive Level: Understand (comprehension) REF: 934 | 947 TOP: Nursing Process: Implementation MSC: NCLEX: Physiological Integrity

Which assessment should the nurse perform first for a patient who just vomited bright red blood? a. Measuring the quantity of emesis b. Palpating the abdomen for distention c. Auscultating the chest for breath sounds d. Taking the blood pressure (BP) and pulse

ANS: D The nurse is concerned about blood loss and possible hypovolemic shock in a patient with acute gastrointestinal (GI) bleeding. BP and pulse are the best indicators of these complications. The other information is important to obtain, but BP and pulse rate are the best indicators for assessing intravascular volume. DIF: Cognitive Level: Apply (application) REF: 925 | 956 OBJ: Special Questions: Prioritization TOP: Nursing Process: Assessment MSC: NCLEX: Physiological Integrity

39. A patient who is vomiting bright red blood is admitted to the emergency department. Which assessment should the nurse perform first? a. Checking the level of consciousness b. Measuring the quantity of any emesis c. Auscultating the chest for breath sounds d. Taking the blood pressure (BP) and pulse

ANS: D The nurse is concerned about blood loss and possible hypovolemic shock in a patient with acute gastrointestinal (GI) bleeding; BP and pulse are the best indicators of these complications. The other information also is important to obtain, but BP and pulse rate are the best indicators for hypoperfusion.

The nurse receives change-of-shift report about the following four patients. Which patient will the nurse assess first? a. A patient who has malnutrition associated with 4+ generalized pitting edema b. A patient whose parenteral nutrition has 10 mL of solution left in the infusion bag c. A patient whose gastrostomy tube is plugged after crushed medications were given through the tube d. A patient who is receiving continuous enteral feedings and has new-onset crackles throughout the lungs

ANS: D The patient data suggest aspiration has occurred and rapid assessment and intervention are needed. The other patients also should be assessed as quickly as possible, but the data about them do not suggest any immediately life-threatening complications.

8. A patient with recurring heartburn receives a new prescription for esomeprazole (Nexium). In teaching the patient about this medication, the nurse explains that this drug a. neutralizes stomach acid and provides relief of symptoms in a few minutes. b. reduces the reflux of gastric acid by increasing the rate of gastric emptying. c. coats and protects the lining of the stomach and esophagus from gastric acid. d. treats gastroesophageal reflux disease by decreasing stomach acid production.

ANS: D The proton pump inhibitors decrease the rate of gastric acid secretion. Promotility drugs such as metoclopramide (Reglan) increase the rate of gastric emptying. Cryoprotective medications such as sucralfate (Carafate) protect the stomach. Antacids neutralize stomach acid and work rapidly.

The nurse explaining esomeprazole (Nexium) to a patient with recurring heartburn describes that the medication a. reduces gastroesophageal reflux by increasing the rate of gastric emptying. b. neutralizes stomach acid and provides relief of symptoms in a few minutes. c. coats and protects the lining of the stomach and esophagus from gastric acid. d. treats gastroesophageal reflux disease by decreasing stomach acid production.

ANS: D The proton pump inhibitors decrease the rate of gastric acid secretion. Promotility drugs such as metoclopramide (Reglan) increase the rate of gastric emptying. Cryoprotective medications such as sucralfate (Carafate) protect the stomach. Antacids neutralize stomach acid and work rapidly. DIF: Cognitive Level: Understand (comprehension) REF: 934 TOP: Nursing Process: Implementation MSC: NCLEX: Physiological Integrity

After change-of-shift report, which patient should the nurse assess first? a. 42-year-old who has acute gastritis and ongoing epigastric pain b. 70-year-old with a hiatal hernia who experiences frequent heartburn c. 53-year-old who has dumping syndrome after a recent partial gastrectomy d. 60-year-old with nausea and vomiting who has dry oral mucosa and lethargy

ANS: D This older patient is at high risk for problems such as aspiration, dehydration, and fluid and electrolyte disturbances. The other patients will also need to be assessed, but the information about them indicates symptoms that are typical for their diagnoses and are not life threatening. DIF: Cognitive Level: Apply (application) REF: 928 OBJ: Special Questions: Prioritization

15. A patient who is nauseated and vomiting up blood-streaked fluid is admitted to the hospital with acute gastritis. To determine possible risk factors for gastritis, the nurse will ask the patient about a. the amount of fat in the diet. b. history of recent weight gain or loss. c. any family history of gastric problems. d. use of nonsteroidal anti-inflammatory drugs (NSAIDs).

ANS: D Use of an NSAID is associated with damage to the gastric mucosa, which can result in acute gastritis. Family history, recent weight gain or loss, and fatty foods are not risk factors for acute gastritis.

A 50-year-old man vomiting blood-streaked fluid is admitted to the hospital with acute gastritis. To determine possible risk factors for gastritis, the nurse will ask the patient about a. the amount of saturated fat in the diet. b. any family history of gastric or colon cancer. c. a history of a large recent weight gain or loss. d. use of nonsteroidal antiinflammatory drugs (NSAIDs).

ANS: D Use of an NSAID is associated with damage to the gastric mucosa, which can result in acute gastritis. Family history, recent weight gain or loss, and fatty foods are not risk factors for acute gastritis. DIF: Cognitive Level: Understand (comprehension) REF: 941 TOP: Nursing Process: Assessment MSC: NCLEX: Physiological Integrity

A 24-year-old female donated a kidney via a laparoscopic donor nephrectomy to a non-related recipient. The patient is experiencing a lot of pain and refuses to get up to walk. How should the nurse handle this situation? A. Have the transplant psychologist convince her to walk. B. Encourage even a short walk to avoid complications of surgery. C. Tell the patient that no other patients have ever refused to walk. D. Tell the patient she is lucky she did not have an open nephrectomy.

B. Encourage even a short walk to avoid complications of surgery. Because ambulating will improve bowel, lung, and kidney function with improved circulation, even a short walk with assistance should be encouraged after pain medication. The transplant psychologist or social worker's role is to determine if the patient is emotionally stable enough to handle donating a kidney, while postoperative care is the nurse's role. Trying to shame the patient into walking by telling her that other patients have not refused and telling the patient she is lucky she did not have an open nephrectomy (implying how much more pain she would be having if it had been open) will not be beneficial to the patient or her postoperative recovery.

The patient with sudden pain in the left upper quadrant radiating to the back and vomiting was diagnosed with acute pancreatitis. What intervention(s) should the nurse expect to include in the patient's plan of care? A. Immediately start enteral feeding to prevent malnutrition. B. Insert an NG and maintain NPO status to allow pancreas to rest. C. Initiate early prophylactic antibiotic therapy to prevent infection. D. Administer acetaminophen (Tylenol) every 4 hours for pain relief.

B. Insert an NG and maintain NPO status to allow pancreas to rest. Initial treatment with acute pancreatitis will include an NG tube if there is vomiting and being NPO to decrease pancreatic enzyme stimulation and allow the pancreas to rest and heal. Fluid will be administered to treat or prevent shock. The pain will be treated with IV morphine because of the NPO status. Enteral feedings will only be used for the patient with severe acute pancreatitis in whom oral intake is not resumed. Antibiotic therapy is only needed with acute necrotizing pancreatitis and signs of infection.

The nurse is caring for a 55-year-old man patient with acute pancreatitis resulting from gallstones. Which clinical manifestation would the nurse expect the patient to exhibit? A. Hematochezia B. Left upper abdominal pain C. Ascites and peripheral edema D. Temperature over 102o F (38.9o C)

B. Left upper abdominal pain Abdominal pain (usually in the left upper quadrant) is the predominant manifestation of acute pancreatitis. Other manifestations of acute pancreatitis include nausea and vomiting, low-grade fever, leukocytosis, hypotension, tachycardia, and jaundice. Abdominal tenderness with muscle guarding is common. Bowel sounds may be decreased or absent. Ileus may occur and causes marked abdominal distention. Areas of cyanosis or greenish to yellow-brown discoloration of the abdominal wall may occur. Other areas of ecchymoses are the flanks (Grey Turner's spots or sign, a bluish flank discoloration) and the periumbilical area (Cullen's sign, a bluish periumbilical discoloration).

The nurse provides discharge instructions for a 64-year-old woman with ascites and peripheral edema related to cirrhosis. Which statement, if made by the patient, indicates teaching was effective? A. "It is safe to take acetaminophen up to four times a day for pain." B. "Lactulose (Cephulac) should be taken every day to prevent constipation." C. "Herbs and other spices should be used to season my foods instead of salt." D. "I will eat foods high in potassium while taking spironolactone (Aldactone)."

C. "Herbs and other spices should be used to season my foods instead of salt." A low-sodium diet is indicated for the patient with ascites and edema related to cirrhosis. Table salt is a well-known source of sodium and should be avoided. Alternatives to salt to season foods include the use of seasonings such as garlic, parsley, onion, lemon juice, and spices. Pain medications such as acetaminophen, aspirin, and ibuprofen should be avoided as these medications may be toxic to the liver. The patient should avoid potentially hepatotoxic over-the-counter drugs (e.g., acetaminophen) because the diseased liver is unable to metabolize these drugs. Spironolactone is a potassium-sparing diuretic. Lactulose results in the acidification of feces in bowel and trapping of ammonia, causing its elimination in feces.

The nurse is caring for a group of patients. Which patient is at highest risk for pancreatic cancer? A. A 38-year-old Hispanic female who is obese and has hyperinsulinemia B. A 23-year-old who has cystic fibrosis-related pancreatic enzyme insufficiency Incorrect C. A 72-year-old African American male who has smoked cigarettes for 50 years D. A 19-year-old who has a 5-year history of uncontrolled type 1 diabetes mellitus

C. A 72-year-old African American male who has smoked cigarettes for 50 years Risk factors for pancreatic cancer include chronic pancreatitis, diabetes mellitus, age, cigarette smoking, family history of pancreatic cancer, high-fat diet, and exposure to chemicals such as benzidine. African Americans have a higher incidence of pancreatic cancer than whites. The most firmly established environmental risk factor is cigarette smoking. Smokers are two or three times more likely to develop pancreatic cancer as compared with nonsmokers. The risk is related to duration and number of cigarettes smoked.

A patient with cholelithiasis needs to have the gallbladder removed. Which patient assessment is a contraindication for a cholecystectomy? A. Low-grade fever of 100° F and dehydration B. Abscess in the right upper quadrant of the abdomen C. Activated partial thromboplastin time (aPTT) of 54 seconds D. Multiple obstructions in the cystic and common bile duct

C. Activated partial thromboplastin time (aPTT) of 54 seconds An aPTT of 54 seconds is above normal and indicates insufficient clotting ability. If the patient had surgery, significant bleeding complications postoperatively are very likely. Fluids can be given to eliminate the dehydration; the abscess can be assessed, and the obstructions in the cystic and common bile duct would be relieved with the cholecystectomy.

An older male patient visits his primary care provider because of burning on urination and production of urine that he describes as "foul smelling." The health care provider should assess the patient for what factor that may put him at risk for a urinary tract infection (UTI)? A. High-purine diet B. Sedentary lifestyle C. Benign prostatic hyperplasia (BPH) D. Recent use of broad-spectrum antibiotics

C. Benign prostatic hyperplasia (BPH) BPH causes urinary stasis, which is a predisposing factor for UTIs. A sedentary lifestyle and recent antibiotic use are unlikely to contribute to UTIs, whereas a diet high in purines is associated with renal calculi.

The patient has scleroderma and is experiencing hypertension. The nurse should know that this could be related to which renal problem? A. Obstructive uropathy B. Goodpasture syndrome C. Chronic glomerulonephritis D. Calcium oxalate urinary calculi

C. Chronic glomerulonephritis Hypertension occurs with chronic glomerulonephritis that may be found in patients with scleroderma. Obstructive uropathy, Goodpasture syndrome, and calcium oxalate urinary calculi are not related to scleroderma and do not cause hypertension.

Which effect of aging on the urinary system is most likely to affect the action of bumetanide (Bumex)? A. Benign enlargement of prostatic tissues B. Decreased sensation of bladder capacity C. Decreased function of the loop of Henle D. Less absorption in the Bowman's capsule

C. Decreased function of the loop of Henle Bumetanide (Bumex) is a loop diuretic that acts in the loop of Henle to decrease reabsorption of sodium and chloride. Because the loop of Henle loses function with aging, the excretion of drugs becomes less and less efficient. Thus the circulating levels of drugs are increased and their effects prolonged. The benign enlargement of prostatic tissue, decreased sensation of bladder capacity, and loss of concentrating ability do not directly affect the action of loop diuretics.

The condition of a patient who has cirrhosis of the liver has deteriorated. Which diagnostic study would help determine if the patient has developed liver cancer? A. Serum α-fetoprotein level B. Ventilation/perfusion scan C. Hepatic structure ultrasound D. Abdominal girth measurement

C. Hepatic structure ultrasound Correct Hepatic structure ultrasound, CT, and MRI are used to screen and diagnose liver cancer. Serum α-fetoprotein level may be elevated with liver cancer or other liver problems. Ventilation/perfusion scans do not diagnose liver cancer. Abdominal girth measurement would not differentiate between cirrhosis and liver cancer.

A patient is recovering in the intensive care unit (ICU) after receiving a kidney transplant approximately 24 hours ago. What is an expected assessment finding for this patient during this early stage of recovery? A. Hypokalemia B. Hyponatremia C. Large urine output D. Leukocytosis with cloudy urine output

C. Large urine output Patients frequently experience diuresis in the hours and days immediately following a kidney transplant. Electrolyte imbalances and signs of infection are unexpected findings that warrant prompt intervention.

When caring for a patient with nephrotic syndrome, the nurse should know the patient understands dietary teaching when the patient selects which food item? A. Peanut butter and crackers B. One small grilled pork chop C. Salad made of fresh vegetables D. Spaghetti with canned spaghetti sauce

C. Salad made of fresh vegetables Of the options listed, only salad made with fresh vegetables would be acceptable for the diet that limits sodium and protein as well as saturated fat if hyperlipidemia is present. Peanut butter and crackers are processed so they contain significant sodium, and peanut butter contains some protein. A pork chop is a high-protein food with saturated fat. Canned spaghetti sauce is also high in sodium.

When preparing a patient for a capsule endoscopy study, what should the nurse do? a. Ensure the patient understands the required bowel preparation. b. Have the patient return to the procedure room for removal of the capsule. c. Teach the patient to maintain a clear liquid diet throughout the procedure. d. Explain to the patient that conscious sedation will be used during placement of the capsule.

Correct answer: a Rationale: A capsule endoscopy study involves the patient performing a bowel prep to cleanse the bowel before swallowing the capsule. The patient will be on a clear liquid diet for 1 to 2 days before the procedure and will remain NPO for 4 to 6 hours after swallowing the capsule. The capsule is disposable and will pass naturally with the bowel movement, although the monitoring device will need to be removed.

The nurse instructs an obese 22-year-old man with a sedentary job about the health benefits of an exercise program. The nurse evaluates that teaching is effective when the patient makes which statement? a. "The goal is to walk at least 10,000 steps every day of the week." b. "Weekend aerobics for 2 hours is better than exercising every day." c. "Aerobic exercise will increase my appetite and result in weight gain." d. "Exercise causes weight loss by decreasing my resting metabolic rate."

Correct answer: a Rationale: A realistic activity goal is to walk 10,000 steps a day. Increased activity does not promote an increase in appetite or lead to weight gain. Exercise should be done daily, preferably 30 minutes to an hour a day. Exercise increases metabolic rate.

What problem should the nurse assess the patient for if the patient was on prolonged antibiotic therapy? a. Coagulation problems b. Elevated serum ammonia levels c. Impaired absorption of amino acids d. Increased mucus and bicarbonate secretion

Correct answer: a Rationale: Bacteria int he colon (1) synthesize vitamin K, which is needed for the production of prothrombin by the liver and (2) deaminate undigested or non absorbed proteins, producing ammonia, which is converted to urea by the liver. A reduction in normal flora bacteria by antibiotic therapy can lead to decreased vitamin K, resulting in decreased prothrombin and coagulation problems. Bowel bacteria do not influence protein absorption or the secretion of mucus.

Priority Decision: The nurse is caring for a patient receiving 1000 mL of parenteral nutrition solution over 24 hours. When it is time to change the solution, 150 mL remain in the bottle. What is the most appropriate action by the nurse? a. Hang the new solution and discard the unused solution. b. Open the IV line and rapidly infuse the remaining solution. c. Notify the health care provider for instructions regarding the infusion rate. d. Wait to change the solution until the remaining solution infuses at the proscribed rate.

Correct answer: a Rationale: Bacterial growth occurs at room temperature in nutritional solutions. Therefore solutions must not be infused for longer than 24 hours. Remaining solution should be discard. Speeding up the solution may cause hyperglycemia and should not be done. The health care provider does not need to be notified.

A patient is jaundiced and her stools are clay colored (gray). This is most likely related to a. decreased bile flow into the intestine. b. increase production of urobilinogen. c. increased production of cholecystokinin. d. increased bile and bilirubin in the blood.

Correct answer: a Rationale: Bile is produced by the hepatocytes and is stored and concentrated in the gallbladder. When bile is released from the common bile duct, it enters the duodenum. In the intestines, bilirubin is reduced to stercobilinogen and urobilinogen by bacterial action. Stercobilinogen accounts for the brown color of stool. Stools may be clay-colored if bile is not released from the common bile duct into the duodenum. Jaundice may result if the bilirubin level in the blood is elevated.

What characterizes auscultation of the abdomen? a. The presence of borborygmi indicates hyper peristalsis. b. The bell of the stethoscope is used to auscultate high-pitched sounds. c. High-pitched, rushing, and tinkling bowel sounds are heard after eating. d. Absence of bowel sounds for 1 minute in each quadrant is reported as abnormal.

Correct answer: a Rationale: Borborygmi are loud gurgles (stomach growling) that indicate hyper peristalsis. Normal bowel sounds are relatively high-pitched and are heard best with the diaphragm of the stethoscope. High-pitched, tinkling bowel sounds occur when the intestines are under tension, as in bowel obstructions. Absent bowel sounds may be reported when no sounds are heard for 2 to 3 minutes in each quadrant.

What information should be included in the dietary teaching for the patient following a Roux-en-Y gastric bypass? a. Avoid sugary foods and limit fluids to prevent dumping syndrome. b. Gradually increase the amount of food ingested to preoperative levels. c. Maintain a long-term liquid diet to prevent damage to the surgical site. d. Consume foods high in complex carbohydrates, protein, and fiber to add bulk to contents.

Correct answer: a Rationale: Fluids and foods high in carbohydrates tend to promote diarrhea and symptoms of dumping syndrome in patients with gastric bypass surgery. The diet generally should be high in protein and low in carbohydrates, fat, and roughage and consists of six small feedings a day because of the small stomach size. Liquid diets are likely to be used longer for the patient with a gastroplasty.

A patient had a stomach resection for stomach cancer. The nurse should teach the patient about the loss of the hormone that stimulates gastric acid secretion and motility and maintains lower esophageal sphincter tone. Which hormone will be decreased with a gastric resection? a. Gastrin b. Secretin c. Cholecystokinin d. Gastric inhibitory peptide

Correct answer: a Rationale: Gastrin is the hormone activated in the stomach (and duodenal mucosa) by stomach distention that stimulates gastric acid secretion and motility and maintains lower esophageal sphincter tone. Secretin inhibits gastric motility and acid secretion and stimulates pancreatic bicarbonate secretion. Cholecystokinin allows increased flow of bile into the duodenum and release of pancreatic digestive enzymes. Gastric inhibitory peptide inhibits gastric acid secretion and motility.

A patient has been on a 1000-calorie diet with a daily exercise routine. In 2 months, the patient has lost 20 lb (9kg) toward a goal of 50 lb (23 kg) but is now discouraged that no weight has been lost in the last 2 weeks. What should the nurse tell the patient about this? a. Plateaus where no weight is lost normally occur during a weight-loss program. b. A weight considered by the body to most efficient for functioning has been reached. c. A return to former eating habits is the most common cause of not continuing to lose weight. d. A steady weight may be due to water gain from eating foods high in sodium.

Correct answer: a Rationale: Plateau periods during which no weight is lost are normal occurrences during weight reduction and may last for several days to several weeks but weight loss will resume if the prescribed weight reduction plan is continued. Weight loss may stop if former eating habits are resumed but this not the most common cause of plateaus.

When considering tube feedings for a patient with severe protein-calorie malnutrition, what is an advantage of a gastrostomy tube versus a nasogastric (NG) tube? a. There is less irritation to the nasal and esophageal mucosa. b. The patient experiences the sights and smells associated with eating. c. Aspiration resulting from reflux of formulas into the esophagus is less common. d. Routine checking for placement is not required because gastrostomy tubes do not become displaced

Correct answer: a Rationale: Standard nasogastric (NG) tubes are used for tube feedings for short-term feeding problems because prolonged therapy can result in irritation and erosion of the mucosa of the upper GI tract. Gastric reflux and the potential for aspiration can occur with both tubes that deliver fluids into the stomach. Both NG and gastrostomy tubes can become displaced and deprive the patient of the sensations associated with eating.

The percentage of daily calories for a healthy individual consists of a. 50% carbohydrates, 25% protein, 25% fat, and <10% of fat from saturated fatty acids. b. 65% carbohydrates, 25% protein, 25% fat, and >10% of fat from saturated fatty acids. c. 50% carbohydrates, 40% protein, 10% fat, and <10% of fat from saturated fatty acids. d. 40% carbohydrates, 30% protein, 30% fat, and >10% of fat from saturated fatty acids.

Correct answer: a Rationale: The 2005 Dietary Guidelines for Americans recommend that 45% to 65% of total calories should come from carbohydrates. Ideally, 10% to 35% of daily caloric needs should come from protein. Individuals should limit their fat intake to 20% to 35% of total calories. Additional recommendations focus on the type of fat consumed because diets high in excess calories, usually in the form of fats, contribute to the development of obesity. Individuals should consume less than 10% of calories from saturated fatty acids, limit intake of fat and oils high in trans fatty acids, and should limit intake of dietary cholesterol to 300 mg/day.

During the initial postoperative period following bariatric surgery, the nurse recognizes the importance of monitoring obese patients for respiratory insufficiency based on what knowledge? a. The body stores anesthetics in adipose tissue. b. Postoperative pain may cause a decreased respiratory rate. c. Intubation may be difficult because of extra chin skinfolds. d. The patient's head must remain flat for a minimum of 2 hours postprocedure.

Correct answer: a Rationale: The body stores anesthetics in adipose tissue, placing patients with excess adipose tissue at risk for re-sedation. As adipose cells release anesthetics back into the bloodstream, the patient may become sedated after surgery, increasing the risk of hypoventilation and resultant respiratory insufficiency. Difficult intubation does not cause respiratory insufficiency. Pain usually increases respiratory rate. The patient's head should be elevated after bariatric surgery to decrease abdominal pressure and facilitate respirations.

The nurse evaluates that patient teaching about a high-calorie, high-protien diet has been effective when the patient selects which breakfast option from the hospital menu? a. Two poached eggs, hash brown potatoes, and whole milk b. Two slices of toast with butter and jelly, orange juice, and skim milk c. Three pancakes with butter and syrup, two slices of bacon, and apple juice d. Cream of wheat with 2 tbsp of skim milk powder, one half grapefruit, and a high-protein milkshake

Correct answer: a Rationale: The breakfast with the eggs provides 24 g or protein, compared with 14 g for the protein-fortified cream of wheat and milkshake breakfast. Whole milk instead of skim milk helps to meet the calorie requirements. The toast has 10 g of protein and the pancakes have about 6 g. Bacon is considered a fat rather than a meat serving.

The nurse cares for a 34-year-old woman after bariatric surgery. The nurse determines that discharge teaching related to diet is successful if the patient makes which statement? a. "A high protein diet that is low in carbohydrates and fat will prevent diarrhea." b. "Food should be high in fiber to prevent constipation from the pain medication." c. "Three meals a day with no snacks between meals will provide optimal nutrition." d. "Fluid intake should be at least 2000 mL per day with meals to avoid dehydration."

Correct answer: a Rationale: The diet generally prescribed is high in protein and low in carbohydrates, fat, and roughage and consists of six small feedings daily. Fluids should not be ingested with the meal, and in some cases, fluids should be restricted to less than 1000 mL per day. Fluids and foods high in carbohydrate tend to promote diarrhea and symptoms of the dumping syndrome. Generally, calorically dense foods (foods high in fat) should be avoided to permit more nutritionally sound food to be consumed.

Checking for the return of the gag reflex and monitoring for LUQ pain, nausea and vomiting are necessary nursing actions after which diagnostic procedure? a. ERCP b. Colonoscopy c. Barium swallow d. Esophagogastroduodenoscopy (EGD)

Correct answer: a Rationale: The left upper quadrant (LUQ) pain and nausea and vomiting could occur from perforation. The return of gag reflex is essential to prevent aspiration after an ERCP. The gag reflex is also assessed with an EGD. These are not relevant assessments for the colonoscopy and barium swallow.

The nurse confirms initial placement of a blindly inserted small-bore NG feeding tube by a. x-ray. b. air insufflation. c. observing patient for coughing. d. pH measurement of gastric aspirate.

Correct answer: a Rationale: The nurse should obtain x-ray confirmation to determine whether a blindly placed nasogastric or orogastric tube (small bore or large bore) is properly positioned in the gastrointestinal tract before administering feedings or medications.

The nurse teaches a 50-year-old woman who has a body mass index (BMI) of 39 kg/m2 about weight loss. Which dietary change would be appropriate for the nurse to recommend to this patient? a. Decrease fat intake and control portion size b. Increase vegetables and decrease fluid intake c. Increase protein intake and avoid carbohydrates d. Decrease complex carbohydrates and limit fiber

Correct answer: a Rationale: The safest dietary guideline for weight loss is to decrease caloric intake by maintaining a balance of nutrients and adequate hydration while controlling portion size and decreasing fat intake.

During assessment of the patient with protein-calorie malnutrition, what should the nurse expect to find (select all that apply)? a. Frequent cold symptoms b. Decreased bowel sounds c. Cool, rough, dry, scaly skin d. A flat or concave abdomen e. Prominent bony structures f. Decreased reflexes and lack of attention

Correct answer: a, b, c, e, f Rationale: In malnutrition, metabolic processes are slowed, leading to increased sensitivity to cold, decreased heart rate (HR) and cardiac output (CO), and decreased neurologic function. Because of slowed GI motility and absorption, the abdomen becomes distended and protruding and bowel sounds are decreased. Skin is rough, dry, and scaly whereas bone structures protrude because of muscle loss. Because the immune system is weakened, susceptibility to respiratory infections is increased.

The stable patient has a gastrostomy tube for enteral feeding. Which care could the RN delegate to the LPN (select all that apply)? a. Administer bolus or continuous feedings. b. Evaluate the nutritional status of the patient. c. Administer medications through the gastrostomy tube. d. Monitor for complications related to the tube and enteral feeding. e. Teach the caregiver about feeding via the gastrostomy tube at home.

Correct answer: a, c Rationale: For the stable patient, the LPN can administer bolus or continuous feedings and administer medications through the gastrostomy. The RN must evaluate the nutritional status of the patient, monitor for complications related to the tube and the enteral feeding, and teach the caregiver about feeding via the gastrostomy tube at home.

A woman is 5 ft, 6 in (166 cm) tall and weighs 200 lb (90.9 kg) with a waist-to-hip ratio of 0.7. The nurse counsels the patient with the knowledge that the patient is at greatest risk for a. heart disease. b. osteoporosis. c. diabetes mellitus. d. endometrial cancer.

Correct answer: b Rationale: A patient who is obese (BMI of 32.2) but has a waist-to-hip ratio of less than 0.8, indicating gynoid obesity, has an increased risk for osteoporosis. The other conditions are risks associated with android obesity.

A 22-year-old female is admitted with anorexia nervosa and a serum potassium level of 2.4 mEq/L. What complication is most important for the nurse to observe for in this patient? a. Muscle weakness b. Cardiac dysrhythmias c. Increased urine output d. Anemia and leukopenia

Correct answer: b Rationale: A serum potassium level less than 2.5 mEq/L indicates severe hypokalemia, which can lead to life-threatening cardiac dysrhythmias (e.g., bradycardia, tachycardia, ventricular dysrhythmias). Other manifestations of potassium deficiency include muscle weakness and renal failure. Patients with anorexia nervosa commonly have iron-deficiency anemia and an elevated blood urea nitrogen level related to intravascular volume depletion and abnormal renal function.

When teaching the older adult about nutritional needs during aging, what does the nurse emphasize? a. Need for all nutrients decreases as one ages. b. Fewer calories, but the same or slightly increased amount of protein, are required as one ages. c. Fats, carbohydrates, and protein should be decreased, but vitamin and mineral intake should be increased. d. High-calorie oral supplements should be taken between meals to ensure that recommended nutrient needs are met.

Correct answer: b Rationale: Although calorie intake should be decreased in the older adult because of decreased activity and basal metabolic rate, the need for specific nutrients, such as proteins and vitamins, does not change.

A patient who has suffered severe burns in a motor vehicle accident will soon be started on parenteral nutrition (PN). Which principle should guide the nurse's administration of the patient's nutrition? a. Administration of PN requires clean technique. b. Central PN requires rapid dilution in a large volume of blood. c. Peripheral PN delivery is preferred over the use of a central line. d. Only water-soluble medications may be added to the PN by the nurse.

Correct answer: b Rationale: Central PN is hypertonic and requires rapid dilution in a large volume of blood. Because PN is an excellent medium for microbial growth, aseptic technique is necessary during administration. Administration through a central line is preferred over the use of peripheral PN, and the nurse may not add any medications to PN.

The nurse is teaching a female patient with type 1 diabetes mellitus about nutrition before discharge. She had surgery to revise a lower leg stump with a skin graft. What food should the nurse teach the patient to eat to best facilitate healing? a. Non-fat milk b. Chicken breast c. Fortified oatmeal d. Olive oil and nuts

Correct answer: b Rationale: High quality protein such as chicken breast is important for tissue repair. Although the non-fat milk, nuts, and fortified oatmeal have some protein, they do not have as much as the chicken breast.

During starvation, the order in which the body obtains substrate for energy is a. visceral protein, skeletal protein, fat, glycogen b. glycogen, skeletal protein, fat stores, visceral protein c. visceral protein, fat stores, glycogen, skeletal protein d. fat stores, skeletal protein, visceral protein, glycogen

Correct answer: b Rationale: Initially, the body selectively uses carbohydrates (e.g., glycogen) rather than fat and protein to meet metabolic needs. These carbohydrate stores, found in the liver and muscles, are minimal and may be totally depleted within 18 hours. After carbohydrate stores are depleted, skeletal protein begins to be converted to glucose for energy. Within 5 to 9 days, body fat is fully mobilized to supply much of the needed energy. In prolonged starvation, up to 97% of calories are provided by fat, and protein is conserved. Depletion of fat stores depends on the amount available, but fat stores typically are used up in 4 to 6 weeks. After fat stores are used, body or visceral proteins, including those in internal organs and plasma, can no longer be spared and rapidly decrease because they are the only remaining body source of energy available.

Priority Decision: When medications are used in the treatment of obesity, what is most important for the nurse to teach the patient? a. Over-the-counter (OTC) diet aids are safer than other agents and con be useful in controlling appetite. b. Drugs should be used only as adjuncts to a diet and exercise program as treatment for a chronic condition. c. All drugs used for weight control are capable of altering central nervous system (CNS) function and should be used with caution. d. The primary effect of the medications is psychologic, controlling the urge to eat in response to stress or feelings of rejection.

Correct answer: b Rationale: Medications are used only as adjuncts to diet and exercise programs in the treatment of obesity. Drugs do not cure obesity; without changes in food intake and physical activity, weight gain will occur when the medications are discontinued. The medications used work in a variety of ways to control appetite but over-the-counter drugs are probably the least effective and most abused of these drugs.

A frail 74-year-old man with recent severe weight loss is instructed to eat a high-protein, high-calorie diet at home. If the man likes all of the items below, which would be the most appropriate for the nurse to suggest? a. Orange juice and dry toast b. Oatmeal, butter, and cream c. Steamed carrots and chicken broth d. Banana and unsweetened applesauce

Correct answer: b Rationale: Oatmeal, butter, and cream are examples of food items that would be appropriate to include for a patient on a high-protein, high-calorie diet.

An 80-year-old man states that, although be adds a lot of salt to his food, it still does not have much taste. The nurse's response is based on the knowledge that the older adult a. should not experience changes in taste. b. has a loss of taste buds, especially for sweet and salty. c. has some loss of taste but no difficulty chewing food. d. loses the sense of taste because the ability to smell is decreased.

Correct answer: b Rationale: Older adults have decreased numbers of taste buds and a decreased sense of smell. These age-related changes diminish the sense of taste (especially of salty and sweet substances).

Priority Decision: The nurse admitting a patient for bariatric surgery obtains the following information from the patient. Which finding should be brought to the surgeon's attention before proceeding with further patient preparation? a. History of hypertension b. History of untreated depression c. History of multiple attempts at weight loss d. History of sleep apnea treated with continuous positive airway pressure (CPAP)

Correct answer: b Rationale: Patients with histories of untreated depression or psychosis are not good candidates for surgery. All other historical information includes medical complications of severe obesity that would help to qualify the patient for the surgery.

An older patient was admitted with a fractured hip after being found on the floor of her home. She was extremely malnourished and started on parenteral nutrition (PN) 3 days ago. Which assessment finding would be of most concern to the nurse? a. Blood glucose level of 125 mg/dL b. Serum phosphate level of 1.9 mg/dL c. White blood cell count of 10,500/µL d. Serum potassium level of 4.6 mEq/L

Correct answer: b Rationale: Refeeding syndrome can occur if a malnourished patient is started on aggressive nutritional support. Hypophosphatemia (serum phosphate level less than 2.4 mg/dL) is the hallmark of refeeding syndrome and could result in cardiac dysrhythmias, respiratory arrest, and neurologic problems. An increase in the blood glucose level is expected during the first few days after PN is started. The goal is to maintain a glucose range of 110 to 150 mg/dL. An elevated white blood cell count (greater than 11,000/µL) could indicate an infection. Normal serum potassium levels are between 3.5 and 5.0 mEq/L.

A patient who has dysphagia as a consequence of a stroke is receiving enteral feedings through a percutaneous endoscopic gastrostomy (PEG). What intervention should the nurse integrate into this patient's care? a. Flush the tube with 30 mL of normal saline every 4 hours. b. Flush the tube before and after feedings if the patient's feedings are intermittent. c. Flush the PEG with 100 mL of sterile water before and after medication administration. d. To prevent fluid overload, avoid flushing when the patient is receiving continuous feeding.

Correct answer: b Rationale: The nurse should flush feeding tubes with 30 mL of water (not normal saline) every 4 hours and before and after medication administration during continuous feeding or before and after intermittent feeding. Flushes of 100 mL are excessive and may cause fluid overload in the patient.

During an examination of the abdomen the nurse should a. position the patient in the supine position with the head of the bed flat and knees straight. b. listen in the epigastrium and all four quadrants for 2 minutes for bowel sounds. c. use the following order of techniques: inspection, palpation, percussion, auscultation. d. describe bowel sounds as absent if no sound is heard in the lower right quadrant after 2 minutes.

Correct answer: b Rationale: The nurse should listen in the epigastrium and all four quadrants for bowel sounds for at least 2 minutes. The patient should be in the supine position and should slightly flex the knees; the head of the bed should be raised slightly. During examination of the abdomen, the nurse auscultates before performing percussion and palpation because the latter procedures may alter the bowel sounds. Bowel sounds cannot be described as absent until no sound is heard for 5 minutes in each quadrant.

The nurse has completed initial instruction with a patient regarding a weight loss program. The nurse determines that the teaching has been effective when the patient makes which statement? a. "I plan to lose 4 lb a week until I have lost the 60-pound goal." b. "I will keep a diary of weekly weights to illustrate my weight loss." c. "I will restrict my carbohydrate intake to less than 30 g/day to maximize weight loss." d."I should not exercise more than my program requires since increased activity increases the appetite."

Correct answer: b Rationale: The patient should monitor and record weight once per week. This prevents frustration at the normal variations in daily weights and may help the patient to maintain motivation to stay on the prescribed diet. Weight loss should occur at a rate of 1 to 2 lb/week. The diet should be well balanced rather than lacking in specific components that may cause an initial weight loss but is not usually sustainable. Exercise is a necessary component of any successful weight loss program.

Which assessment should the nurse prioritize in the care of a patient who has recently begun receiving parenteral nutrition (PN)? a. Skin integrity and bowel sounds b. Electrolyte levels and daily weights c. Auscultation of the chest and tests of blood coagulability d. Peripheral vascular assessment and level of consciousness (LOC)

Correct answer: b Rationale: The use of PN necessitates frequent and thorough assessments. Key focuses of these assessments include daily weights and close monitoring of electrolyte levels. Assessments of bowel sounds, integument, peripheral vascular system, LOC, chest sounds, and blood coagulation may be variously performed, but close monitoring of fluid and electrolyte balance supersedes these in importance.

The patient cannot afford to buy the food she needs for her family, so she makes sure her children eat first, and then she eats. When she comes to the clinic, she reports bleeding gums, loose teeth, and dry, itchy skin. The nurse should know that this patient is most likely lacking which vitamin? a. Folic acid b. Vitamin C c. Vitamin D d. Vitamin K

Correct answer: b Rationale: This patient is lacking Vitamin C as evidenced by the bleeding gums, loose teeth, and dry, itchy skin. Clinical manifestations of folic acid deficiency include megaloblastic anemia, anorexia, fatigue, sore tongue, diarrhea, or forgetfulness. Clinical manifestations of Vitamin D deficiency include muscular weakness, excess sweating, diarrhea, bone pain, rickets, or osteomalacia. Clinical manifestations of Vitamin K deficiency include defective blood coagulation.

Which female patient is most likely to have metabolic syndrome? a. BP 128/78 mm Hg, triglycerides 160 mg/dL, fasting blood glucose 102 mg/dL b. BP 142/90 mm Hg, high-density lipoproteins 45 mg/dL, fasting blood glucose 130 mg/dL c. Waist circumference 36 in, triglycerides 162 mg/dL, high-density lipoproteins 55 mg/dL d. Waist circumference 32 in, high-density lipoproteins 38 mg/dL, fasting blood glucose 122 mg/dL

Correct answer: b Rationale: Three of the following five measures are needed for a woman to be diagnosed with metabolic syndrome: waist circumference >35 in, triglycerides >150 mg/dL, high-density lipoproteins <50 mg/dL, BP >130 mm Hg systolic or >85 mm Hg diastolic, fasting blood glucose >110 mg/dL. Although the other options have some abnormal measures, none has all three measures in the diagnostic ranges. The criteria for metabolic syndrome for both women and men are listed in Table 41-10.

Which statement about obesity is explained by genetics? a. Older obese patients have exacerbated changes of aging. b. Android body shape and weight gain are influenced by genetics. c. White Americans have a higher incidence of obesity than African Americans. d. Men have a harder time losing weight, as they have more muscle mass than women.

Correct answer: b Rationale: Twin studies and studies with adopted children have shown that body shape and weight gain are influenced by genetics but more research is needed. Older obese people do have exacerbated aging problems related to declines in physical function. African Americans and Hispanics have a higher incidence of obesity than whites. Women have a higher incidence of obesity and more difficulty losing weight than men because women have a higher percentage of metabolically less-active fat.

Inspection of an older patient's mouth reveals the presence of white, curd-like lesions on the patient's tongue. What is the most likely etiology for this abnormal assessment finding? a. Herpesvirus b. Candida albicans c. Vitamin deficiency d. Irritation from ill-fitting dentures

Correct answer: b Rationale: White, curd-like lesions surrounded by erythematous mucosa are associated with oral candidiasis. Herpesvirus causes benign vesicular lesions in the mouth. Vitamin deficiencies may cause a reddened, ulcerated, swollen tongue. Irritation from ill-fitting dentures will cause friable, edematous, painful, bleeding gingivae.

Priority Decision: Before administering a bolus of intermittent tube feeding to a patient with a percutaneous endoscopic gastrostomy (PEG), the nurse aspirates 220 mL of gastric contents. How should the nurse respond? a. Return the aspirate to the stomach and recheck the volume of aspirate in an hour. b. Return the aspirate to the stomach and continue with tube feeding as planned. c. Discard the aspirate to prevent over distending the stomach when the new feeding is given. d. Notify the health care provider that the feedings have been scheduled too frequently to allow for stomach emptying.

Correct answer: b Rationale: With intermittent feedings, less than 250 mL residual does not require further action. With continuous feedings and a residual of 250 mL or more after the second residual check, a pro motility agent should be considered.

Which digestive substances are active or activated in the stomach (select all that apply)? a. Bile b. Pepsin c. Gastrin d. Maltase e. Secretin f. Amylase

Correct answer: b, c Rationale: Pepsinogen is changed to pepsin by acidity of the stomach, where it begins to break down proteins. Gastrin stimulates gastric acid secretion and motility and maintains lower esophageal sphincter tone. The stomach also secretes lipase for fat digestion.Bile is secreted by the liver and stored in the gallbladder for emulsifying fats. Maltase is secreted in the small intestine and converts maltose to glucose. Secretin is secreted y the duodenal mucosa and inhibits gastric motility and acid secretion. Amylase is secured in the small intestine and by the pancreas for carbohydrate digestion.

The nurse should recognize that the liver performs which functions (select all that apply) a. Bile storage b. Detoxification c. Protein metabolism d. Steroid metabolism e. Red blood cell (RBC) destruction

Correct answer: b, c, d Rationale: The liver performs multiple major functions that aid in the maintenance of homeostasis. These include metabolism of proteins and steroids as well as detoxification of drugs and metabolic waste products. The Kupffer cells of the liver participate in the breakdown of old RBCs. The liver produces bile, but storage occurs in the gall bladder.

The patient has parenteral nutrition infusing with amino acids and dextrose. In report, the oncoming nurse is told that the tubing, the bag, and the dressing were changed 22 hours ago. What care should the nurse coming on be prepared to do (select all that apply)? a. Give the patient insulin. b. Check amount of feeding left in the bag. c. Check that the next bag has been ordered. d. Check the insertion site and change the tubing. e. Check the label to ensure ingredients and solution are as ordered.

Correct answer: b,c,e Rationale: The nurse should check the amount of feeding left in the bag, and that the next bag has been ordered to be sure the solution will not run out before the next bag is available. Parenteral nutrition solutions are only good for 24 hours and usually take some time for the pharmacy to mix for each patient. The label on the bag should be checked to ensure that the ingredients and solution are what was ordered. The patient would only receive insulin if the patient is experiencing hyperglycemia and was receiving sliding scale insulin or had diabetes mellitus. The insertion site should be checked, but the tubing is only changed every 72 hours unless lipids are being used.

The patient tells the nurse she had a history of abdominal pain, so she had a surgery to make an opening into the common bile duct to remove stones. The nurse knows that this surgery is called a a. colectomy b. cholecystectomy c. choledocholithotomy d. choledochojejunostomy

Correct answer: c Rationale: A choledocholithotomy is an opening into the common bile duct for the removal of stones. A colectomy is the removal of the colon. The cholecystectomy is the removal of the gallbladder. The choledochojejunostomy is an opening between the common bile duct and the jejunum.

What is a normal finding during physical assessment of the mouth? a. A red, slick appearance of the tongue b. Uvular deviation to the side on saying "Ahh" c. A thin, white coating of the dorsum of the tongue d. Scattered red, smooth areas on the dorsum of the tongue

Correct answer: c Rationale: A thin white coating of the dorsum (top) of the tongue is normal. A red, slick appearance is characteristic of cobalamin deficiency and scattered red, smooth areas on the tongue are known as geographic tongue. The uvula should remain in the midline while the patient is saying "Ahh"

The nurse recognizes that the majority of patients' caloric needs should come from which source? a. Fats b. Proteins c. Polysaccharides d. Monosaccharides

Correct answer: c Rationale: Carbohydrates should constitute between 45% and 65% of caloric needs, compared with 20% to 35% from fats and 10% to 35% from proteins. Polysaccharides are the complex carbohydrates that are contained in breads and grains. Monosaccharides are simple sugars.

When assessing a patient's abdomen, what would be most appropriate for the nurse to do? a. Palpate the abdomen before auscultation. b. Percuss the abdomen before auscultation. c. Auscultate the abdomen before palpation. d. Perform deep palpation before light palpation.

Correct answer: c Rationale: During examination of the abdomen, auscultation is done before percussion and palpation because these latter procedures may alter the bowel sounds.

In developing an effective weight reduction plan for an overweight patient who states a willingness to try to lose weight, it is most important for the nurse to first assess which factor? a. The length of time the patient has been obese b. The patient's current level of physical activity c. The patient's social, emotional, and behavioral influences on obesity d. Anthropometric measurements, such as body mass index and skinfold thickness

Correct answer: c Rationale: Eating patterns are established early in life, and eating has many meanings for people. To establish a weight reduction plan that will be successful for the patient, the nurse should first explore the social, emotional, and behavioral influences on the patient's eating patterns. The duration of obesity, current physical activity level, and current anthropometric measurements are not as important for the weight reduction plan.

The health care team is assessing a male patient for acute pancreatitis after he presented to the emergency department with severe abdominal pain. Which laboratory value is the best diagnostic indicator of acute pancreatitis? a. Gastric pH b. Blood glucose c. Serum amylase d. Serum potassium

Correct answer: c Rationale: Elevated serum amylase levels indicate early pancreatic dysfunction and are used to diagnose acute pancreatitis. Serum lipase levels stay elevated longer than serum amylase in acute pancreatitis. Blood glucose, gastric pH, and potassium levels are not direct indicators of acute pancreatic dysfunction.

The nurse is evaluating the nutritional status of a 55-year-old man who is undergoing radiation treatment for oropharyngeal cancer. Which laboratory test would be the best indicator to determine if the patient has protein-calorie malnutrition? a. Serum transferrin b. C-reactive protein c. Serum prealbumin d. Alanine transaminase (ALT)

Correct answer: c Rationale: In the absence of an inflammatory condition, the best indicator of protein-calorie malnutrition (PCM) is prealbumin; prealbumin is a protein synthesized by the liver and indicates recent or current nutritional status. Decreased albumin and transferrin levels are other indicators that protein is deficient. C-reactive protein (CRP) is elevated during inflammation and is used to determine if prealbumin, albumin, and transferrin are decreased related to protein deficiency or an inflammatory process. Other indicators of protein deficiency include elevated serum potassium levels, low red blood cell counts and hemoglobin levels, decreased total lymphocyte count, elevated liver enzyme levels (ALT), and decreased levels of both fat-soluble and water-soluble vitamins.

Which statement accurately describes vitamin deficiencies? a. The two nutrients most often lacking in the diet of a vegan are vitamin B6 and folic acid. b. Vitamin imbalances occur frequently in the United States because of excessive fat intake. c. Surgery on the GI tract may contribute to vitamin deficiencies because of impaired absorption. d. Vitamin deficiencies in adults most commonly are clinically manifested by disorders of the skin.

Correct answer: c Rationale: Patients who have surgery on the GI tract may be at risk for vitamin deficiencies because of inability to absorb or metabolize them. The strict vegan diet most often lacks cobalamin (vitamin B12) and iron. Although the high intake of fat is a major nutritional problem in the United States, vitamin deficiencies are rare in developed countries except in those with eating disorders or chronic alcohol abusers. Some vitamin deficiencies in adults have neurologic manifestations.

Priority Decision: During care of the severely obese patient, what is most important for the nurse to do? a. Avoid reference to the patient's weight to avoid embarrassing the patient. b. Emphasize to the patient how important it is to lose weight to maintain health. c. Plan for necessary modifications in equipment and nursing techniques before initiating care. d. Recognize that a full assessment of each body system might not be possible because of numerous layers of skinfolds.

Correct answer: c Rationale: Special considerations are needed for the care of the severely obese patient because most hospital units are not prepared with beds, chairs, BP cuffs, and other equipment that will need to be used with the very obese patient. Consideration of all aspects of care should be made before implementing care for the patient, including extra time and perhaps assistance for positioning, physical assessment, and transferring the patient.

How will an obstruction at the ampulla of Vater affect the digestion of all nutrients? a. Bile is responsible for emulsification of all nutrients and vitamins. b. Intestinal digestive enzymes are released through the ampulla of Vater. c. Both bile and pancreatic enzymes enter the duodenum at the ampulla of Vater. d. Gastric contents can ply pass to the duodenum when the ampulla of Vater is open.

Correct answer: c Rationale: The ampulla of Vater is the site where the pancreatic duct and common bile duct enter the duodenum and the opening and closing of the ampulla is controlled by the sphincter of Oddi. Because bile from the common bile duct is needed for emulsification of fat to promote digestion and pancreatic enzymes from the pancreas are needed for digestion of all nutrients, a blockage at this point would affect the digestion of all nutrients. Gastric contents pass into the duodenum through the pylorus or pyloric valve.

A patient's serum liver enzyme tests reveal an elevated aspartate aminotransferase (AST). The nurse recognizes what about the elevated AST? a. It eliminates infection as a cause of liver damage. b. It is diagnostic for liver inflammation and damage. c. Tissue damage in organs other than the liver may be identified. d. Nervous system symptoms related to hepatic encephalopathy may be the cause.

Correct answer: c Rationale: The aspartate aminotransferase (AST) level is elevated in liver disease but it is important to note that it is also elevated in damage to the heart and lungs and is not a specific test for liver function. Measurements of most of the transaminases involves nonspecific tests unless isoenzyme fractions are determined. Hepatic encephalopathy is related to elevated ammonia levels.

The nurse is assessing a 50-year-old woman admitted with a possible bowel obstruction. Which assessment finding would be expected in this patient? a. Tympany to abdominal percussion b. Aortic pulsation visible in epigastric region c. High-pitched sounds on abdominal auscultation d. Liver border palpable 1 cm below the right costal margin

Correct answer: c Rationale: The bowel sounds are more high pitched (rushes and tinkling) when the intestines are under tension, as in intestinal obstruction. Bowel sounds may also be diminished or absent with an intestinal obstruction. Normal findings include aortic pulsations on inspection and tympany with percussion, and the liver may be palpable 1 to 2 cm along the right costal margin.

Which statement best describes the etiology of obesity? a. Obesity primarily results from a genetic predisposition. b. Psychosocial factors can override the effects of genetics in the etiology of obesity. c. Obesity is the result of complex interactions between genetic and environmental factors. d. Genetic factors are more important than environmental factors in the etiology of obesity.

Correct answer: c Rationale: The cause of obesity involves significant genetic and biologic susceptibility factors that are highly influenced by environmental and psychosocial factors.

The nurse is caring for a patient admitted to the hospital for asthma who weighs 186 lb (84.5 kg). During dietary counseling, the patient asks the nurse how much protein he should ingest each day. How many grams of protein does the nurse recommend should be included in the diet based on the patient's current weight? a. 24 b. 41 c. 68 d. 93

Correct answer: c Rationale: The daily intake of protein should be between 0.8 and 1 g/kg of body weight. Thus this patient should take in between 68 and 84 g of protein per day in the diet.

A patient receives atropine, an anticholinergic drug, in preparation for surgery. The nurse expects this drug to affect the GI tract by doing what? a. Increasing gastric emptying b. Relaxing pyloric and ileocecal sphincters c. Decreasing secretions and peristaltic action d. Stimulation the nervous system of the GI tract

Correct answer: c Rationale: The parasympathetic nervous system stimulates activity of the gastrointestinal (GI) tract, increasing motility and secretions and relaxing sphincters to promote movement of contents. A drug that blocks this activity decreases secretions and peristalsis, slows gastric emptying, and contracts sphincters. The enteric nervous system of the GI tract is modulated by sympathetic and parasympathetic influence.

What may occur with failure of the sodium-potassium pump during severe protein depletion? a. Ascites b. Anemia c. Hyperkalemia d. Hypoalbuminemia

Correct answer: c Rationale: The sodium-potassium pump uses 20% to 50% of all calories ingested. When energy sources are decreased, the pump fails to function, sodium and water are left in the cell, and potassium remains in extracellular fluids. Hyperkalemia, as well as hyponatremia, can occur.

Priority Decision: When caring for a patient who has had most of the stomach surgically removed, what is important for the nurse to teach the patient? a. Extra iron will need to be taken to prevent anemia. b. Avoid foods with lactose to prevent bloating and diarrhea. c. Lifelong supplementation of cobalamin (vitamin B12) will be needed. d. Because of the absence of digestive enzymes, protein malnutrition is likely.

Correct answer: c Rationale: The stomach secretes intrinsic factor, necessary for cobalamin (vitamin B12) absorption in the intestine. When part or all of the stomach is removed, cobalamin must be supplemented for life. The other options will not be a problem.

The nurse is providing care for a 23-year-old woman who is a strict vegetarian. To prevent the consequences of iron deficiency, what should the nurse recommend? a. Brown rice and kidney beans b. Cauliflower and egg substitutes c. Soybeans and hot breakfast cereal d. Whole-grain bread and citrus fruits

Correct answer: c Rationale: Vegetarians are at a particular risk for iron deficiency, a problem that can be prevented by regularly consuming high-iron foods such as hot cereals and soybeans. The other foods listed are not classified as high sources of iron.

When the nurse is assessing the health perception-health maintenance pattern as related to GI function, an appropriate question to ask is a. "What is your usual bowel elimination pattern?" b. "What percentage of your income is spent on food?" c. "Have you traveled to a foreign country in the last year?" d. "Do you have diarrhea when you are under a lot of stress?"

Correct answer: c Rationale: When assessing gastrointestinal function in relation to the health perception-health management pattern, the nurse should ask the patient about recent foreign travel with possible exposure to hepatitis, parasitic infestation, or bacterial infection.

Which nursing actions are indicated for a liver biopsy (select all that apply)? a. Observe for white stools b. Monitor for rectal bleeding c. Monitor for internal bleeding d. Position to right side after test e. Ensure bowel preparation was done f. Check coagulation status before test

Correct answer: c, d, f Rationale: Because the liver is a vascular organ, vital signs are monitored to assess for internal bleeding. Prevention of bleeding is the reason for positioning on the right side for at least 2 hours and for splinting the puncture site. Again, because of the vasculature of the liver, coagulation status is checked before the biopsy is done. White stools occur with upper gastrointestinal (UGI) or barium swallow tests. No smoking is to be done after midnight before the study with an UGI. The bowel must be cleared before a lower GI or barium enema, a virtual colonoscopy, or a colonoscopy. Rectal bleeding may occur with a sigmoidoscopy or colonoscopy. A perforation may occur with an esophagogastroduodenoscopy (EGD), ERCP, or peritoneoscopy.

When the nurse identifies an individual at risk for malnutrition with nutritional screening, what is the next step for the nurse to take? a. Supply supplements between meals. b. Encourage eating meals with others. c. Have family bring in food from home. d. Complete a full nutritional assessment.

Correct answer: d Rationale: A full nutritional assessment includes history and physical examination and laboratory data. The nutritional assessment will need to be done to provide the basis for nutrition intervention. The interventions may include supplements if ordered, family bringing food from home, and socializing with meals.

Which patient is at highest risk for developing metabolic syndrome? a. A 62-year-old white man who has coronary artery disease with chronic stable angina b. A 54-year-old Hispanic woman who is sedentary and has nephrogenic diabetes insipidus c. A 27-year-old Asian American woman who has preeclampsia and gestational diabetes mellitus d. A 38-year-old Native American man who has diabetes mellitus and elevated hemoglobin A1C

Correct answer: d Rationale: African Americans, Hispanics, Native Americans, and Asians are at an increased risk for development of metabolic syndrome. Other risk factors include individuals who have diabetes that cannot maintain a normal glucose level, have hypertension, and secrete a large amount of insulin, or who have survived a heart attack and have hyperinsulinemia.

In the immediate postoperative period a nurse cares for a severely obese 72-year-old man who had surgery for repair of a lower leg fracture. Which assessment would be most important for the nurse to make? a. Cardiac rhythm b. Surgical dressing c. Postoperative pain d. Oxygen saturation

Correct answer: d Rationale: After surgery an older and/or severely obese patient should be closely monitored for oxygen desaturation. The body stores anesthetics in adipose tissue, placing patients with excess adipose tissue (e.g., obesity, older) at risk for resedation. As adipose cells release anesthetic back into the bloodstream, the patient may become sedated after surgery. This may depress the respiratory rate and result in a drop in oxygen saturation.

The nurse is performing a focused abdominal assessment of a patient who has been recently admitted. In order to palpate the patient's liver, where should the nurse palpate the patient's abdomen? a. Left lower quadrant b. Left upper quadrant c. Right lower quadrant d. Right upper quadrant

Correct answer: d Rationale: Although the left lobe of the liver is located in the left upper quadrant of the abdomen, the bulk of the liver is located in the right upper quadrant.

To evaluate the effect of nutritional interventions for a patient with protein-calorie malnutrition, what is the best indicator for the nurse to use? a. Height and weight b. Body mass index (BMI) c. Weight in relation to ideal body weight d. Mid-upper arm circumference and triceps skinfold

Correct answer: d Rationale: Anthropometric measurements, including mid-upper arm circumference and triceps skinfold measurements, are good indicators of lean body mass and skeletal protein reserves and are valuable in evaluating persons who may have been or are being treated for acute protein malnutrition. The other measurements do not specifically address muscle mass.

After eating, a patient with an inflamed gallbladder experiences pain caused by contraction of the gallbladder. What is the mechanism responsible for this action? a. Production of bile by the liver b. Production of secretin by the duodenum c. Release of gastrin from the stomach antrum d. Production of cholecystokinin by the duodenum

Correct answer: d Rationale: Cholecystokinin is secreted by the duodenal mucosa when fats and amino acids enter the duodenum and stimulate the gallbladder to release bile to emulsify the fats for digestion. The bile is produced by the liver but stored in the gallbladder. Secretin is responsible for stimulating pancreatic bicarbonate secretion and gastrin increases gastric motility and acid secretion.

What is the most common cause of secondary protein-calorie malnutrition in the United States? a. The unavailability of foods high in protein b. A lack of knowledge about nutritional needs c. A lack of money to purchase high-protien foods d. An alteration in ingestion, digestion, absorption, or metabolism

Correct answer: d Rationale: In the United States, where rote in intake is high and of good quality, protein-calorie malnutrition most commonly results from problems of the GI system. In developing countries, adequate food sources might not exist, the inhabitants may not be well educated about nutritional needs, and economic conditions can prevent purchase of balanced diets.

The obesity classification that is most often associated with cardiovascular health problems is a. primary obesity. b. secondary obesity. c. gynoid fat distribution. d. android fat distribution.

Correct answer: d Rationale: Individuals with fat located primarily in the abdominal area (i.e., whose body is apple-shaped) are at greater risk for obesity-related complications (e.g., heart disease) than are those whose fat is primarily located in the upper legs (i.e., whose body is pear-shaped). Individuals whose fat is distributed over the abdomen and upper body (i.e., neck, arms, and shoulders) are classified as having android obesity.

Which patient has the highest risk for poor nutritional balance related to decreased ingestion? a. Tuberculosis infection b. Malabsorption syndrome c. Draining decubitus ulcers d. Severe anorexia resulting from radiation therapy

Correct answer: d Rationale: Malnutrition that results form a deceased intake of food is most common in individuals with severe anorexia where there is a decreased desire to eat. Infections created a hypermetabolic state that increases nutritional demand, malabsorption causes loss of nutrients that are ingested, and draining decubitus ulcers are examples of disorders that cause both loss of protein and hypermetabolic states.

The nurse has completed initial instruction with a patient regarding a weight-loss program. Which patient comment indicates to the nurse that the teaching has been effective? a. "I will keep a diary of daily weight to illustrate my weight loss." b. "I plan to lose 4 lb a week until I have lost the 60 lb I want to lose." c. "I should not exercise more than what is required so I don't increase my appetite." d. "I plan to join a behavior modification group to help establish long-term behavior changes."

Correct answer: d Rationale: People who have undergone behavior therapy are more successful in maintaining weight losses over time because most programs deemphasize the diet, focus on how and when the person eats and education, and provide support from others. Weighing daily is not recommended and plateaus may not allow for consistent weight loss. A goal for weight loss must be set and 1 to 2 pounds a week is realistic. A more rapid loss often causes skin and underlying tissue to lose elasticity and become flabby folds of tissue. Exercising more often depresses appetite and exercise need not be limited.

The nurse is reviewing the laboratory test results for a 71-year-old patient with metastatic lung cancer. The patient was admitted with a diagnosis of malnutrition. The serum albumin level is 4.0 g/dL, and prealbumin is 10 mg/dL. What should this indicate to the nurse? a. The albumin level is normal, and therefore the patient does not have protein malnutrition. b. The albumin level is increased, which is a common finding in patients with cancer who have malnutrition. c. Both the serum albumin and prealbumin levels are reduced, consistent with the admitting diagnosis of malnutrition. d. Although the serum albumin level is normal, the prealbumin level more accurately reflects the patient's nutritional status.

Correct answer: d Rationale: Prealbumin has a half-life of 2 days and is a better indicator of recent or current nutritional status. Serum albumin has a half-life of approximately 20 to 22 days. The serum level may lag behind actual protein changes by more than 2 weeks and is therefore not a good indicator of acute changes in nutritional status.

A patient with anorexia nervosa shows signs of malnutrition. During initial referring, the nurse carefully assesses the patient for a. hyperkalemia. b. hypoglycemia. c. hypercalcemia. d. hypophosphatemia.

Correct answer: d Rationale: Refeeding syndrome is characterized by fluid retention, electrolyte imbalances (e.g., hypophosphatemia, hypokalemia, hypomagnesemia), and hyperglycemia. Conditions that predispose patients to refeeding syndrome include long-standing malnutrition states such as those induced by chronic alcoholism, vomiting and diarrhea, chemotherapy, and major surgery. Refeeding syndrome can occur any time a malnourished patient is started on aggressive nutritional support. Hypophosphatemia is the hallmark of refeeding syndrome, and it is associated with serious outcomes, including cardiac dysrhythmias, respiratory arrest, and neurologic disturbances (e.g., paresthesias).

The severely obese patient has elected to have the Roux-en-Y gastric bypass (RYGB) procedure. The nurse will know the patient understands the preoperative teaching when the patient makes which statement? a. "This surgery will preserve the function of my stomach." b. "This surgery will remove the fat cells from my abdomen." c. "This surgery can be modified whenever I need it to be changed." d. "This surgery decreases how much I can eat and how many calories I can absorb."

Correct answer: d Rationale: The RYGB decreases the size of the stomach to a gastric pouch and attaches it directly to the small intestine so food bypasses 90% of the stomach, the duodenum, and a small segment of the jejunum. The vertical sleeve gastrectomy removes 85% of the stomach, but preserves the function of the stomach. Lipectomy and liposuction remove fat tissue from the abdomen or other areas. Adjustable gastric banding can be modified or reversed at a later date.

The patient being admitted has been diagnosed with anorexia nervosa. What clinical manifestations should the nurse expect to see on admission assessment? a. Tan skin, blonde hair, and diarrhea b. Sensitivity to heat, fatigue, and polycythemia c. Dysmenorrhea, gastric ulcer pain, and hunger d. Hair loss; dry, yellowish skin; and constipation

Correct answer: d Rationale: The patient with anorexia nervosa, along with abnormal weight loss, is likely to have hair loss; dry, yellow skin; constipation; sensitivity to cold, and absent or irregular menstruation. Other signs of malnutrition are also noted during physical examination.

Priority Decision: An 18-year-old female patient with anorexia nervosa is admitted to the hospital for treatment. On admission she weighs 82 lb (37 kg) and is 5 ft. 3 in (134.6 cm). Her laboratory test results include the following: K+ 2.8 mEq/L (2.8 mmol/L), Hgb 8.9 g/dL (89 g/L), and BUN 64 mg/dL (22.8 mmol/L). In planning care for the patient, the nurse gives the highest priority to which of the following nursing diagnoses? a. Risk for injury related to dizziness and weakness resulting from anemia b. Imbalanced nutrition: less than body requirements related to inadequate food intake c. Risk for impaired urinary elimination related to elevated BUN resulting from renal failure d. Risk for decreased cardiac output (CO) related to dysrhythmias resulting from hypokalemia

Correct answer: d Rationale: The potential life-threatening cardiac complications related to the hypokalemia are the most important immediate considerations in the patient's care. The other nursing diagnoses are important for the patient's care but do not pose immediate risk that the hypokalemia does.

What is a postoperative nursing intervention for the obese patient who has undergone bariatric surgery? a. Irrigating and repositioning the nasogastric (NG) tube as needed b. Delaying ambulation until the patient has enough strength to support self c. Keeping the patient positioned on the side to facilitate respiratory function d. Providing adequate support to the incision during coughing, deep breathing, and turning

Correct answer: d Rationale: Turning, coughing, and deep breathing are essential to prevent postoperative complications. Protecting the incision from strain is important since wound dehiscence is a problem for obese patients. If a nasogastric (NG) tube that is present following gastric surgery for severe obesity becomes blocked or needs repositioning, the health care provider should be notified. Ambulation is usually started on the evening of surgery and addition help will be needed to support the patient. Respiratory function is promoted by keeping the head of the bed elevated at an angle of 35 to 40 degrees.

A patient who is unable to swallow because of progressive amyotrophic lateral sclerosis is prescribed enteral nutrition through a newly placed gastrostomy tube. Which task is appropriate for the nurse to delegate to unlicensed assistive personnel (UAP)? a. Irrigate the tube between feedings. b. Provide wound care at the gastrostomy site. c. Administer prescribed liquid medications through the tube. d.Position the patient with a 45-degree head of bed elevation.

Correct answer: d Rationale: Unlicensed assistive personnel (UAP) may position the patient receiving enteral feedings with the head of bed elevated. A licensed practical nurse/licensed vocational nurse (LPN/LVN) or an RN could perform the other activities.

A 68-year-old patient is in the office for a physical. She notes that she no longer has regular bowel movements. Which suggestion by the nurse would be most helpful to the patient? a. Take an additional laxative to stimulate defecation. b. Eat less acidic foods to enable the gastrointestinal system to increase peristalsis. c. Eat less food at each meal to prevent feces from backing up related to slowed peristalsis. d. Attempt defecation after breakfast because gastrocolic reflexes increase colon peristalsis at that time.

Correct answer: d Rationale: When food inters the stomach and duodenum, the gastrocolic and duodenocolic reflexes are initiated and are more active after the first daily meal. Additional laxatives or laxative abuse contribute to constipation in older adults. Decreasing food intake is not recommended, as many older adults have a decreased appetite. Fibre and fluids should be increased.

The ED nurse has inspected, auscultated, and palpated the abdomen with no obvious abnormalities, except pain. When the nurse palpates the abdomen for rebound tenderness, there is severe pain. The nurse should know that this could indicate what problem? a. Hepatic cirrhosis b. Hypersplenomegaly c. Gall bladder distention d. Peritoneal inflammation

Correct answer: d Rationale: When palpating for rebound tenderness, the problem area of the abdomen will produce pain and severe muscle spasm when there is peritoneal inflammation. Hepatic cirrhosis, hypersplenomegaly, and gall bladder distention do not manifest with rebound tenderness.

This bariatric surgical procedure involves creating a stoma and gastric pouch that is reversible, and no malabsorption occurs. What surgical procedure is this? a. Vertical gastric banding b. Biliopancreatic diversion c. Roux-en-Y gastric bypass d. Adjustable gastric banding

Correct answer: d Rationale: With adjustable gastric banding (AGB), the stomach size is limited by an inflatable band placed around the fundus of the stomach. The band is connected to a subcutaneous port and can be inflated or deflated to change the stoma size to meet the patient's needs as weight is lost. The procedure is performed laparoscopically and, if necessary, can be modified or reversed after the initial procedure.

Priority Decision: The nurse is teaching a moderately obese woman interventions for the management of obesity. Initially, which strategies will support restricting dietary intake to below energy requirements (select all that apply)? a. Limit alcohol b. Rest when fatigued c. Determine portion sizes d. 1800- to 2200-calorie diet e. Attend Overeaters Anonymous

Correct answers: a, c Rationale: To restrict dietary intake so that it is below energy requirements, the moderately obese woman should limit or avoid alcohol intake because it increases caloric intake and has low nutritional value. Portion sizes have increased over the years and are larger than they should be. Teach the patient to determine portion sizes by weight or learn equivalencies such as that a serving of fruit is the size of a baseball. A progressive exercise program will increase energy requirements and a diet with an initial 800- to 1200-calorie limit would decrease calorie intake. Overeaters Anonymous would not restrict dietary intake below energy requirements, although it may offer support for the patient.

The family of a patient newly diagnosed with hepatitis A asks the nurse what they can do to prevent becoming ill themselves. Which response by the nurse is most appropriate? A. "The hepatitis vaccine will provide immunity from this exposure and future exposures." B. "I am afraid there is nothing you can do since the patient was infectious before admission." C. "You will need to be tested first to make sure you don't have the virus before we can treat you." D. "An injection of immunoglobulin will need to be given to prevent or minimize the effects from this exposure."

D. "An injection of immunoglobulin will need to be given to prevent or minimize the effects from this exposure." Immunoglobulin provides temporary (1-2 months) passive immunity and is effective for preventing hepatitis A if given within 2 weeks after exposure. It may not prevent infection in all persons, but it will at least modify the illness to a subclinical infection. The hepatitis vaccine is only used for preexposure prophylaxis.

When teaching the patient with acute hepatitis C (HCV), the patient demonstrates understanding when the patient makes which statement? A. "I will use care when kissing my wife to prevent giving it to her." B. "I will need to take adofevir (Hepsera) to prevent chronic HCV." C. "Now that I have had HCV, I will have immunity and not get it again." D. "I will need to be checked for chronic HCV and other liver problems."

D. "I will need to be checked for chronic HCV and other liver problems." The majority of patients who acquire HCV usually develop chronic infection, which may lead to cirrhosis or liver cancer. HCV is not transmitted via saliva, but percutaneously and via high-risk sexual activity exposure. The treatment for acute viral hepatitis focuses on resting the body and adequate nutrition for liver regeneration. Adofevir (Hepsera) is taken for severe hepatitis B (HBV) with liver failure. Chronic HCV is treated with pegylated interferon with ribavirin. Immunity with HCV does not occur as it does with HAV and HBV, so the patient may be reinfected with another type of HCV.

he patient has had type 1 diabetes mellitus for 25 years and is now reporting fatigue, edema, and an irregular heartbeat. On assessment, the nurse finds that the patient has newly developed hypertension and difficulty with blood glucose control. The nurse should know that which diagnostic study will be most indicative of chronic kidney disease (CKD) in this patient? A. Serum creatinine B. Serum potassium C. Microalbuminuria D. Calculated glomerular filtration rate (GFR)

D. Calculated glomerular filtration rate (GFR) The best study to determine kidney function or chronic kidney disease (CKD) that would be expected in the patient with diabetes is the calculated GFR that is obtained from the patient's age, gender, race, and serum creatinine. It would need to be abnormal for 3 months to establish a diagnosis of CKD. A creatinine clearance test done with a blood sample and a 24-hour urine collection is also important. Serum creatinine is not the best test for CKD because the level varies with different patients. Serum potassium levels could explain why the patient has an irregular heartbeat. The finding of microalbuminuria can alert the patient with diabetes about potential renal involvement and potentially failing kidneys. However, urine albumin levels are not used for diagnosis of CKD.

The patient with suspected pancreatic cancer is having many diagnostic studies done. Which one can be used to establish the diagnosis of pancreatic adenocarcinoma and for monitoring the response to treatment? A. Spiral CT scan B. A PET/CT scan C. Abdominal ultrasound D. Cancer-associated antigen 19-9

D. Cancer-associated antigen 19-9 The cancer-associated antigen 19-9 (CA 19-9) is the tumor marker used for the diagnosis of pancreatic adenocarcinoma and for monitoring the response to treatment. Although a spiral CT scan may be the initial study done and provides information on metastasis and vascular involvement, this test and the PET/CT scan or abdominal ultrasound do not provide additional information.

The health care provider orders lactulose for a patient with hepatic encephalopathy. The nurse will monitor for effectiveness of this medication for this patient by assessing what? A. Relief of constipation B. Relief of abdominal pain C. Decreased liver enzymes D. Decreased ammonia levels

D. Decreased ammonia levels. Hepatic encephalopathy is a complication of liver disease and is associated with elevated serum ammonia levels. Lactulose traps ammonia in the intestinal tract. Its laxative effect then expels the ammonia from the colon, resulting in decreased serum ammonia levels and correction of hepatic encephalopathy

The patient was diagnosed with prerenal AKI. The nurse should know that what is most likely the cause of the patient's diagnosis? A. IV tobramycin (Nebcin) B. Incompatible blood transfusion C. Poststreptococcal glomerulonephritis D. Dissecting abdominal aortic aneurysm

D. Dissecting abdominal aortic aneurysm A dissecting abdominal aortic aneurysm is a prerenal cause of AKI because it can decrease renal artery perfusion and therefore the glomerular filtrate rate. Aminoglycoside antibiotic administration, a hemolytic blood transfusion reaction, and poststretpcoccal glomerulonephritis are intrarenal causes of AKI.

The patient in the intensive care unit is receiving gentamicin for pneumonia from Pseudomonas. What assessment results should the nurse report to the health care provider? A. Decreased weight B. Increased appetite C. Increased urinary output D. Elevated creatinine level

D. Elevated creatinine level Gentamicin can be toxic to the kidneys and the auditory system. The elevated creatinine level must be reported to the physician as it probably indicates renal damage. Other factors that may occur with renal damage would include increased weight and decreased urinary output. Many medications have side effects of anorexia.

A patient who has hepatitis B surface antigen (HBsAg) in the serum is being discharged with pain medication after knee surgery. Which medication order should the nurse question because it is most likely to cause hepatic complications? A. Tramadol (Ultram) B. Hydromorphone (Dilaudid) C. Oxycodone with aspirin (Percodan) D. Hydrocodone with acetaminophen (Vicodin)

D. Hydrocodone with acetaminophen (Vicodin) The analgesic with acetaminophen should be questioned because this patient is a chronic carrier of hepatitis B and is likely to have impaired liver function. Acetaminophen is not suitable for this patient because it is converted to a toxic metabolite in the liver after absorption, increasing the risk of hepatocellular damage.

When planning care for a patient with cirrhosis, the nurse will give highest priority to which nursing diagnosis? A. Impaired skin integrity related to edema, ascites, and pruritus B. Imbalanced nutrition: less than body requirements related to anorexia C. Excess fluid volume related to portal hypertension and hyperaldosteronism D. Ineffective breathing pattern related to pressure on diaphragm and reduced lung volume

D. Ineffective breathing pattern related to pressure on diaphragm and reduced lung volume Although all of these nursing diagnoses are appropriate and important in the care of a patient with cirrhosis, airway and breathing are always the highest priorities.

When caring for a patient during the oliguric phase of acute kidney injury (AKI), what is an appropriate nursing intervention? A. Weigh patient three times weekly. B. Increase dietary sodium and potassium. C. Provide a low-protein, high-carbohydrate diet. D. Restrict fluids according to previous daily loss.

D. Restrict fluids according to previous daily loss. Patients in the oliguric phase of acute kidney injury will have fluid volume excess with potassium and sodium retention. Therefore they will need to have dietary sodium, potassium, and fluids restricted. Daily fluid intake is based on the previous 24-hour fluid loss (measured output plus 600 ml for insensible loss). The diet also needs to provide adequate, not low, protein intake to prevent catabolism. The patient should also be weighed daily, not just three times each week.

The patient with a history of lung cancer and hepatitis C has developed liver failure and is considering liver transplantation. After the comprehensive evaluation, the nurse knows that which factor discovered may be a contraindication for liver transplantation? A. Has completed a college education B. Has been able to stop smoking cigarettes C. Has well-controlled type 1 diabetes mellitus D. The chest x-ray showed another lung cancer lesion.

D. The chest x-ray showed another lung cancer lesion. Contraindications for liver transplant include severe extrahepatic disease, advanced hepatocellular carcinoma or other cancer, ongoing drug and/or alcohol abuse, and the inability to comprehend or comply with the rigorous post-transplant course.

Which urinalysis result should the nurse recognize as an abnormal finding? A. pH 6.0 B. Amber yellow color C. Specific gravity 1.025 D. White blood cells (WBCs) 9/hpf

D. White blood cells (WBCs) 9/hpf Normal WBC levels in urine are below 5/hpf, with levels exceeding this indicative of inflammation or urinary tract infection. A urine pH of 6.0 is average; amber yellow is normal coloration, and the reference ranges for specific gravity are 1.003 to 1.030.

A client with a history of chronic kidney disease is hospitalized. The nurse assesses the client for signs of related kidney insufficiency, which include:

Edema and pruritus

A nurse plans to teach the signs of rejection to a client who just had a transplanted kidney. What sign of rejection should the nurse include?

Elevated blood pressure

A client with acute kidney failure is to receive peritoneal dialysis and asks why the procedure is necessary. The nurse's best response is, "It:

Helps perform some of the work usually done by the kidneys."

A nurse is caring for a client receiving hemodialysis for chronic kidney disease. The nurse should monitor the client for what complication?

Hepatitis B

Prednisone (Meticorten), an adrenal steroid, is prescribed for a client with an exacerbation of colitis. When administering the first dose of the medication, the nurse should inform the client that the medication:

Is not curative but does cause a suppression of the inflammatory process

A nurse is caring for a client who had a nephrectomy because of cancer of the kidney. Which factor will influence the client's ability to deep breathe and cough postoperatively?

Location of the surgical incision

A health care provider prescribes furosemide (Lasix) for a client with hypervolemia. The nurse recalls that furosemide exerts its effects in what part of the renal system?

Loop of Henle

A nurse is caring for a client with a diagnosis of renal calculi secondary to hyperparathyroidism. Which type of diet should the nurse explore with the client when providing discharge information?

Low calcium

A nurse is monitoring a client with renal failure for signs of fluid excess. Which finding does the nurse identify as inconsistent with fluid excess?

Orthostatic hypotension

A nurse is caring for a 6-year-old child with a diagnosis of glomerulonephritis. The child's urine output decreases to less than100 mL/24 hr, the creatinine clearance is 60 mL/min, and there is an irregular apical pulse. A diagnosis of acute renal failure is made. Blood is drawn for testing. Which serum level requires immediate intervention?

Potassium 6.1 mEq/L

The nurse performs a detailed assessment of the abdomen of a patient with a possible bowel obstruction, knowing that manifestations of a obstruction in the large intestine are (select all that apply): a. persistent abdominal pain b. marked abdominal distention c. diarrhea that is loose or liquid d. colicky, severe, intermittent pain e. profuse committing that relieves abdominal pain

a & b Rationale: With lower intestinal obstructions, abdominal distention is markedly increased and pain is persistent. Onset of a large intestine obstruction is gradual, vomiting is rare, and there is usually absolute constipation, not diarrhea.

Which of the following client responses shows a correct understanding of continuous ambulatory peritoneal dialysis (CAPD)? a) I am expected to perform the procedure at home b) the procedure lasts for one hour c) I have to sit and raise my legs during the procedure d) I have to go to the hospital for this procedure

a) I am expected to perform the procedure at home

The nurse instructs a client with renal failure who is receiving hemodialysis about dietary modifications. The nurse determines that the client understands these dietary modifications if the client selects which items from the dietary menu? a) cream of wheat, blueberries, coffee b) sausage and eggs, banana, orange juice c) bacon, cantaloupe melon, tomato juice d) cured pork, strawberries, orange juice

a) cream of wheat, blueberries, coffee - the diet for a client with renal failure who is receiving hemodialysis should include controlled amounts of sodium, phosphorus, calcium, potassium, and fluids. Option B, C, and D are high in sodium, phosphorus, and potassium.

A client with renal failure is receiving epoetin alfa (Epogen) to support erythropoiesis. The nurse questions the client about compliance with taking which of the following medications that supports red blood cell (RBC) production? a) iron supplement b) zinc supplement c) calcium supplement d) magnesium supplement

a) iron supplement

Which of the following should be considered in the diet of the client with end-stage-renal-disease (ESRD)? a) limit fluid intake during anuric phase b) limit phosphorus and vitamin D-rich food c) limit calcium-rich food d) limit carbohydrates

a) limit fluid intake during anuric phase during ESRD, fluid intake of the client should be limited during anuric phase to prevent fluid overload. Fluid overload increases renal workload, pulmonary edema, and congestive heart failure.

Which of the following should the nurse include in the nursing care plan of the client who is diagnosed to have renal failure, whose BUN is 32 mg/dl, serum creatinine is 4 mg/dl, hematocrit is 38%. He is complaining of fatigue and edema. a) low protein diet and fluid restriction b) high protein diet and fluid restriction c) low protein diet and increase in fiber d) high protein diet and potassium restriction

a) low protein diet and fluid restriction

The client with chronic renal failure is on chronic hemodialysis. Which of the following indicate improvement of the client's condition due to hemodialysis? Select all that apply a) the client's BP is 130/90 b) the client's serum potassium is 4.8 mEq/L c) the client's hemoglobin level is 10 g/dL d) the client's serum calcium is 7.7 mg/dL e) the client's serum sodium is 140 mEg/L f) the client's serum magnesium is 4 mEq/L g) the client's weight has increased from 60 kg to 63 kg

a) the client's BP is 130/90 b) the client's serum potassium is 4.8 mEq/L e) the client's serum sodium is 140 mEg/L

When a 35-year-old female patient is admitted to the emergency department with acute abdominal pain, which possible diagnosis should you consider that may be the cause of her pain (select all that apply)? a. Gastroenteritis b. Ectopic pregnancy c. GI bleeding d. Irritable bowel syndrome e. Inflammatory bowel disease

a, b, c, d & e Rationale: All these conditions could cause acute abdominal pain.

Prevention of AKI is important because of the high mortality rate. Which patients are at increased risk for AKI (select all that apply)? a. An 86-year-old woman scheduled for a cardiac catheterization b. A 48-year-old man with multiple injuries from a motor vehicle accident c. A 32-year-old woman following a C-section delivery for abruptio placentae d. A 64-year-old woman with chronic heart failure admitted with bloody stools e. A 58-year-old man with prostate cancer undergoing preoperative workup for prostatectomy

a, b, c, d, e. High-risk patients include those exposed to nephrotoxic agents and advanced age (a), massive trauma (b), prolonged hypovolemia or hypotension (possibly b and c), obstetric complications (c), cardiac failure (d), preexisting chronic kidney disease, extensive burns, or sepsis. Patients with prostate cancer may have obstruction of the outflow tract, which increases risk of postrenal AKI (e).

Which drugs will be used to treat the patient with CKD for mineral and bone disorder (select all that apply)? a. Cinacalcet (Sensipar) b. Sevelamer (Renagel) c. IV glucose and insulin d. Calcium acetate (PhosLo) e. IV 10% calcium gluconate

a, b, d. Cinacalcet (Sensipar), a calcimimetic agent to control secondary hyperparathyroidism; sevelamer (Renagel), a noncalcium phosphate binder; and calcium acetate (PhosLo), a calcium-based phosphate binder are used to treat mineral and bone disorder in CKD. IV glucose and insulin and IV 10% calcium gluconate along with sodium polystyrene sulfonate (Kayexalate) are used to treat the hyperkalemia of CKD.

The patient with CKD is receiving dialysis, and the nurse observes excoriations on the patient's skin. What pathophysiologic changes in CKD can contribute to this finding (select all that apply)? a. Dry skin b. Sensory neuropathy c. Vascular calcifications d. Calcium-phosphate skin deposits e. Uremic crystallization from high BUN

a, b, d. Pruritus is common in patients receiving dialysis. It causes scratching from dry skin, sensory neuropathy, and calcium-phosphate deposition in the skin. Vascular calcifications contribute to cardiovascular disease, not to itching skin. Uremic frost rarely occurs without BUN levels greater than 200 mg/dL, which should not occur in a patient on dialysis; urea crystallizes on the skin and also causes pruritis.

For the patient hospitalized with inflammatory bowel disease (IBD), which treatments would be used to rest the bowel (select all that apply)? a. NPO d. Sedatives b. IV fluids e. Nasogastric suction c. Bed rest f. Parenteral nutrition

a, b, e, f. With an acute exacerbation of inflammatory bowel disease (IBD), to rest the bowel the patient will be NPO, receive IV fluids and parenteral nutrition, and have nasogastric suction. Sedatives would be used to alleviate stress. Enteral nutrition will be used as soon as possible

A 20-year old patient with a history of Crohn's disease comes to the clinic with persistent diarrhea. What are characteristics of Crohn's disease (select all that apply)? a. Weight loss d. Toxic megacolon b. Rectal bleeding e. Has segmented distribution c. Abdominal pain f. Involves the entire thickness of the bowel wall

a, c, e, f. Crohn's disease may have severe weight loss, segmented distribution through the entire wall of the bowel, and crampy abdominal pain. Rectal bleeding and toxic megacolon are more often seen with ulcerative colitis.

The patient calls the clinic and describes a bump at the site of a previous incision that disappears when he lies down. The nurse suspects that this is which type of hernia (select all that apply)? a. Ventral d. Reducible b. Inguinal e. Incarcerated c. Femoral f. Strangulated

a, d. The ventral or incisional hernia is due to a weakness of the abdominal wall at the site of a previous incision. It is reducible when it returns to the abdominal cavity. Inguinal hernias are at the weak area of the abdominal wall where the spermatic cord in men or the round ligament in women emerges. A femoral hernia is a protrusion through the femoral ring into the femoral canal. Incarcerated hernias do not reduce.

When evaluating the patient's understanding about the care of the ileostomy, what statement by the patient indicates the patient needs more teaching? a. "I will be able to regulate when I have stools." b. "I will be able to wear the pouch until it leaks." c. "Dried fruit and popcorn must be chewed very well." d. "The drainage from my stoma can damage my skin."

a. "I will be able to regulate when I have stools." The ileostomy is in the ileum and drains liquid stool frequently, unlike the colostomy which has more formed stool the further distal the ostomy is in the colon. The ileostomy pouch is usually worn 4-7 days or until it leaks. It must be changed immediately if it leaks because the drainage is very irritating to the skin. To avoid obstruction, popcorn, dried fruit, coconut, mushrooms, olives, stringy vegetables, food with skin, and meats with casings must be chewed extremely well before swallowing because of the narrow diameter of the ileostomy lumen.

The nurse is preparing to insert a nasogastric (NG) tube into a 68-year-old female patient who is nauseated and vomiting. She has an abdominal mass and suspected small intestinal obstruction. The patient asks the nurse why this procedure is necessary. What response by the nurse is most appropriate? a. "The tube will help to drain the stomach contents and prevent further vomiting." b. "The tube will push past the area that is blocked and thus help to stop the vomiting." c. "The tube is just a standard procedure before many types of surgery to the abdomen." d. "The tube will let us measure your stomach contents so that we can plan what type of IV fluid replacement would be best."

a. "The tube will help to drain the stomach contents and prevent further vomiting." The NG tube is used to decompress the stomach by draining stomach contents and thereby prevent further vomiting. The NG tube will not push past the blocked area. Potential surgery is not currently indicated. The location of the obstruction will determine the type of fluid to use, not measure the amount of stomach contents.

When obtaining a nursing history from the patient with colorectal cancer, the nurse should specifically ask the patient about a. dietary intake. b. sports involvement. c. environmental exposure to carcinogens. d. long-term use of nonsteroidal antiinflammatory drugs (NSAIDs).

a. A diet high in red meat and low fruit and vegetable intake is associated with development of colorectal cancer (CRC), as are alcohol intake and smoking. Family and personal history of CRC also increases the risk. Other environmental agents are not known to be related to colorectal cancer. Long-term use of nonsteroidal antiinflammatory drugs (NSAIDs) is associated with reduced CRC risk.

The nurse plans teaching for the patient with a colostomy but the patient refuses to look at the nurse or the stoma, stating, "I just can't see myself with this thing." What is the best nursing intervention for this patient? a. Encourage the patient to share concerns and ask questions. b. Refer the patient to a chaplain to help cope with this situation. c. Explain that there is nothing the patient can do about it and must take care of it. d. Tell the patient that learning about it will prevent stool leaking and the sounds of flatus.

a. Encouraging the patient to share concerns and ask questions will help the patient to begin to adapt to living with the colostomy. The other options do not support the patient and do not portray the nurse's focus on helping the patient or treating the patient as an individual.

Which method is preferred for immediate treatment of an acute episode of constipation? a. An enema c. Stool softeners b. Increased fluid d. Bulk-forming medication

a. Enemas are fast acting and beneficial in the immediate treatment of acute constipation but should be limited in their use. Bulk-forming medication stimulates peristalsis but takes 24 hours to act. Stool softeners have a prolonged action, taking up to 72 hours for an effect, and fluids can help to decrease the incidence of constipation

A stroke patient who primarily uses a wheelchair for mobility has diarrhea with fecal incontinence. What should the nurse assess first? a. Fecal impaction b. Perineal hygiene c. Dietary fiber intake d. Antidiarrheal agent use

a. Fecal impaction Patients with limited mobility are at risk for fecal impactions due to constipation that may lead to liquid stool leaking around the hardened impacted feces, so assessing for fecal impaction is the priority. Perineal hygiene can be assessed at the same time. Assessing the dietary fiber and fluid intake and antidiarrheal agent use will be assessed and considered next.

Priority Decision: During the immediate postoperative care of a recipient of a kidney transplant, what should the nurse expect to do? a. Regulate fluid intake hourly based on urine output. b. Monitor urine-tinged drainage on abdominal dressing. c. Medicate the patient frequently for incisional flank pain. d. Remove the urinary catheter to evaluate the ureteral implant.

a. Fluid and electrolyte balance is critical in the transplant recipient patient, especially because diuresis often begins soon after surgery. Fluid replacement is adjusted hourly based on kidney function and urine output. Urine-tinged drainage on the abdominal dressing may indicate leakage from the ureter implanted into the bladder and the health care provider should be notified. The donor patient may have a flank or laparoscopic incision(s) where the kidney was removed. The recipient has an abdominal incision where the kidney was placed in the iliac fossa. The urinary catheter is usually used for 2 to 3 days to monitor urine output and kidney function.

A patient received a kidney transplant last month. Because of the effects of immunosuppressive drugs and CKD, what complication of transplantation should the nurse be assessing the patient for to decrease the risk of mortality? a. Infection b. Rejection c. Malignancy d. Cardiovascular disease

a. Infection is a significant cause of morbidity and mortality after transplantation because the surgery, the immunosuppressive drugs, and the effects of CKD all suppress the body's normal defense mechanisms, thus increasing the risk of infection. The nurse must assess the patient as well as use aseptic technique to prevent infections. Rejection may occur but for other reasons. Malignancy occurrence increases later due to immunosuppressive therapy. Cardiovascular disease is the leading cause of death after renal transplantation but this would not be expected to cause death within the first month after transplantation.

Priority Decision: A patient with ulcerative colitis has a total proctocolectomy with formation of a terminal ileum stoma. What is the most important nursing intervention for this patient postoperatively? a. Measure the ileostomy output to determine the status of the patient's fluid balance. b. Change the ileostomy appliance every 3 to 4 hours to prevent leakage of drainage onto the skin. c. Emphasize that the ostomy is temporary and the ileum will be reconnected when the large bowel heals. d. Teach the patient about the high-fiber, low-carbohydrate diet required to maintain normal ileostomy drainage.

a. Initial output from a newly formed ileostomy may be as high as 1500 to 2000 mL daily and intake and output must be accurately monitored for fluid and electrolyte imbalance. Ileostomy bags may need to be emptied every 3 to 4 hours but the appliance should not be changed for several days unless there is leakage onto the skin. A terminal ileum stoma is permanent and the entire colon has been removed. A return to a normal, presurgical diet is the goal for the patient with an ileostomy, with restrictions based only on the patient's individual tolerances.

Priority Decision: A patient with a gunshot wound to the abdomen complains of increasing abdominal pain several hours after surgery to repair the bowel. What action should the nurse take first? a. Take the patient's vital signs. b. Notify the health care provider. c. Position the patient with the knees flexed. d. Determine the patient's IV intake since the end of surgery.

a. It is likely that the patient could be developing a peritonitis, which could be life-threatening, and assessment of vital signs for hypovolemic shock should be done to report to the health care provider. If an IV line is not in place, it should be inserted and pain may be eased by flexing the knees.

The nurse is conducting discharge teaching for a patient with metastatic lung cancer who was admitted with a bowel impaction. Which instructions would be most helpful to prevent further episodes of constipation? a. Maintain a high intake of fluid and fiber in the diet. b. Reduce intake of medications causing constipation. c. Eat several small meals per day to maintain bowel motility. d. Sit upright during meals to increase bowel motility by gravity.

a. Maintain a high intake of fluid and fiber in the diet. Increased fluid intake and a high-fiber diet reduce the incidence of constipation caused by immobility, medications, and other factors. Fluid and fiber provide bulk that in turn increases peristalsis and bowel motility. Analgesics taken for lung cancer probably cannot be reduced. Other medications may decrease constipation, but it is best to avoid laxatives. Eating several small meals per day and position do not facilitate bowel motility. Defecation is easiest when the person sits on the commode with the knees higher than the hips.

A patient with inflammatory bowel disease has a nursing diagnosis of imbalanced nutrition: less than body requirements related to decreased nutritional intake and decreased intestinal absorption. Which assessment data support this nursing diagnosis? a. Pallor and hair loss c. Anorectal excoriation and pain b. Frequent diarrhea stools d. Hypotension and urine output below 30 mL/hr

a. Signs of malnutrition include pallor from anemia, hair loss, bleeding, cracked gingivae, and muscle weakness, which support a nursing diagnosis that identifies impaired nutrition. Diarrhea may contribute to malnutrition but is not a defining characteristic. Anorectal excoriation and pain relate to problems with skin integrity. Hypotension relates to problems with fluid deficit.

Priority Decision: A postoperative patient has a nursing diagnosis of pain related to effects of medication and decreased GI motility as evidenced by abdominal pain and distention and inability to pass flatus. Which nursing intervention is most appropriate for this patient? a. Ambulate the patient more frequently. b. Assess the abdomen for bowel sounds. c. Place the patient in high Fowler's position. d. Withhold opioids because they decrease bowel motility.

a. The abdominal pain and distention that occur from the decreased motility of the bowel should be treated with increased ambulation and frequent position changes to increase peristalsis. If the pain is severe, cholinergic drugs, rectal tubes, or application of heat to the abdomen may be prescribed. Assessment of bowel sounds is not an intervention to relieve the pain and a high Fowler's position is not indicated. Opioids may still be necessary for pain control and motility can be increased by other means.

82-year-old man is admitted with an acute attack of diverticulitis. What should the nurse include in his care? a. Monitor for signs of peritonitis. b. Treat with daily medicated enemas. c. Prepare for surgery to resect the involved colon. d. Provide a heating pad to apply to the left lower quadrant.

a. The inflammation and infection of diverticula cause small perforations with spread of the inflammation to the surrounding area in the intestines. Abscesses may form or complete perforation with peritonitis may occur. Systemic antibiotic therapy is often used but medicated enemas would increase intestinal motility and increase the possibility of perforation, as would the application of heat. Surgery is only necessary to drain abscesses or to resect an obstructing inflammatory mass.

A man with end-stage kidney disease is scheduled for hemodialysis following healing of an arteriovenous fistula (AVF). What should the nurse explain to him that will occur during dialysis? a. He will be able to visit, read, sleep, or watch TV while reclining in a chair. b. He will be placed on a cardiac monitor to detect any adverse effects that might occur. c. The dialyzer will remove and hold part of his blood for 20 to 30 minutes to remove the waste products. d. A large catheter with two lumens will be inserted into the fistula to send blood to and return it from the dialyzer.

a. While patients are undergoing hemodialysis, they can perform quiet activities that do not require the limb that has the vascular access. Blood pressure is monitored frequently and the dialyzer monitors dialysis function but cardiac monitoring is not usually indicated. The hemodialysis machine continuously circulates both the blood and the dialysate past the semipermeable membrane in the machine. Graft and fistula access involve the insertion of two needles into the site: one to remove blood from and the other to return blood to the dialyzer.

Two days following a colectomy for an abdominal mass, a patient reports gas pains and abdominal distention. The nurse plans care for the patient based on the knowledge that the symptoms are occurring as a result of a. impaired peristalsis. b. irritation of the bowel. c. nasogastric suctioning. d. inflammation of the incision site.

a. impaired peristalsis. Until peristalsis returns to normal following anesthesia, the patient may experience slowed gastrointestinal motility leading to gas pains and abdominal distention. Irritation of the bowel, nasogastric suctioning, and inflammation of the surgical site do not cause gas pains or abdominal distention.

The appropriate collaborative therapy for the patient with acute diarrhea caused by a viral infection is to: a. increase fluid intake b. administer an antibiotic c. administer antimotality drugs d. quarantine the patient to prevent spread of the virus

a. increase fluid intake Rationale: Acute diarrhea resulting from infectious causes (e.g., virus) is usually self-limiting. The major concerns are transmission prevention, fluid and electrolyte replacement, and resolution of the diarrhea. Antidiarrheal agents are contraindicated in the treatment of infectious diarrhea because they potentially prolong exposure to the infectious organism. Antibiotics are rarely used to treat acute diarrhea. To prevent transmission of diarrhea caused by a virus, hands should be washed before and after contact with the patient and when body fluids of any kind are handled. Vomitus and stool should be flushed down the toilet, and contaminated clothing should be washed immediately with soap and hot water.

The nurse determines that the goals of dietary teaching have been met when the patient with celiac disease selects from the menu: a. scrambled eggs and sausage b. buckwheat pancakes with syrup c. oatmeal, skim milk, and orange juice d. yogurt, strawberries, and rye toast with butter

a. scrambled eggs and sausage Rationale: Celiac disease is treated with lifelong avoidance of dietary gluten. Wheat, barley, oats, and rye products must be avoided. Although pure oats do not contain gluten, oat products can become contaminated with wheat, rye, and barley during the milling process. Gluten is also found in some medications and in many food additives, preservatives, and stabilizers.

Three year old Carlo has been admitted to the pediatric unit with a tentative diagnosis of nephrotic syndrome Prednisone is prescribed for Carlo. The nurse evaluate its effectiveness by a) checking his BP every 4 hours b) checking his urine for protein c) weighing him each morning before breakfast d) observing him for behavioral changes

b) checking his urine for protein Monitor side effect of prolonged steroid therapy Hyperglycemia - test urine monitor growth of child by checking height because steroid has growth suppressing effect by preventing calcium deposition in the bones Gastric Irritation - give milk or meals, test for occult blood, administer with antacids Avoid exposure to infection because child is immunosuppressed

The client with acute renal failure has a serum potassium of 6.0 mEq/L. The nurse would plan which of the following as a priority action? a) check the sodium level b) place the client on a cardiac monitor c) encourage increased vegetables in the diet d) allow an extra 500 ml of fluid intake to dilute the electrolyte concentration

b) place the client on a cardiac monitor

What is the primary way that a nurse will evaluate the patency of an AVF? a. Palpate for pulses distal to the graft site. b. Auscultate for the presence of a bruit at the site. c. Evaluate the color and temperature of the extremity. d. Assess for the presence of numbness and tingling distal to the site.

b. A patent arteriovenous fistula (AVF) creates turbulent blood flow that can be assessed by listening for a bruit or palpated for a thrill as the blood passes through the graft. Assessment of neurovascular status in the extremity distal to the graft site is important to determine that the graft does not impair circulation to the extremity but the neurovascular status does not indicate whether the graft is open.

In a patient with AKI, which laboratory urinalysis result indicates tubular damage? a. Hematuria b. Specific gravity fixed at 1.010 c. Urine sodium of 12 mEq/L (12 mmol/L) d. Osmolality of 1000 mOsm/kg (1000 mmol/kg)

b. A urine specific gravity that is consistently 1.010 and a urine osmolality of about 300 mOsm/kg is the same specific gravity and osmolality as plasma. This indicates that tubules are damaged and unable to concentrate urine. Hematuria is more common with postrenal damage. Tubular damage is associated with a high sodium concentration (greater than 40 mEq/L).

The patient asks the nurse to explain what the physician meant when he said the patient had an anorectal abscess. Which description should the nurse use to explain this to the patient? a. Ulcer in anal wall c. Sacrococcygeal hairy tract b. Collection of perianal pus d. Tunnel leading from the anus or rectum

b. An anorectal abscess is a collection of perianal pus. An ulcer in the anal wall is an anal fissure. Sacrococcygeal hairy tract describes a pilonidal sinus. A tunnel leading from the anus or rectum is an anorectal fistula.

In which type of dialysis does the patient dialyze during sleep and leave the fluid in the abdomen during the day? a. Long nocturnal hemodialysis b. Automated peritoneal dialysis (APD) c. Continuous venovenous hemofiltration (CVVH) d. Continuous ambulatory peritoneal dialysis (CAPD)

b. Automated peritoneal dialysis (APD) is the type of dialysis in which the patient dialyzes during sleep and leaves the fluid in the abdomen during the day. Long nocturnal hemodialysis occurs while the patient is sleeping and is done up to six times per week. Continuous venovenous hemofiltration (CVVH) is a type of continuous renal replacement therapy used to treat AKI. Continuous ambulatory peritoneal dialysis (CAPD) is dialysis that is done with exchanges of 1.5 to 3 L of dialysate at least four times daily.

Priority Decision: When caring for a patient with irritable bowel syndrome (IBS), what is most important for the nurse to do? a. Recognize that IBS is a psychogenic illness that cannot be definitively diagnosed. b. Develop a trusting relationship with the patient to provide support and symptomatic care. c. Teach the patient that a diet high in fiber will relieve the symptoms of both diarrhea and constipation. d. Inform the patient that new medications for IBS are available and effective for treatment of IBS manifested by either diarrhea or constipation.

b. Because there is no definitive treatment for irritable bowel syndrome (IBS) and patients become frustrated and discouraged with uncontrolled symptoms, it is important to develop a trusting relationship that will support the patient as different treatments are implemented and evaluated. Diagnosis of IBS can be established by Rome criteria and by elimination of other problems. Although IBS can be precipitated and aggravated by stress and emotions, it is not a psychogenic illness. High-fiber diets may help but they might also increase the bloating and gas pains of IBS. Medications are available but usually used as a last resort because of side effects.

A patient with AKI has a serum potassium level of 6.7 mEq/L (6.7 mmol/L) and the following arterial blood gas results: pH 7.28, PaCO2 30 mm Hg, PaO2 86 mm Hg, HCO3 − 18 mEq/L (18 mmol/L). The nurse recognizes that treatment of the acid-base problem with sodium bicarbonate would cause a decrease in which value? a. pH b. Potassium level c. Bicarbonate level d. Carbon dioxide level

b. During acidosis, potassium moves out of the cell in exchange for H+ ions, increasing the serum potassium level. Correction of the acidosis with sodium bicarbonate will help to shift the potassium back into the cells. A decrease in pH and the bicarbonate and PaCO2 levels would indicate worsening acidosis.

What information should be included when the nurse teaches a patient about colostomy irrigation? a. Infuse 1500 to 2000 mL of warm tap water as irrigation fluid. b. Allow 30 to 45 minutes for the solution and feces to be expelled. c. Insert a firm plastic catheter 3 to 4 inches into the stoma opening. d. Hang the irrigation bag on a hook about 36 inches above the stoma.

b. Following infusion of the fluid into the stoma, the solution and feces will take about 30 to 45 minutes to return and the patient can plan to read or perform other quiet activities during the wait time. Between 500 and 1000 mL of warm tap water should be used. A cone tip on the end of the tubing prevents bowel damage that could occur if a stiff plastic catheter is used. Fluid should be elevated about 18 to 24 inches above the stoma, or to about shoulder level, to prevent too rapid infusion of the solution and cramping.

Priority Decision: A dehydrated patient is in the Injury stage of the RIFLE staging of AKI. What would the nurse first anticipate in the treatment of this patient? a. Assess daily weight b. IV administration of fluid and furosemide (Lasix) c. IV administration of insulin and sodium bicarbonate d. Urinalysis to check for sediment, osmolality, sodium, and specific gravity

b. Injury is the stage of RIFLE classification when urine output is less than 0.5 mL/kg/hr for 12 hours, the serum creatinine is increased times two or the glomerular filtration rate (GFR) is decreased by 50%. This stage may be reversible by treating the cause or, in this patient, the dehydration by administering IV fluid and a low dose of a loop diuretic, furosemide (Lasix). Assessing the daily weight will be done to monitor fluid changes but it is not the first treatment the nurse should anticipate. IV administration of insulin and sodium bicarbonate would be used for hyperkalemia. Checking the urinalysis will help to determine if the AKI has a prerenal, intrarenal, or postrenal cause by what is seen in the urine but with this patient's dehydration, it is thought to be prerenal to begin treatment.

The patient with CKD asks why she is receiving nifedipine (Procardia) and furosemide (Lasix). The nurse understands that these drugs are being used to treat the patient's a. anemia. b. hypertension. c. hyperkalemia. d. mineral and bone disorder.

b. Nifedipine (Procardia) is a calcium channel blocker and furosemide (Lasix) is a loop diuretic. Both are used to treat hypertension.

The nurse should administer an as-needed dose of magnesium hydroxide (MOM) after noting what information while reviewing a patient's medical record? a. Abdominal pain and bloating b. No bowel movement for 3 days c. A decrease in appetite by 50% over 24 hours d. Muscle tremors and other signs of hypomagnesemia

b. No bowel movement for 3 days MOM is an osmotic laxative that produces a soft, semisolid stool usually within 15 minutes to 3 hours. This medication would benefit the patient who has not had a bowel movement for 3 days. MOM would not be given for abdominal pain and bloating, decreased appetite, or signs of hypomagnesemia.

To prevent the most common serious complication of PD, what is important for the nurse to do? a. Infuse the dialysate slowly. b. Use strict aseptic technique in the dialysis procedures. c. Have the patient empty the bowel before the inflow phase. d. Reposition the patient frequently and promote deep breathing.

b. Peritonitis is a common complication of peritoneal dialysis (PD) and may require catheter removal and termination of dialysis. Infection occurs from contamination of the dialysate or tubing or from progression of exit-site or tunnel infections and strict sterile technique must be used by health professionals as well as the patient to prevent contamination. Too-rapid infusion may cause shoulder pain and pain may be caused if the catheter tip touches the bowel. Difficulty breathing, atelectasis, and pneumonia may occur from pressure of the fluid on the diaphragm, which may be prevented by elevating the head of the bed and promoting repositioning and deep breathing.

What is a nursing intervention that is indicated for a male patient following an inguinal herniorrhaphy? a. Applying heat to the inguinal area c. Applying a truss to support the operative site b. Elevating the scrotum with a scrotal support d. Encouraging the patient to cough and deep breathe

b. Scrotal edema is a common and painful complication after an inguinal hernia repair and can be relieved in part by application of ice and elevation of the scrotum with a scrotal support. Heat would increase the edema and the discomfort and a truss is used to keep unrepaired hernias from protruding. Coughing is discouraged postoperatively because it increases intraabdominal pressure and stress on the repair site.

Which patient is most likely to be diagnosed with short bowel syndrome? a. History of ulcerative colitis c. Diagnosed with irritable bowel syndrome b. Had extensive resection of the ileum d. Had colectomy performed for cancer of the bowel

b. Short bowel syndrome results from extensive resection of portions of the small bowel and would occur if a patient had an extensive resection of the ileum. The other conditions primarily affect the large colon and result in fewer and less severe symptoms.

The patient has persistent and continuous pain at McBurney's point. The nursing assessment reveals rebound tenderness and muscle guarding with the patient preferring to lie still with the right leg flexed. What should the nursing interventions for this patient include? a. Laxatives to move the constipated bowel b. NPO status in preparation for possible appendectomy c. Parenteral fluids and antibiotic therapy for 6 hours before surgery d. NG tube inserted to decompress the stomach and prevent aspiration

b. The patient's manifestations are characteristic of appendicitis. After laboratory test and CT scan confirmation, the patient will have surgery. Laxatives are not used. The 6 hours of fluids and antibiotics preoperatively would be used only if the appendix was ruptured. The NG tube is more likely to be used with abdominal trauma.

A 68-year-old man with a history of heart failure resulting from hypertension has AKI as a result of the effects of nephrotoxic diuretics. Currently his serum potassium is 6.2 mEq/L (6.2 mmol/L) with cardiac changes, his BUN is 108 mg/dL (38.6 mmol/L), his serum creatinine is 4.1 mg/dL (362 mmol/L), and his serum HCO3 − is 14 mEq/L (14 mmol/L). He is somnolent and disoriented. Which treatment should the nurse expect to be used for him? a. Loop diuretics b. Renal replacement therapy c. Insulin and sodium bicarbonate d. Sodium polystyrene sulfonate (Kayexalate)

b. This patient has at least three of the six common indications for renal replacement therapy (RRT), including (1) high potassium level, (2) metabolic acidosis, and (3) changed mental status. The other indications are (4) volume overload, resulting in compromised cardiac status (this patient has a history of hypertension), (5) BUN greater than 120 mg/dL, and (6) pericarditis, pericardial effusion, or cardiac tamponade. Although the other treatments may be used, they will not be as effective as RRT for this older patient. Loop diuretics and increased fluid are used if the patient is dehydrated. Insulin and sodium bicarbonate can be used to temporarily drive the potassium into the cells. Sodium polystyrene sulfonate (Kayexalate) is used to actually decrease the amount of potassium in the body.

Situation: Three year old Carlo has been admitted to the pediatric unit with a tentative diagnosis of nephrotic syndrome. The diagnosis of Idiopathic Nephrotic Syndrome has been confirmed. Which unexpected finding would the nurse report? a) proteinuria b) distended abdomen c) blood in the urine d) elevated serum lipid levels

c) blood in the urine hematuria is rare in nephrotic syndrome but it is profuse is acute glomerulonephritis The manifestations of nephrotic syndrome are: Proteinuria - nephrosis is believed to be due to immunologic response that results in increased permeability of glomerular membrane to proteins resulting in massive protein losses in the urine -- proteinuria and albuminuria (+3 +4), the child losses 50-100 mg/kg weight/day from proteinuria. Hypoalbuminemia - loss of protein in blood results in hypoalbumenimia Edema - cardinal sign and appears first in the periorbital region followed by dependent edema and accompanied by pallor, fatigue and lethargy. Hypoalbuminemia leads to decreased oncotic pressure resulting in fluid shift from intravascular to interstitial causing generalized edema or anasarca.The child has lost appetite but gained weight -- puffiness of the eyes on awakening decreases during the day but appears on the legs and abdomen. Fluid shift causes decreased blood volume that leads to decreased blood supply to kidney. Decreased blood supply to kidney initiates release of aldosterone. Aldosterone causes sodium retention (in interstitial spaces so child will have hyponatremia) and water retention contributing to edema. Hypocholesteronemia and hyperlipidemia - triglycerides and fats are released by the liver in the blood to make up for the protein loss

Three year old Carlo has been admitted to the pediatric unit with a tentative diagnosis of nephrotic syndrome. Carlo's potential for impairment of skin integrity is related to: a) joint inflammation b) drug therapy c) edema d) generalized body rash

c) edema - management: reduce protein excretion Prevention of Skin Breakdown from Edema frequent turning keep nails short to prevent scratching meticulous skin care to dependent and edematous areas - sacrum, scrotum, labia, abdomen, legs loose clothing Monitor Edema weigh daily and monitor I and O check for pulmonary edema manifested by crackles on auscultation ascites - measure abdominal girth Prevention of Infection - pulmonary edema predisposes to respiratory infection and generalized edema predisposes to skin breakdown. Avoid contact with persons who have infection. Diet - usually anorexic because of GI edema high protein diet sodium restriction if with severe edema fluid intake equal to output and insensible loss vitamin and iron supplements small feedings, give favorite foods

A client has been diagnosed to have chronic renal failure. Sodium polysterene sulfonate (exchange resin kayexalate) is prescribed. The action of the medication is that it releases a) bicarbonate in exchange for primarily sodium ions b) sodium ions in exchange for primarily bicarbonate ions c) sodium ions in exchange for primarily potassium ions d) potassium ions in exchange for primarily sodium ions

c) sodium ions in exchange for primarily potassium ions

The client arrives at the emergency department with complaints of low abdominal pain and hematuria. The client is afebrile. The nurse next assesses the client to determine a history of: a) pyelonephritis b) glomerulonephritis c) trauma to the bladder or abdomen d) renal cancer in the client's family

c) trauma to the bladder or abdomen Use the process of elimination. Eliminate options A and B, knowing that any inflammatory disease or infection is accompanied by fever. Because this client is afebrile, these are not possible options. Use knowledge of anatomy and pain assessment to select option C. Pain from renal cancer is a later finding and is localized in the flank area.

A patient on hemodialysis develops a thrombus of a subcutaneous arteriovenous (AV) graft, requiring its removal. While waiting for a replacement graft or fistula, the patient is most likely to have what done for treatment? a. Peritoneal dialysis b. Peripheral vascular access using radial artery c. Silastic catheter tunneled subcutaneously to the jugular vein d. Peripherally inserted central catheter (PICC) line inserted into subclavian vein

c. A more permanent, soft, flexible Silastic double-lumen catheter is used for long-term access when other forms of vascular access have failed. These catheters are tunneled subcutaneously and have Dacron cuffs that prevent infection from tracking along the catheter.

The patient asks the nurse why she needs to have surgery for a femoral, strangulated hernia. What is the best explanation the nurse can give the patient? a. The surgery will relieve her constipation. b. The abnormal hernia must be replaced into the abdomen. c. The surgery is needed to allow intestinal flow and prevent necrosis. d. The hernia is because the umbilical opening did not close after birth as it should have.

c. A strangulated femoral hernia obstructs intestinal flow and blood supply, thus requiring emergency surgery. The other options are incorrect.

Priority Decision: A patient returns to the surgical unit with a nasogastric (NG) tube to low intermittent suction, IV fluids, and a Jackson-Pratt drain at the surgical site following an exploratory laparotomy and repair of a bowel perforation. Four hours after admission, the patient experiences nausea and vomiting. What is a priority nursing intervention for the patient? a. Assess the abdomen for distention and bowel sounds. b. Inspect the surgical site and drainage in the Jackson-Pratt. c. Check the amount and character of gastric drainage and the patency of the NG tube. d. Administer prescribed prochlorperazine (Compazine) to control the nausea and vomiting.

c. An adequately functioning nasogastric (NG) tube should prevent nausea and vomiting because stomach contents are continuously being removed. The first intervention in this case is to check the amount and character of the recent drainage and check the tube for patency. Decreased or absent bowel sounds are expected after a laparotomy and the Jackson-Pratt drains only fluid from the tissue of the surgical site. Antiemetics may be given if the NG tube is patent because anesthetic agents may cause nausea.

The nurse identifies a need for additional teaching when a patient with acute infectious diarrhea makes which statement? a. "I can use A&D ointment or Vaseline jelly around the anal area to protect my skin." b. "Gatorade is a good liquid to drink because it replaces the fluid and salts I have lost." c. "I may use over-the-counter Imodium or Parepectolin when I need to control the diarrhea." d. "I must wash my hands after every bowel movement to prevent spreading the diarrhea to my family."

c. Antiperistaltic agents, such as loperamide (Imodium) and paregoric, should not be used in infectious diarrhea because of the potential of prolonging exposure to the infectious agent. Demulcent agents may be used to coat and protect mucous membranes in these cases. The other options are all appropriate measures to use in cases of infectious diarrhea.

The medications prescribed for the patient with inflammatory bowel disease include cobalamin and iron injections. What is the rationale for using these drugs? a. Alleviate stress c. Correct malnutrition b. Combat infection d. Improve quality of life

c. Cobalamin and iron injections will help to correct malnutrition. Correcting malnutrition will also indirectly help to improve quality of life and fight infections.

What does the dialysate for PD routinely contain? a. Calcium in a lower concentration than in the blood b. Sodium in a higher concentration than in the blood c. Dextrose in a higher concentration than in the blood d. Electrolytes in an equal concentration to that of the blood

c. Dextrose or icodextrin or amino acid is added to dialysate fluid to create an osmotic gradient across the membrane to remove excess fluid from the blood. The dialysate fluid has no potassium so that potassium will diffuse into the dialysate from the blood. Dialysate also usually contains higher calcium to promote its movement into the blood. Dialysate sodium is usually less than or equal to that of blood to prevent sodium and fluid retention.

What should the nurse teach the patient with diverticulosis to do? a. Use anticholinergic drugs routinely to prevent bowel spasm. b. Have an annual colonoscopy to detect malignant changes in the lesions. c. Maintain a high-fiber diet and use bulk laxatives to increase fecal volume. d. Exclude whole grain breads and cereals from the diet to prevent irritating the bowel.

c. Formation of diverticula is common when decreased bulk of stool, combined with a more narrowed lumen in the sigmoid colon, causes high intraluminal pressures that result in saccular dilation or outpouching of the mucosa through the muscle of the intestinal wall. To prevent the high intraluminal pressure, fecal volume should be increased with use of high-fiber diets and bulk laxatives, such as psyllium (Metamucil). Anticholinergic drugs are used only during an acute episode of diverticulitis and the lesions are not premalignant.

The patient comes to the emergency department with intermittent crampy abdominal pain, nausea, projectile vomiting, and dehydration. The nurse suspects a GI obstruction. Based on the manifestations, what area of the bowel should the nurse suspect is obstructed? a. Large intestine c. Upper small intestine b. Esophageal sphincter d. Lower small intestine

c. Intermittent crampy abdominal pain, nausea, projectile vomiting, and dehydration are characteristics of mechanical upper small intestinal obstruction. With continued vomiting, metabolic alkalosis may occur. Large bowel obstruction is characterized by constipation, low-grade abdominal pain, and abdominal distention. Fecal vomiting is seen with lower small intestinal obstruction.

Following bowel resection, a patient has a nasogastric (NG) tube to suction, but complains of nausea and abdominal distention. The nurse irrigates the tube as necessary as ordered, but the irrigating fluid does not return. What should be the priority action by the nurse? a. Notify the physician. b. Auscultate for bowel sounds. c. Reposition the tube and check for placement. d. Remove the tube and replace it with a new one.

c. Reposition the tube and check for placement. The tube may be resting against the stomach wall. The first action by the nurse (since this is intestinal surgery and not gastric surgery) is to reposition the tube and check it again for placement. The physician does not need to be notified unless the tube function cannot be restored by the nurse. The patient does not have bowel sounds, which is why the NG tube is in place. The NG tube would not be removed and replaced unless it was no longer in the stomach or the obstruction of the tube could not be relieved.

A patient is diagnosed with celiac disease following a workup for iron-deficiency anemia and decreased bone density. The nurse identifies that additional teaching about disease management is needed when the patient makes which statement? a. "I should ask my close relatives to be screened for celiac disease." b. "If I do not follow the gluten-free diet, I might develop a lymphoma." c. "I don't need to restrict gluten intake because I don't have diarrhea or bowel symptoms." d. "It is going to be difficult to follow a gluten-free diet because it is found in so many foods."

c. The autoimmune process associated with celiac disease continues as long as the body is exposed to gluten, regardless of the symptoms it produces, and a lifelong gluten-free diet is necessary. The other statements regarding celiac disease are all true

For a patient with CKD the nurse identifies a nursing diagnosis of risk for injury: fracture related to alterations in calcium and phosphorus metabolism. What is the pathologic process directly related to the increased risk for fractures? a. Loss of aluminum through the impaired kidneys b. Deposition of calcium phosphate in soft tissues of the body c. Impaired vitamin D activation resulting in decreased GI absorption of calcium d. Increased release of parathyroid hormone in response to decreased calcium levels

c. The calcium-phosphorus imbalances that occur in CKD result in hypocalcemia, from a deficiency of active vitamin D and increased phosphorus levels. This leads to an increased rate of bone remodeling with a weakened bone matrix. Aluminum accumulation is also believed to contribute to the osteomalacia. Osteitis fibrosa involves replacement of calcium in the bone with fibrous tissue and is primarily a result of elevated levels of parathyroid hormone resulting from hypocalcemia.

During the nursing assessment of the patient with renal insufficiency, the nurse asks the patient specifically about a history of a. angina. b. asthma. c. hypertension. d. rheumatoid arthritis.

c. The most common causes of CKD in the United States are diabetes mellitus and hypertension. The nurse should obtain information on long-term health problems that are related to kidney disease. The other disorders are not closely associated with renal disease.

What causes the gastrointestinal (GI) manifestation of stomatitis in the patient with CKD? a. High serum sodium levels b. Irritation of the GI tract from creatinine c. Increased ammonia from bacterial breakdown of urea d. Iron salts, calcium-containing phosphate binders, and limited fluid intake

c. Uremic fetor, or the urine odor of the breath, is caused by high urea content in the blood. Increased ammonia from bacterial breakdown of urea leads to stomatitis and mucosal ulcerations. Irritation of the gastrointestinal (GI) tract from urea in CKD contributes to anorexia, nausea, and vomiting. Ingestion of iron salts and calcium-containing phosphate binders, limited fluid intake, and limited activity cause constipation.

Priority Decision: What is the most important thing the nurse should do when caring for a patient who has contracted Clostridium difficile? a. Clean the entire room with ammonia. b. Feed the patient yogurt with probiotics. c. Wear gloves and wash hands with soap and water. d. Teach the family to use alcohol-based hand cleaners.

c. Wearing gloves will avoid hand contamination and washing hands with soap and water will remove more Clostridium difficile spores than alcohol-based hand cleaners and ammonia-based disinfectants. The entire room will need to be disinfected with a 10% solution of household bleach. Probiotics may help to prevent diarrhea in the patient on antibiotics by replacing normal intestinal bacteria.

A patient with stage I colorectal cancer is scheduled for surgery. Patient teaching for this patient would include an explanation that: a. chemotherapy will begin after the patient recovers from the surgery b. both chemotherapy and radiation can be used as palliative treatments c. follow-up colonoscopies will be needed to ensure that the cancer does not occur d. a wound, ostomy, and continence nurse will visit the patient to identify an abdominal site for the ostomy

c. follow-up colonoscopies will be needed to ensure that the cancer does not occur Rationale: Stage 1 colorectal cancer is treated with surgical removal of the tumor and reanastomosis, and so there is no ostomy. Chemotherapy is not recommended for stage I tumors. Follow-up colonoscopy is recommended because colorectal cancer can recur.

In planning care for the patient with Crohn's disease, the nurse recognizes that a major difference between ulcerative colitis and Crohn's disease is that Crohn's disease: a. frequently results in toxic megacolon b. causes fever nutritional deficiencies than ulcerative colitis c. often recurs after surgery, whereas ulcerative colitis is curable with a colectomy d. is manifested by rectal bleeding and anemia more frequently than is ulcerative colitis

c. often recurs after surgery, whereas ulcerative colitis is curable with a colectomy Rationale: Ulcerative colitis affects only the colon and rectum; it can cause megacolon and rectal bleeding, but not nutrient malabsorption. Surgical removal of the colon and rectum cures it. Crohn's disease usually involves the ileum, where bile salts and vitamin cobalamin are absorbed. After surgical treatment, disease recurrence at the site is common.

A patient with diabetes who has bacterial pneumonia is being treated with IV gentamicin (Garamycin) 60 mg IV BID. The nurse will monitor for adverse effects of the medication by evaluating the patient's a. blood glucose. b. urine osmolality. c. serum creatinine. d. serum potassium.

c. serum creatinine.

In planning preoperative teaching for a patient undergoing a RouxenY gastric bypass as treatment for morbid obesity, the nurse places the highest priority on a. demonstrating passive rangeofmotion exercises to the legs. b. discussing the necessary postoperative modifications in lifestyle. c. teaching the patient proper coughing and deep breathing techniques. d. educating the patient about the postoperative presence of a nasogastric (NG) tube.

c. teaching the patient proper coughing and deep breathing techniques. Coughing and deep breathing can prevent major postoperative complications such as carbon monoxide retention and hypoxemia. Information about passive range of motion, the NG tube, and postoperative modifications in lifestyle also will be discussed, but avoidance of respiratory complications is the priority goal after surgery.

The nurse develops a post-procedure plan of care for a client who had a renal biopsy. The nurse avoids documenting which intervention in the plan? a) administering analgesics as needed b) encouraging fluids to at least 3L in the first 24 hours c) testing serial urine samples with dipstick for occult blood d) ambulating the client in the room and hall for short distances

d) ambulating the client in the room and hall for short distances

Which of the following may be included in the diet of the client with chronic renal failure? a) orange slices b) watermelon slices c) cantaloupe slices d) apple slices

d) apple slices the client with renal failure should be given low potassium diet because of hyperkalemia. Apple contains very little potassium. So, it can be given to the client.

A client has an arteriovenous (AV) fistula in place in the right upper extremity for hemodialysis treatments. When planning care for this client, which of the following measures should the nurse implement to promote client safely? a) take blood pressures only on the right arm to ensure accuracy b) use the fistula for all venipunctures and intravenous infusions c) ensure that small clamps are attached to the AV fistula dressing d) assess the fistula for the presence of a bruit and thrill every 4 hours

d) assess the fistula for the presence of a bruit and thrill every 4 hours

The client with an external arteriovenous shunt in place for hemodialysis is at risk for bleeding. The priority nurse action would be to: a) check the shunt for the presence of bruit and thrill b) observe the site once as time permits during the shift c) check the results of the prothrombin time as they are determined d) ensure that small clamps are attached to the arteriovenous shunt dressing

d) ensure that small clamps are attached to the arteriovenous shunt dressing - An arteriovenous shunt is a less common form of access site but carries a risk for bleeding when it is used because two ends of an external cannula are tunneled subcutaneously into an artery and a vein, and the ends of the cannula are joined. If accidental disconnection occurs, the client could lose blood rapidly. For this reason, small clamps are attached to the dressing that covers the insertion site for use if needed. The shunt site also should be assessed at least every 4 hours.

What are intrarenal causes of acute kidney injury (AKI) (select all that apply)? a. Anaphylaxis b. Renal stones c. Bladder cancer d. Nephrotoxic drugs e. Acute glomerulonephritis f. Tubular obstruction by myoglobin

d, e, f. Intrarenal causes of acute kidney injury (AKI) include conditions that cause direct damage to the kidney tissue, including nephrotoxic drugs, acute glomerulonephritis, and tubular obstruction by myoglobin, or prolonged ischemia. Anaphylaxis and other prerenal problems are frequently the initial cause of AKI. Renal stones and bladder cancer are among the postrenal causes of AKI.

Delegation Decision: The RN coordinating the care for a patient who is 2 days postoperative following an anterior- posterior resection with colostomy may delegate which interventions to the licensed practical nurse (LPN) (select all that apply)? a. Irrigate the colostomy. b. Teach ostomy and skin care. c. Assess and document stoma appearance. d. Monitor and record the volume, color, and odor of the drainage. e. Empty the ostomy bag and measure and record the amount of drainage.

d, e. The licensed practical nurse (LPN) can monitor and record observations related to the drainage and can measure and record the amount. The LPN could also monitor the skin around the stoma for breakdown. LPNs can irrigate a colostomy in a stable patient but this patient is only 2 days postoperative. The other actions are responsibilities of the RN (teaching, assessing stoma, and developing a care plan).

The nurse is caring for a postoperative patient with a colostomy. The nurse is preparing to administer a dose of famotidine (Pepcid) when the patient asks why the medication was ordered since the patient does not have a history of heartburn or gastroesophageal reflux disease (GERD). What response by the nurse would be the most appropriate? a. "This will prevent air from accumulating in the stomach, causing gas pains." b. "This will prevent the heartburn that occurs as a side effect of general anesthesia." c. "The stress of surgery is likely to cause stomach bleeding if you do not receive it." d. "This will reduce the amount of HCl in the stomach until the nasogastric tube is removed and you can eat a regular diet again."

d. "This will reduce the amount of HCl in the stomach until the nasogastric tube is removed and you can eat a regular diet again." Famotidine is an H2-receptor antagonist that inhibits gastric HCl secretion and thus minimizes damage to gastric mucosa while the patient is not eating a regular diet after surgery. Famotidine does not prevent air from accumulating in the stomach or stop the stomach from bleeding. Heartburn is not a side effect of general anesthesia.

When developing a weight reduction plan for an obese patient who wants to lose weight, which question should the nurse ask first? a. "Which food types do you like best?" b. "How long have you been overweight?" c. "What kind of physical activities do you enjoy?" d. "What factors do you think led to your obesity?"

d. "What factors do you think led to your obesity?" The nurse should obtain information about the patient's perceptions of the reasons for the obesity to develop a plan individualized to the patient. The other information also will be obtained from the patient, but the patient is more likely to make changes when the patient's beliefs are considered in planning.

Acute tubular necrosis (ATN) is the most common cause of intrarenal AKI. Which patient is most likely to develop ATN? a. Patient with diabetes mellitus b. Patient with hypertensive crisis c. Patient who tried to overdose on acetaminophen d. Patient with major surgery who required a blood transfusion

d. Acute tubular necrosis (ATN) is primarily the result of ischemia, nephrotoxins, or sepsis. Major surgery is most likely to cause severe kidney ischemia in the patient requiring a blood transfusion. A blood transfusion hemolytic reaction produces nephrotoxic injury if it occurs. Diabetes mellitus, hypertension, and acetaminophen overdose will not contribute to ATN.

The nurse asks a 68-year-old patient scheduled for colectomy to sign the operative permit as directed in the physician's preoperative orders. The patient states that the physician has not really explained very well what is involved in the surgical procedure. What is the most appropriate action by the nurse? a. Ask family members whether they have discussed the surgical procedure with the physician. b. Have the patient sign the form and state the physician will visit to explain the procedure before surgery. c. Explain the planned surgical procedure as well as possible and have the patient sign the consent form. d. Delay the patient's signature on the consent and notify the physician about the conversation with the patient.

d. Delay the patient's signature on the consent and notify the physician about the conversation with the patient. The patient should not be asked to sign a consent form unless the procedure has been explained to the satisfaction of the patient. The nurse should notify the physician, who has the responsibility for obtaining consent.

A patient rapidly progressing toward end-stage kidney disease asks about the possibility of a kidney transplant. In responding to the patient, the nurse knows that what is a contraindication to kidney transplantation? a. Hepatitis C infection b. Coronary artery disease c. Refractory hypertension d. Extensive vascular disease

d. Extensive vascular disease is a contraindication for renal transplantation, primarily because adequate blood supply is essential for the health of the new kidney. Other contraindications include disseminated malignancies, refractory or untreated cardiac disease, chronic respiratory failure, chronic infection, or unresolved psychosocial disorders. Coronary artery disease (CAD) may be treated with bypass surgery before transplantation and transplantation can relieve hypertension. Hepatitis B or C infection is not a contraindication.

While caring for the patient in the oliguric phase of AKI, the nurse monitors the patient for associated collaborative problems. When should the nurse notify the health care provider? a. Urine output is 300 mL/day. b. Edema occurs in the feet, legs, and sacral area. c. Cardiac monitor reveals a depressed T wave and elevated ST segment. d. The patient experiences increasing muscle weakness and abdominal cramping.

d. Hyperkalemia is a potentially life-threatening complication of AKI in the oliguric phase. Muscle weakness and abdominal cramping are signs of the neuromuscular impairment that occurs with hyperkalemia. In addition, hyperkalemia can cause the cardiac conduction abnormalities of peaked T wave, prolonged PR interval, prolonged QRS interval, and depressed ST segment. Urine output of 300 mL/day is expected during the oliguric phase, as is the development of peripheral edema.

A nurse is doing a nursing assessment on a patient with chronic constipation. What data obtained during the interview may be a factor contributing to the constipation? a. Taking methylcellulose (Citrucel) daily c. History of hemorrhoids and hypertension b. High dietary fiber with high fluid intake d. Suppressing the urge to defecate while at work

d. Ignoring the urge to defecate causes the muscles and mucosa in the rectal area to become insensitive to the presence of feces and drying of the stool occurs. The urge to defecate is decreased and stool becomes more difficult to expel. Taking a bulk-forming agent with fluids or highfiber diet with fluids prevent constipation. Hemorrhoids are the most common complication of chronic constipation, caused by straining to pass hardened stool. The straining may cause problems in patients with hypertension but these do not cause constipation. Other things that may cause constipation are a history of diverticulosis, which is seen in individuals with low fiber intake, small stool mass, and hard stools. Chronic laxative use and chronic dilation and loss of colonic tone may also cause chronic constipation.

What indicates to the nurse that a patient with oliguria has prerenal oliguria? a. Urine testing reveals a low specific gravity. b. Causative factor is malignant hypertension. c. Urine testing reveals a high sodium concentration. d. Reversal of oliguria occurs with fluid replacement.

d. In prerenal oliguria, the oliguria is caused by a decrease in circulating blood volume and there is no damage yet to the renal tissue. It can be reversed by correcting the precipitating factor, such as fluid replacement for hypovolemia. Prerenal oliguria is characterized by urine with a high specific gravity and a low sodium concentration, whereas oliguria of intrarenal failure is characterized by urine with a low specific gravity and a high sodium concentration. Malignant hypertension causes damage to renal tissue and intrarenal oliguria.

What accurately describes the care of the patient with CKD? a. A nutrient that is commonly supplemented for the patient on dialysis because it is dialyzable is iron. b. The syndrome that includes all of the signs and symptoms seen in the various body systems in CKD is azotemia. c. The use of morphine is contraindicated in the patient with CKD because accumulation of its metabolites may cause seizures. d. The use of calcium-based phosphate binders in the patient with CKD is contraindicated when serum calcium levels are increased.

d. In the patient with CKD, when serum calcium levels are increased, calcium-based phosphate binders are not used. The nutrient supplemented for patients on dialysis is folic acid. The various body system manifestations occur with uremia, which includes azotemia. Meperidine is contraindicated in patients with CKD related to possible seizures.

The nurse is preparing to administer a dose of bisacodyl (Dulcolax). In explaining the medication to the patient, the nurse would explain that it acts in what way? a. Increases bulk in the stool b. Lubricates the intestinal tract to soften feces c. Increases fluid retention in the intestinal tract d. Increases peristalsis by stimulating nerves in the colon wall

d. Increases peristalsis by stimulating nerves in the colon wall Bisacodyl is a stimulant laxative that aids in producing a bowel movement by irritating the colon wall and stimulating enteric nerves. It is available in oral and suppository forms. Fiber and bulk forming drugs increase bulk in the stool; water and stool softeners soften feces, and saline and osmotic solutions cause fluid retention in the intestinal tract.

In caring for the patient with AKI, what should the nurse be aware of? a. The most common cause of death in AKI is irreversible metabolic acidosis. b. During the oliguric phase of AKI, daily fluid intake is limited to 1000 mL plus the prior day's measured fluid loss. c. Dietary sodium and potassium during the oliguric phase of AKI are managed according to the patient's urinary output. d. One of the most important nursing measures in managing fluid balance in the patient with AKI is taking accurate daily weights.

d. Measuring daily weights with the same scale at the same time each day allows for the evaluation and detection of excessive body fluid gains or losses. Infection is the leading cause of death in AKI, so meticulous aseptic technique is critical. The fluid limitation in the oliguric phase is 600 mL plus the prior day's measured fluid loss. Dietary sodium and potassium intake are managed according to the plasma levels.

A male patient who has undergone an anterior-posterior repair is worried about his sexuality. What is an appropriate nursing intervention for this patient? a. Have the patient's sexual partner reassure the patient that he is still desirable. b. Reassure the patient that sexual function will return when healing is complete. c. Remind the patient that affection can be expressed in ways other than through sexual intercourse. d. Explain that physical and emotional factors can affect sexual function but not necessarily the patient's sexuality.

d. Sexual dysfunction may result from an anterior-posterior repair but the nurse should discuss with the patient that different nerve pathways affect erection, ejaculation, and orgasm and that a dysfunction of one does not mean total sexual dysfunction and also that an alteration in sexual activity does not have to alter sexuality. Simple reassurance of desirability and ignoring concerns about sexual function do not help the patient to regain positive feelings of sexuality.

What should the nurse instruct the patient to do to best enhance the effectiveness of a daily dose of docusate sodium (Colace)? a. Take a dose of mineral oil at the same time. b. Add extra salt to food on at least one meal tray. c. Ensure dietary intake of 10 g of fiber each day. d. Take each dose with a full glass of water or other liquid.

d. Take each dose with a full glass of water or other liquid. Docusate lowers the surface tension of stool, permitting water and fats to penetrate and soften the stool for easier passage. The patient should take the dose with a full glass of water and should increase overall fluid intake, if able, to enhance effectiveness of the medication. Dietary fiber intake should be a minimum of 20 g daily to prevent constipation. Mineral oil and extra salt are not recommended.

What indicates to the nurse that a patient with AKI is in the recovery phase? a. A return to normal weight b. A urine output of 3700 mL/day c. Decreasing sodium and potassium levels d. Decreasing blood urea nitrogen (BUN) and creatinine levels

d. The blood urea nitrogen (BUN) and creatinine levels remain high during the oliguric and diuretic phases of AKI. The recovery phase begins when the glomerular filtration returns to a rate at which BUN and creatinine stabilize and then decrease. Urinary output of 3 to 5 L/ day, decreasing sodium and potassium levels, and fluid weight loss are characteristic of the diuretic phase of AKI.

Priority Decision: In instituting a bowel training program for a patient with fecal incontinence, what should the nurse first plan to do? a. Teach the patient to use a perianal pouch. b. Insert a rectal suppository at the same time every morning. c. Place the patient on a bedpan 30 minutes before breakfast. d. Assist the patient to the bathroom at the time of the patient's normal defecation.

d. The first intervention to establish bowel regularity includes promoting bowel evacuation at a regular time each day, preferably by placing the patient on the bedpan, using a bedside commode, or walking the patient to the bathroom. To take advantage of the gastrocolic reflex, an appropriate time is 30 minutes after the first meal of the day or at the patient's usual individual time. Perianal pouches are used to protect the skin only when regularity cannot be established and evacuation suppositories are also used only if other techniques are not successful.

The patient with a new ileostomy needs discharge teaching. What should the nurse plan to include in this teaching? a. The pouch can be worn for up to 2 weeks before changing it. b. Decrease the amount of fluid intake to decrease the amount of drainage. c. The pouch can be removed when bowel movements have been regulated. d. If leakage occurs, promptly remove the pouch, clean the skin, and apply a new pouch

d. The ileostomy drainage is extremely irritating to the skin, so the skin must be cleaned and a new solid skin barrier and pouch applied as soon as a leak occurs to prevent skin damage. The pouch is usually worn for 4 to 7 days unless there is a leak. Because the initial drainage from the ileostomy is high, the fluid intake must not be decreased. The pouch must always be worn, as the liquid drainage, not formed bowel movements, is frequent.

A 22-year-old patient calls the outpatient clinic complaining of nausea and vomiting and right lower abdominal pain. What should the nurse advise the patient to do? a. Use a heating pad to relax the muscles at the site of the pain. b. Drink at least 2 quarts of juice to replace the fluid lost in vomiting. c. Take a laxative to empty the bowel before examination at the clinic. d. Have the symptoms evaluated by a health care provider right away.

d. The patient is having symptoms of an acute abdomen and should be evaluated by a health care provider immediately. The patient's age, location of pain, and other symptoms are characteristic of appendicitis. Heat application and laxatives should not be used in patients with undiagnosed abdominal pain because they may cause perforation of the appendix or other inflammations. Fluids should not be taken until vomiting is controlled, nor should they be taken in the event that surgery may be performed.

In report, the nurse learns that the patient has a transverse colostomy. What should the nurse expect when providing care for this patient? a. Semiliquid stools with increased fluid requirements b. Liquid stools in a pouch and increased fluid requirements c. Formed stools with a pouch, needing irrigation, but no fluid needs d. Semiformed stools in a pouch with the need to monitor fluid balance

d. The patient with a transverse colostomy has semiliquid to semiformed stools needing a pouch and needs to have fluid balance monitored. The ascending colostomy has semiliquid stools needing a pouch and increased fluid. The ileostomy has liquid to semiliquid stools needing a pouch and increased fluid. The sigmoid colostomy has formed stools and may or may not need a pouch but will need irrigation.

What extraintestinal manifestations are seen in both ulcerative colitis and Crohn's disease? a. Celiac disease and gallstones c. Conjunctivitis and colonic dilation b. Peptic ulcer disease and uveitis d. Erythema nodosum and osteoporosis

d. Ulcerative colitis and Crohn's disease have many of the same extraintestinal symptoms, including erythema nodosum and osteoporosis, as well as gallstones, uveitis, and conjunctivitis. Colonic dilation and celiac disease are not extraintestinal.

What should a patient be taught after a hemorrhoidectomy? a. Take a mineral oil before bedtime b. Eat a low-fiber diet to rest the colon c. Administer oil-retention enema to empty the colon d. Use prescribed pain medication before a bowel movement

d. Use prescribed pain medication before a bowel movement Rationale: After a hemorrhoidectomy, the patient usually dreads the first bowel movement and often resists the urge to defecate. Pain medication may be given before the bowel movement to reduce discomfort. The patient should avoid constipation and straining. A high-fiber diet can reduce constipation. A stool softener such as docusate (Colace) is usually ordered for the first few postoperative days. If the patient does not have a bowel movement within 2 to 3 days, an oil-retention enema is administered.

Following a hemorrhoidectomy, what should the nurse advise the patient to do? a. Use daily laxatives to facilitate bowel emptying. b. Use ice packs to the perineum to prevent swelling. c. Avoid having a bowel movement for several days until healing occurs. d. Take warm sitz baths several times a day to promote comfort and cleaning.

d. Warm sitz baths provide comfort, healing, and cleansing of the area following all anorectal surgery and may be done three or four times a day for 1 to 2 weeks. Stool softeners may be prescribed for several days postoperatively to help keep stools soft for passage but laxatives may cause irritation and trauma to the anorectal area and are not used postoperatively. Early passage of a bowel movement, although painful, is encouraged to prevent drying and hardening of stool, which would result in an even more painful bowel movement.

When a patient returns to the clinical unit after an abdominal-perineal resection (APR), what should the nurse expect? a. An abdominal dressing c. A temporary colostomy and drains b. An abdominal wound and drains d. A perineal wound, drains, and a stoma

d. With an abdominal perineal-resection (APR), an abdominal incision is made and the proximal sigmoid colon is brought through the abdominal wall and formed into a permanent colostomy. The patient is repositioned, a perineal incision is made, and the distal sigmoid colon, rectum, and anus are removed through the perineal incision, which may be left open, packed, and have drains.

A nursing intervention that is most appropriate to decrease post-operative edema and pain after an inguinal herniorrhaphy is: a. applying a truss to the hernia site b. allowing the patient to stand to void c. supporting the incision during coughing d. applying a scrotal support with ice bag

d. applying a scrotal support with ice bag Rationale: Scrotal edema is a painful complication after an inguinal hernia repair. Scrotal support with application of an ice bag may help relieve pain and edema.

PD dialysis solution

- 1 or 2 L plastic bags w/ glucose concentrations of 1.5%, 2.5%, & 4.25% - electrolyte composition similar to plasma - warmed to body temp

inflow

- 1st phase of PD cycle - prescribed amt of sol (usually 2L) is infused thru catheter over ~ 10 mins - flow rate may be decreased if pt has pain - after infused, inflow clamp closed to prevent air from entering tubing

Continual Renal Replacement Therapy (CRRT)

- 2 types: venous & arterial access - Alternative or adjunctive method for treating AKI - Uremic toxins & fluids are removed - Acid base status/electrolyte balance adjusted slowly & continuously - Pts selected for this therapy are usually those who do not respond to dietary interventions and drug therapy - Can be used in conjunction w/ HD - *Contraindicated: presence of uremia* - Continued for 30-40 days - Hemofilter changed q24-48hrs - Ultrafiltrate should be clear yellow - ultrafiltration rate (UFR) may range from 0 to 500 mL/hr - ultrafiltrate drains out of the hemofilter, fluid and electrolyte replacements can be infused into the infusion port

effectiveness & adaptation of HD

- Cannot fully replace normal functions of kidneys - Can ease many of the symptoms - Can prevent certain complications

CRRT vs HD

- Continuous rather than intermittent - Solute removal by convection (no dialysate required) in addition to osmosis and diffusion - Less hemodynamic instability - Does not require constant monitoring by HD nurse - Does not require complicated HD equipment

phases of PD cycle

- Inflow (fill) - Dwell (equilibration) - Drain

continuous ambulatory peritoneal dialysis (CAPD)

- Manual exchange dialysis - Exchanges are carried out manually w/ 1.5-3 L of peritoneal dialysate at least 4 times daily, with dwell times averaging 4 hrs

osmosis

- Movement of fluid from an area of lesser concentration of solutes to area of greater concentration - Occurs when glucose is added to dialysate; creates osmotic gradient across membrane -> pulls excess fluid from blood

diffusion

- Movement of solutes from an area of greater concentration to an area of lesser concentration - how dialysis works

hernias

- PD complication - can occur bc of increased intraabdominal pressure secondary to the dialysate infusion - can develop in predisposed individuals -> multiparous women, older men

peritonitis

- PD complication - most frequently occurs bc of improper technique in making or breaking connections for exchanges

pulmonary

- PD complication. ______ complications like atelectasis, pneumonia, and bronchitis may occur from repeated upward displacement of the diaphragm, resulting in decreased lung expansion

ultrafiltration

- Water and fluid removal during PD - Results when there is an osmotic gradient across the membrane - most effective osmotic agent = glucose

Dialysis

- a technique in which substances move from blood through a semipermeable membrane and into a dialysis solution (dialysate). - used to correct fluid/electrolyte imbalances and to remove waste products in renal failure - treat drug overdoses - begun when pts uremia cannot be managed - initiated when GFR (or creat clearance is < 15 mL/min 2 methods: Peritoneal dialysis (PD) & Hemodialysis (HD)

Automated peritoneal dialysis (APD)

- most popular form of PD - allows pt to accomplish dialysis while they sleep - cycler delivers dialysate - machine cycles 4+ exchanges per night -times & controls fill, dwell & drain - 1 or 2 daytime manual exchanges may also be prescribed to ensure adequate dialysis

HD access

- obtaining vascular access = diff prob Types of access: - Arteriovenous fistulas (AVFs) and grafts - Temporary vascular access

dialysate

- the fluid used in dialysis to remove impurities - purity is monitored & controlled bc it can pass into pts blood

Arteriovenous fistulas (AVFs)

- this type of HD access has the best overall patency rates & least amt of complications (like thrombosis, infection) - subcutaneous AVF most commonly created in forearm w/ anastomosis btwn artery & vein - should be placed @ least 3 monts b4 need to initiate HD - normally a thrill can be palpated & bruit can be heard

Continuous venovenous hemofiltration (CVVH)

- type of continual renal replacement therapy (CRRT) - lg vols of fluid removed hrly then replaced - fluid replacement dependent on stability/individualized needs of pt

A client with chronic kidney disease has been on hemodialysis for two years. The client relates to a nurse in the dialysis unit in an angry, critical manner and frequently does not follow the prescribed diet or take prescribed medications. What does the nurse identify as the most likely underlying cause of this behavior?

A defense against underlying depression and fear

A client with chronic renal failure has been on hemodialysis for two years. The client communicates with the nurse in the dialysis unit in an angry, critical manner and is frequently noncompliant with medications and diet. The nurse can best intervene by first considering that the client's behavior is most likely:

A defense against underlying depression and fear

What does the nurse determine is the most likely cause of renal calculi in clients with paraplegia?

Accelerated bone demineralization

The nurse is providing care to a client who is being treated for bacterial cystitis. Before discharge, it is most important for the client to:

Achieve relief of symptoms and to maintain kidney function

A client with acute kidney failure becomes confused and irritable. Which does the nurse determine is the most likely cause of this behavior?

An increased blood urea nitrogen level

For which clinical indicator should a nurse monitor a client with end-stage renal disease?

Azotemia Reason Why: Azotemia is an increase in nitrogenous waste (particularly urea) in the blood, which is common with end-stage renal disease. Excessive nephron damage in end-stage renal disease causes oliguria, not polyuria; excessive urination is common in early kidney insufficiency because of the inability to concentrate urine. Jaundice is common to biliary obstruction, not to end-stage renal disease. The blood pressure may be elevated as a result of hypervolemia associated with increased total body fluid.

Which clinical manifestations does a nurse expect that a client with renal calculi might report? (Select all that apply.)

Blood in the urine, Frequency and urgency of urination

A nurse concludes that the anemia that accompanies chronic kidney disease should be treated because it contributes to:

Chronic fatigue

A client with the diagnosis of chronic kidney disease develops hypocalcemia. Which clinical manifestations should the nurse expect the client with hypocalcemia to exhibit? (Select all that apply.)

Chvostek sign, Muscle cramps

A client arrives at a health clinic reporting hematuria, frequency, urgency, and pain on urination. What does the nurse suspect is the cause of these signs and symptoms?

Cystitis

Which clinical manifestation should a nurse expect a client with diabetes insipidus to exhibit?

Decreased urine osmolarity

After a successful kidney transplant for a client with end-stage kidney disease, the nurse anticipates that laboratory studies will demonstrate:

Decreasing serum creatinine

A nurse assesses a newly admitted client with renal colic to determine the signs and symptoms that are present. The nurse assesses the client for which primary subjective symptom?

Flank discomfort

The nurse is caring for a client with a diagnosis of acute kidney failure associated with drug toxicity. When the client complains of thirst, the nurse should offer:

Hard candy

A client with chronic kidney disease is scheduled to begin peritoneal dialysis. When discussing the procedure, the nurse explains that the purpose of the dialysis is to:

Help do some of the work usually done by the kidneys

A client is diagnosed as having kidney failure. During the oliguric phase the nurse should assess the client for:

Hyperkalemia

A nurse is caring for a client with a diagnosis of chronic kidney failure who has just been told by the health care provider that hemodialysis is necessary. Which clinical manifestation indicates the need for hemodialysis?

Hyperkalemia

Which is the most serious complication for which the nurse must monitor a client with kidney failure?

Hyperkalemia

A nurse anticipates that dialysis will be necessary for a 12-year-old child with chronic kidney disease when the child begins to exhibit:

Hypervolemia

HD complications

Hypotension Muscle cramps Loss of blood Hepatitis

The nurse providing postoperative care for a client who had kidney surgery reviews the client's urinalysis results. The nurse concludes that the presence of what substance in the urine needs to be reported to the health care provider?

Large proteins

A nurse is caring for a client with chronic kidney failure. Which clinical findings should the nurse expect when assessing this client? (Select all that apply.)

Lethargy, Muscle twitching

during HD

Nurse should be alert to changes in condition; Measure vital signs every 30 to 60 minutes

When a client returns from the postanesthesia care unit after a kidney transplant, the nurse should plan to measure the client's urinary output every:

One hour

A client who is 5 feet, 8 inches tall and weighs 220 pounds is admitted to the hospital with ureteral colic, blood in the urine, and a blood pressure of 150/90. The immediate objective of nursing care for this client is to decrease:

Pain

A client is admitted to the hospital with a diagnosis of chronic kidney disease. Which responses should the nurse expect the client to exhibit? (Select all that apply.)

Paresthesias, Hypertension

A nurse is caring for a client with chronic kidney failure. What should the nurse teach the client to limit the intake of to help control uremia associated with end stage renal disease (ESRD)?

Protein

A nurse is caring for a client who had a kidney transplant. Which test is most important for determining whether a client's newly transplanted kidney is working effectively?

Serum creatinine

A nurse is providing postoperative care to a client who had a kidney transplant. What assessment is the best indicator of the functioning of the newly transplanted kidney?

Serum creatinine

benefits of PD

Short training program Independence Ease of traveling Fewer dietary restrictions Greater mobility than with HD

ESKD

Treated w/ dialysis bc of: - lack of organ donors - some ppl are physically or mentally unsuitable for transplant - some ppl do not want transplant

A nurse is caring for a client with acute kidney failure who is receiving a protein-restricted diet. The client asks why this diet is necessary. What information should the nurse include in a response to the client's questions?

This supplies only essential amino acids, reducing the amount of metabolic waste products, thus decreasing stress on the kidneys.

A lithotripsy to break up renal calculi is unsuccessful, and a nephrolithotomy is performed. Which postoperative clinical indicator should the nurse report to the health care provider?

Urine output of 20 to 30 mL/hr

A client who has a renal mass asks the nurse why an ultrasound has been scheduled, as opposed to other diagnostic tests that may be ordered. The nurse formulates a response based on the understanding that: a) all other tests are more invasive than an ultrasound b) all other tests require more elaborate postprocedure care c) an ultrasound can differentiate a solid mass from a fluid-filled cyst d) an ultrasound is much more cost effective than other diagnostic tests

c) an ultrasound can differentiate a solid mass from a fluid-filled cyst

ultrafiltrate

collective term for plasma water and nonprotein solutes that are removed by hemofilter during CRRT

15

dialysis is initiated when GFR (or creatinine clearance) is < 15mL/min

A client undergoes total gastrectomy. Several hours after surgery, the nurse notes that the client's nasogastric (NG) tube has stopped draining. How should the nurse respond?

1. Notify the physician.

HD procedure

- use 2 large bore needles (14-16 gauge) - insert them into fistula or graft to obtain vascular access - One needle is placed to pull blood from the circulation to the HD machine - The other needle is used to return the dialyzed blood to the patient - Heparin is added to blood as it flows into the dialyzer so blood doesn't clot - Dialyzer/blood lines primed with saline solution to eliminate air - Terminated by flushing dialyzer with saline to remove all blood - Needles removed and firm pressure applied

Tenckhoff catheter

- used for PD - about 60cm long - has 1 or 2 polyester (Dacron) cuffs on its subcutaneous and peritoneal portions - cuffs act as anchors & prevent the migration of microorganisms down the shaft from the skin - after insertion: skin is cleaned w/ antiseptic sol & steril dressing applied - irrigated immediately

dialyzers

- used in HD - are long plastic cartridges that contain thousands of parallel hollow tubes or fibers - fibers are semipermeable membranes - the blood is pumped into the top of the cartridge and is dispersed into all of the fibers - dialysate is pumped into btm of the cartridge & bathes the outside of the fibers

--

--

One hour before a client is to undergo abdominal surgery, the physician orders atropine, 0.3 mg I.M. The client asks the nurse why this drug must be administered. How should the nurse respond?

1. "Atropine decreases salivation and gastric secretions."

A client admitted for treatment of a gastric ulcer is being prepared for discharge on antacid therapy. Discharge teaching should include which instruction?

1. "Continue to take antacids, even if your symptoms subside."

A client with extreme weakness, pallor, weak peripheral pulses, and disorientation is admitted to the emergency department. His wife reports that he has been "spitting up blood." A Mallory-Weiss tear is suspected, and the nurse begins taking a client history from the client's wife. The question by the nurse that demonstrates her understanding of Mallory-Weiss tearing is:

1. "Tell me about your husband's alcohol usage."

A nurse is assigned to care for four clients. Which client should a nurse assess first?

1. A postoperative client who just returned from surgery and is vomiting

Why are antacids administered regularly, rather than as needed, to treat peptic ulcer disease?

1. To keep gastric pH at 3.0 to 3.5

A client comes to the emergency department with suspected cholecystitis. Which data collection findings are characteristic of this diagnosis?

1. Transient epigastric pain radiating to the back and right shoulder, 2. Burning in the chest after eating fried foods, 3. Flatulence, 4. Nausea

When collecting data on a client during a routine checkup, the nurse reviews the history and notes that the client had aphthous stomatitis at the time of the last visit. Aphthous stomatitis is best described as:

1. a canker sore of the oral soft tissues.

A client had a nephrectomy 2 days ago and is now complaining of abdominal pressure and nausea. The first nursing action should be to:

1. auscultate bowel sounds.

The nurse is caring for a client with a colostomy. The client tells the nurse that he makes small pin holes in the drainage bag to help relieve gas. The nurse should teach him that this action:

1. destroys the odor-proof seal.

A client who can't tolerate oral feedings begins receiving intermittent enteral feedings. When monitoring for evidence of intolerance to these feedings, the nurse must stay alert for:

1. diaphoresis, vomiting, and diarrhea.

A client is scheduled for an endoscopy. On admission, the nurse asks the client if he has an advance directive, and the client states, "No." What should the nurse do next?

2. Provide the client with information about an advance directive.

During a client-teaching session, which instruction should the nurse give to a client receiving kaolin and pectin (Kaopectate) for treatment of diarrhea?

4. "Drink 8 to 13 8-oz glasses (2 to 3 L) of fluid daily."

To prevent gastroesophageal reflux in a client with hiatal hernia, the nurse should provide which discharge instruction?

2. "Avoid coffee and alcoholic beverages."

The nurse is teaching an elderly client about good bowel habits. Which statement by the client would indicate to the nurse that additional teaching is required?

2. "I need to use laxatives regularly to prevent constipation."

A client who is about to undergo gastric bypass surgery calls the nurse into the room. The client says she's concerned that friends will learn about her upcoming surgery. She pleads with the nurse to keep her surgery a secret. Which response by the nurse is best?

2. "I'm not at liberty to discuss your case with anyone except those directly involved in your care unless you authorize me to do so."

The nurse must administer an enema to an adult client. The appropriate distance for inserting an enema into an average-sized adult is:

2. 3" to 4".

A client is diagnosed with a hiatal hernia. Which statement indicates effective client teaching about hiatal hernia and its treatment?

4. "I'll eat frequent, small, bland meals that are high in fiber."

A client who has just been diagnosed with hepatitis A asks, "How could I have gotten this disease?" What is the nurse's best response?

4. "You may have eaten contaminated restaurant food."

A client with a peptic ulcer is about to begin a therapeutic regimen that includes a bland diet, antacids, and ranitidine (Zantac). Before the client is discharged, the nurse should provide which instruction?

3. "Avoid aspirin and products that contain aspirin."

The nurse is providing dietary instructions to a client with a history of pancreatitis. Which instruction is correct?

3. "Maintain a high-carbohydrate, low-fat diet."

A client with acute diarrhea is prescribed paregoric, 5 ml by mouth up to four times daily, until the diarrhea subsides. The client asks the nurse how soon the medication will start to work after the first dose is taken. How should the nurse respond?

3. "Within 1 hour"

After checking the client's chart for possible contraindications, the nurse is administering meperidine (Demerol), 50 mg I.M., to a client with pain after an appendectomy. Which type of drug would contraindicate the use of meperidine?

3. A monoamine oxidase (MAO) inhibitor

A client is admitted with increased ascites related to cirrhosis. Which nursing diagnosis should receive top priority?

3. Ineffective breathing pattern

A nurse approaches a client with an 0800 dose of his scheduled pancreatin. The client states, "I'm not going to take that medicine. It makes me nauseated." What should the nurse do first?

3. Instruct the client about the benefit of taking the medication.

A client with pancreatitis has been receiving total parenteral nutrition (TPN) for the past week. Which nursing intervention helps determine if TPN is providing adequate nutrition?

3. Monitoring the client's weight every day

A client with Crohn's disease is admitted to a semiprivate room late in the afternoon. The next day, the client reports that he was not able to sleep during the night because the hallway lights bothered him. He asks that he be moved to a bed next to a window. What should the nurse do?

3. Move him to the next available window-side bed.

A client with abdominal pain secondary to a malignant mass in the colon is receiving fentanyl (Duragesic) by transdermal patch. His current patch expires in 48 hours and he reports a pain level of 8 on a 1-to-10 scale. What should a nurse do?

3. Notify the client's physician.

A nursing assistant is assisting a nurse with feeding clients. Which client should the nurse assign to the nursing assistant?

4. A client with bilateral blindness

A 53-year-old client undergoes colonoscopy for colorectal cancer screening. A polyp was removed during the procedure. Which nursing interventions are necessary when caring for the client immediately after colonoscopy?

3. Observe the client closely for signs and symptoms of bowel perforation., 4. Monitor vital signs frequently until they're stable., 5. Inform the client that there may be blood in his stool and that he should report excessive blood immediately.

The nurse should expect to administer which vaccine to the client after a splenectomy?

3. Pneumovax 23

The nurse is caring for a client with cirrhosis. Which data collection findings indicate that the client has deficient vitamin K absorption caused by this hepatic disease?

3. Purpura and petechiae

Which of the following is a warning sign of colon cancer?

3. Rectal bleeding

A client is admitted to the emergency department with complaints of double vision, difficulty swallowing, dry mouth, and muscle weakness. A nurse also observes that the client has drooping eyelids and slurred speech. He states that he recently ate home-canned green beans. The nurse suspects exposure to botulism. What type of infection control precaution is necessary?

3. Standard precautions

A client comes to the emergency department complaining of acute GI distress. When obtaining the client's history, the nurse inquires about his family history. Which disorder has a familial basis?

3. Ulcerative colitis

An elderly client with Alzheimer's disease begins supplemental tube feedings through a gastrostomy tube to provide adequate calorie intake. The nurse should be concerned most with the potential for:

3. aspiration.

A client is admitted to the health care facility with a diagnosis of a bleeding gastric ulcer. The nurse expects this client's stools to be:

3. black and tarry.

The physician orders a stool culture to help diagnose a client with prolonged diarrhea. The nurse who obtains the stool specimen should:

3. collect the specimen in a sterile container.

A client with cholecystitis is receiving propantheline bromide (Pro-Banthine). The client is given this medication because it:

3. inhibits contraction of the bile duct and gallbladder.

A client with recent onset of epigastric discomfort is scheduled for an upper GI series (barium swallow). When teaching the client how to prepare for the test, which instruction should the nurse provide?

4. "Avoid eating or drinking anything for 6 to 8 hours before the test."

A nurse is working with a nursing assistant, who is given the task of calculating three clients' intake and output at the end of the shift. When the nurse reviews the nursing assistant's work, she discovers inaccuracies in the nursing assistant's results. What should the nurse do?

4. Ask the nursing assistant to show her how she determined the results.

Which food should be included in a client's diet during the first 6 to 8 weeks after ileostomy surgery?

4. Banana

One year ago, a client was diagnosed with cirrhosis of the liver caused by alcohol abuse. Since then, he has been noncompliant with the prescribed protein-restricted diet. After a friend finds him semiconscious at home, the client is admitted to the hospital. When initial laboratory test results show an elevated ammonia level, he's diagnosed with hepatic encephalopathy. The physician prescribes lactulose (Cephulac), 200 g diluted in 700 ml of tap water, given as a retention enema every 4 hours. For which other condition is lactulose prescribed?

4. Constipation

The nurse knows the patient with AKI has entered the diuretic phase when what assessments occur? Select all that apply. A. Dehydration B. Hypokalemia C. Hypernatremia D. BUN increases E. Serum creatinine increases

A, B. Dehydration, hypokalemia, and hyponatremia occur in the diuretic phase of AKI because the nephrons can excrete wastes but not concentrate urine. Therefore the serum BUN and serum creatinine levels also begin to decrease.

In addition to urine function, the nurse recognizes that the kidneys perform numerous other functions important to the maintenance of homeostasis. Which physiologic processes are performed by the kidneys? Select all that apply. A. Production of renin B. Activation of vitamin D C. Carbohydrate metabolism D. Erythropoietin production E. Hemolysis of old red blood cells (RBCs)

A, B, D. In addition to urine formation, the kidneys release renin to maintain blood pressure, activate vitamin D to maintain calcium levels, and produce erythropoietin to stimulate RBC production. Carbohydrate metabolism and hemolysis of old RBCs are not physiologic functions that are performed by the kidneys.

2. A patient who has been NPO during treatment for nausea and vomiting caused by gastric irritation is to start oral intake. Which of these should the nurse offer to the patient? a. A glass of orange juice b. A dish of lemon gelatin c. A cup of coffee with cream d. A bowl of hot chicken broth

ANS: B Clear liquids are usually the first foods started after a patient has been nauseated. Acidic foods such as orange juice, very hot foods, and coffee are poorly tolerated when patients have been nauseated.

A patient who has a wound infection after major surgery has only been taking in about 50% to 75% of the ordered meals and states, "Nothing on the menu really appeals to me." Which action by the nurse will be most effective in improving the patient's oral intake? a. Make a referral to the dietician. b. Order at least six small meals daily. c. Teach the patient about high-calorie, high-protein foods. d. Have family members bring in favorite foods from home.

ANS: D The patient's statement that the hospital foods are unappealing indicates that favorite home-cooked foods might improve intake. The other interventions also may help improve the patient's intake, but the most effective action will be to offer the patient more appealing foods.

Which patient should be taught preventive measures for CKD by the nurse because this patient is most likely to develop CKD? A. A 50-year-old white female with hypertension B. A 61-year-old Native American male with diabetes C. A 40-year-old Hispanic female with cardiovascular disease D. A 28-year-old African American female with a urinary tract infection

B. A 61-year-old Native American male with diabetes It is especially important for the nurse to teach CKD prevention to the 61-year-old Native American with diabetes. This patient is at highest risk because diabetes causes about 50% of CKD. This patient is the oldest, and Native Americans with diabetes develop CKD 6 times more frequently than other ethnic groups. Hypertension causes about 25% of CKD. Hispanics have CKD about 1.5 times more than non-Hispanics. African Americans have the highest rate of CKD because hypertension is significantly increased in African Americans. A UTI will not cause CKD unless it is not treated or UTIs occur recurrently.

Which nursing intervention is most appropriate in providing care for an adult patient with newly diagnosed adult onset polycystic kidney disease (PKD)? A. Help the patient cope with the rapid progression of the disease. B. Suggest genetic counseling resources for the children of the patient. C. Expect the patient to have polyuria and poor concentration ability of the kidneys. D. Implement appropriate measures for the patient's deafness and blindness in addition to the renal problems.

B. Suggest genetic counseling resources for the children of the patient. PKD is one of the most common genetic diseases. The adult form of PKD may range from a relatively mild disease to one that progresses to chronic kidney disease. Polyuria, deafness, and blindness are not associated with PKD.

A nurse teaches a client with calcium-based renal calculi about foods that can be eaten on a low-calcium diet (400 mg/day). The nurse concludes that the teaching was effective when the client selects what food items from the menu? (Select all that apply.)

Baked chicken, Roast beef with mashed potato

A client is scheduled for computed tomography (CT) of the kidneys to rule out renal disease. As an essential preprocedure component of the nursing assessment, the nurse plans to ask the client about a history of: a) familial renal disease b) frequent antibiotic use c) long-term diuretic therapy d) allergy to shellfish or iodine

d) allergy to shellfish or iodine

The nurse instructs a 50-year-old woman about cholestyramine to reduce pruritis caused by gallbladder disease. Which statement by the patient to the nurse indicates she understands the instructions? A. "This medication will help me digest fats and fat-soluble vitamins." B. "I will apply the medicated lotion sparingly to the areas where I itch." C. "The medication is a powder and needs to be mixed with milk or juice." D. "I should take this medication on an empty stomach at the same time each day."

C. "The medication is a powder and needs to be mixed with milk or juice." For treatment of pruritus, cholestyramine may provide relief. This is a resin that binds bile salts in the intestine, increasing their excretion in the feces. Cholestyramine is in powder form and should be mixed with milk or juice before oral administration.

A normal physical assessment finding of the GI system is/are (select all that apply) a. nonpalpable liver and spleen. b. borborygmi in upper right quadrant. c. tympany on percussion of the abdomen. d. liver edge 2 to 4 cm below the costal margin. e. finding of a firm, nodular edge on the rectal examination.

Correct answers: a, c Rationale: Normal assessment findings for the gastrointestinal system include a nonpalpable liver and spleen and generalized tympany on percussion. Normally, bowel sounds are high pitched and gurgling; loud gurgles indicate hyperperistalsis and are called borborygmi (stomach growling). If the patient has chronic obstructive pulmonary disease, large lungs, or a low-set diaphragm, the liver may be palpated 0.4 to 0.8 inch (1 to 2 cm) below the right costal margin. On palpation, the rectal wall should be soft and smooth and should have no nodules.

A nurse is caring for a client who is scheduled for cystoscopy. What should the nurse include in the client's postprocedure teaching plan?

Increase fluid intake for three to four days postoperatively

A client with acute renal failure moves into the diuretic phase after one week of therapy. For which signs during this phase should the nurse assess the client? (Select all that apply.)

Dehydration, Hypovolemia

A nurse is notified that the latest potassium level for a client in acute renal failure is 6.2 mEq. What action should the nurse take?

Take vital signs and notify the primary health care provider

Locate the abdominal quadrant where the nurse would expect to palpate the liver.

The liver is located in the right upper abdominal quadrant.

Which of the following is an expected finding in the client with chronic renal failure? a) anemia b) polyuria c) increased creatinine clearance d) increased serum calcium levels

a) anemia


Related study sets

Mastering A&P: Chapter 1 End of Chapter Review

View Set

Chapter 12: Imports, Customs, and Tariff Law

View Set

OB course point practice questions for Exam 1

View Set

AP Environmental Science Non/Renewable Resources Test Practice [<+))><< <*))>=<]

View Set

Intro to Sociology Final (Chapter 9 - Week 8)

View Set